BASIC SCIENCE ORTHOPEDIC MCQS ONLINE BANK

What assay is commonly used to examine the level of messenger RNA (mRNA) of a particular gene? 

 

  1. Western blot

  2. Southern blot

  3. Reverse transcription polymerase chain reaction (RT-PCR)

  4. Immunocytochemical analysis

  5. Enzyme-linked immunosorbent assay (ELISA) PREFERRED RESPONSE 3

RT-PCR is the only method to quantify the level of mRNA inside the cells. The western blot is an assay to detect protein expression level inside the cells. ELISA is a biochemical technique used mainly in immunology to detect the presence of an antibody or an antigen in a sample. Immunocytochemical analysis is a technique to stain cells using antibody to localize a certain protein within the cells. Southern blot is a technique to detect genomic DNA expression of a certain gene.

 

(SAE13BS.20) During endochondral ossification of the growth plate, the process that most contributes to the longitudinal growth of long bones is 

 

  1. chondrocyte apoptosis.

  2. chondrocyte hypertrophy.

  3. chondrocyte proliferation.

  4. growth plate matrix synthesis. PREFERRED RESPONSE 2

The growth plate is divided into 5 distinct zones: reserve, proliferative, maturation, hypertrophy, and vascular invasion. During growth-plate chondrocyte hypertrophy, intracellular volume and an increase in chondrocyte height are responsible for most growth of long bones. Other factors that contribute to bone growth are chondrocyte proliferation and matrix synthesis, but to a lesser degree than chondrocyte hypertrophy. Growth plate chondrocytes undergo programmed cell death (apoptosis) after hypertrophy takes place.

(SAE08UE.118) What is the most common complication following interscalene nerve block for shoulder surgery? 

 

  1. Temporary paresthesia to the affected arm and hand for up to 6 months

  2. Temporary motor weakness of the affected arm for up to 6 months

  3. Pneumothorax

  4. Seizures

  5. Complex regional pain syndrome PREFERRED RESPONSE 1

All of these complications have been documented after interscalene nerve block. Other serious complications such as cardiac arrest and respiratory distress have also been noted. However, the most common complication after interscalene nerve block appears to be temporary paresthesia to the hand that can occur in up to 2.3% of the patients.

 

(SAE08PA.4) A healthy 52-year-old woman is seeking professional advice about management of osteoporosis. She has no risk factors for osteoporosis. What is the best recommendation for bone health for this patient? 

 

  1. Bone mineral density testing performed semiannually

  2. No treatment

  3. A healthy diet high in calcium

  4. 1,000 to 1,500 mg calcium supplement plus 400 to 800 IU vitamin D per day

  5. Estrogen therapy

 

PREFERRED RESPONSE 4

 

Women older than age 50 years should receive daily supplementation with calcium and vitamin D to help preserve bone density. Bone mineral density testing is recommended for women age 65 years or older and postmenopausal women with at least one risk factor for osteoporotic fractures: prior fragility fracture, low estrogen levels, premature menopause, long-term secondary amenorrhea, glucocorticoid therapy, maternal history of hip fracture, or low body mass index. Hormone therapy is not approved for the treatment of osteoporosis.

(SAE13BS.93) A 48-year-old man who is scheduled to undergo total knee replacement has an X-linked clotting disorder that leads to abnormal bleeding and recurrent, spontaneous hemarthrosis. Before undergoing surgery, he should have replacement therapy of 

 

  1. protein C and S.

  2. vitamin K.

  3. von Willebrand factor.

  4. factor VIII.

 

PREFERRED RESPONSE 4

 

Hemophilia A is an X-linked recessive deficiency of factor VIII that can lead to significant bleeding problems including recurrent spontaneous hemarthroses that can lead to synovitis and joint destruction. von Willebrand disease is a lack of von Willebrand factor that leads to decreased platelet aggregation; more commonly patients have mucosal bleeding and not hemarthroses. Vitamin K deficiency is not hereditary; it is typically attributable to inadequate dietary intake, malabsorption, and loss of storage sites from hepatocellular disease. Protein C and S deficiencies are autosomal-dominant diseases that lead to thrombosis, not bleeding, as protein C and S shut off thrombin formation.

 

(SAE07SM.94) Storage of musculoskeletal allografts by cryopreservation is achieved by 

 

  1. replacing water in the tissue with alcohol to a moisture level of 5% and then using a vacuum process to remove the alcohol from the tissue.

  2. maintaining maximum cellular viability of fresh tissue without long-term storage.

  3. using chemicals to remove cellular water and controlled rate freezing to prevent ice crystal formation.

  4. freezing the graft twice and packaging the tissue without solution at minus 80 degrees C.

  5. freezing the graft in water without an antibiotic solution soak during quarantine, with final storage in liquid nitrogen.

 

PREFERRED RESPONSE 3

 

Cryopreservation uses chemicals to remove cellular water and controlled rate freezing to prevent ice crystal formation. The tissue is procured, cooled to wet ice temperature for quarantine, and then stored in a container with cryoprotectant solution of dimethyl

sulfoxide or glycerol which displaces the cellular water. The controlled rate freezing is then done to prevent ice crystal formation. Fresh allografts are not frozen in order to maintain maximum cellular viability, and this process limits the shelf life of osteochondral allografts. Freeze-drying involves replacement of water in the tissue with alcohol to a moisture level of 5% and then uses a vacuum process to remove the alcohol from the tissue. Preparation of fresh frozen grafts involves freezing the graft twice and packaging the tissue without solution at minus 80 degrees C.

 

(SAE13BS.19) A patient sustains a grade III medial collateral ligament injury. One year later, when compared to collagen in an uninjured ligament, an increase is likely in the 

 

  1. gross number of fibers.

  2. proportion of type III fibers.

  3. cross-linking.

  4. mass and diameter of fibers. PREFERRED RESPONSE 1

Studies on animal models have shown that there is a change in collagen fiber type and distribution early in the healing process. There is a higher portion of type III fibers than in normal ligament initially, but this ratio returns to normal about 1 year after the injury occurs. Healing ligaments show an increased number of collagen fibers, but the number of mature collagen cross-links is 45% of predicted value after 1 year. There is also a decrease in the mass and diameter of the collagen fibers.

 

(OBQ13.100) A 32-year-old runner sustains a trimalleolar left ankle fracture. She undergoes open reduction and internal fixation and is kept non-weightbearing after surgery. At 2 months, what changes will occur in the articular cartilage of both her knees as a result of her current weightbearing regimen? 

 

  1. Cartilage thickening in the left (ipsilateral) knee and no change in cartilage thickness in the right (contralateral) knee

  2. Cartilage thinning in both knees

  3. Cartilage thinning in the left (ipsilateral) knee and no change in cartilage thickness in the right (contralateral) knee

  4. Cartilage thinning in the left (ipsilateral) knee and increased cartilage thickness in the right (contralateral) knee

  5. Increased cartilage thickness in both knees PREFERRED RESPONSE 3

After a period of off-loading, the off-loaded limb will experience cartilage thinning. The contralateral limb will not demonstrate any cartilage changes.

 

Physiologic loading of cartilage increases proteoglycan synthesis and cell proliferation and is chondroprotective. Joint immobilization leads to cartilage thinning, tissue softening, and reduced proteoglycan content, leading to cartilage erosion. Joint overuse leads to cartilage damage (in vitro only).

 

Hinterwimmer et al. examined cartilage atrophy after partial load bearing using quantitative MRI. They found cartilage thinning in all knee compartments (greatest thinning, medial tibia; least thinning, patella). There was no change in cartilage morphology in the contralateral knee.

 

Sun reviewed the relationship between mechanical loading and cartilage degeneration. In OA, cartilage breakdown occurs at the articular surface, and is then fueled by synovial proteases and cytokines. In RA, synovial cells and macrophages are the source of degradative enzymes and incite cartilage destruction.

 

Milward-Sadler et al. examined mRNA levels following mechanical stimulation in normal and osteoarthritic chondrocytes. Normal chondrocytes showed increased aggrecan mRNA and decreased matrix metalloproteinase 3 (MMP-3) mRNA after stimulation. This chondroprotective response was absent in osteoarthritic chondrocytes.

 

Illustration A shows pro- and anti-inflammatory mechanisms of mechanical loading on chondrocytes. Underloading and overloading induce cartilage damage through pathways involving the upregulation of MMPs and ADAMTSs (ADAMTS, a disintegrin and metalloproteinase with thrombospondin motifs, or aggrecanase). Physiological loading blocks these increases.

 

Incorrect Answers:

Answer 1: Cartilage THINNING (not thickening) will occur on the offloaded limb (left).

Answers 2 and 5: There will be a difference in cartilage thickness between knees as a result of different weightbearing status on both lower extremities. Answer 4: Noticeable cartilage hypertrophy does not occur on the uninjured limb.

(SAE08UE.112) A 65-year-old patient is undergoing a right total shoulder replacement. He has been on long-standing narcotics for chronic pain. He has pulmonary disease, diabetes, coronary heart disease and BMI=31. His medication list includes a corticosteroid inhaler and aspirin. A pre-operative ECG work up showed first degree heart block. Lung function testing revealed a forced expiratory volume (FEV1) of 45%. What would be a relative contraindication to an interscalene nerve block for this patient? Review Topic

 

  1. Age

  2. Medications

  3. Obesity

  4. Forced expiratory volume (FEV1) of 45%

  5. First degree heart block PREFERRED RESPONSE 4

FEV1=45% is classified as severe obstructive disease. Severe lung disease is considered a contraindication to interscalene nerve block for shoulder surgery.

 

A common side effect of interscalene nerve block for shoulder surgery is the blockade of the ipsilateral phrenic nerve. If this was to occur, paresis of the diaphragm can result in up to 30% reduction in pulmonary function volumes. As a result, interscalene nerve block are generally is not recommended for patients with moderate or severe lung disease. Other contraindications include allergy to local anesthetics, contralateral phrenic nerve injury, infection at the injection site, uncontrolled seizure disorder, coagulation abnormality, and preexisting neurologic injury.

 

Misamore et al. assessed the safety and efficacy of interscalene brachial plexus block anesthesia when performed on patients who were anesthetized with a general anesthetic prior to the performance of the block. Persistent neurological complications occurred in 4.4% (40 of 910) of patients, and long-term neurologic complications occurred in 0.8% (8 of 910).

 

Incorrect Answers:

Answer 1: Age is not a contraindication Answer 2: Neuraxial blocks can be safely performed on patients taking aspirin or NSAIDs

Answer 3: Obesity alone is not a contraindication, unless it is the cause of restrictive lung disease.

Answer 5: Heart disease is not a contraindication

(SAE12SN.68) Which of the following actions increases radiation exposure to patients and personnel when using fluoroscopy? 

 

  1. The use of lead glasses, thyroid shield, and a lead apron with a equivalent lead thickness of 0.25 mm

  2. Orienting the cathode ray tube beneath the patient with the image intensifier receptor as close to the patient as possible

  3. Limiting the beam on time to only what is clinically important

  4. The use of continuous fluoroscopy to ensure proper placement of implants

  5. Orienting the beam in the opposite direction of the working team and keeping the team outside a 6-foot radius from the fluoroscopy machine

 

PREFERRED RESPONSE 4

 

Continuous fluoroscopy and cineradiography exposes the patient and personnel to markedly increased levels of direct and scatter radiation exposure. Continuous fluoroscopy should be limited to only what is absolutely needed for safe completion of the procedure. By orienting the cathode ray tube beneath the patient and placing the image intensifier as close as clinically possible to the patient, scatter radiation exposure to the personnel is minimized.

 

(SAE10PE.3) A 9-year-old girl has had bilateral knee and leg pain for the past 2 years. The family has noted increasing deformity in both lower extremities. She is less than the fifth percentile for height. Examination reveals bilateral femoral bowing, mild medial-lateral laxity of the knees, and the deformities shown in the radiograph seen in Figure 3. What is the most likely diagnosis? Review Topic

 

 

  1. Renal osteodystrophy

  2. Diastrophic dysplasia

  3. Metaphyseal dysplasia

  4. Osteogenesis imperfecta

  5. Fibrous dysplasia

 

PREFERRED RESPONSE 1

 

The widening, bowing, and cupping of the physes indicate some form of metabolic bone disease; therefore, the most likely diagnosis is renal osteodystrophy. The age of onset makes X-linked hypophosphatemic rickets less likely. The ground glass lesions and widening of the medullary canal characteristic of fibrous dysplasia are not present. There are no fractures creating the deformities indicating osteogenesis imperfecta. There is an asymmetry of the deformities that makes diastrophic dysplasia less likely.

 

(SAE08PA.86) What is the mechanism of action of bisphosphonates? 

 

  1. Directly stimulating osteoblastic new bone formation

  2. Increasing gut absorption of calcium

  3. Increasing efficiency of 1,25 dihydroxylation of vitamin D in the kidney

  4. Inhibiting bone resorption by osteoclasts

  5. Decreasing release of parathyroid hormone PREFERRED RESPONSE 4

Bisphosphonates are stable analogues of pyrophosphate that have a strong affinity for bone hydroxyapatite; these agents inhibit bone resorption by reducing the recruitment and activity of osteoclasts and increasing apoptosis. Bone formed while patients are receiving bisphosphonate treatment is histologically normal. Bisphosphonates have been shown to be effective in decreasing pathologic fractures, bone pain, and the need for radiation therapy in patients with multiple myeloma and metastatic carcinoma to bone. The most effective method of administration is via monthly intravenous infusion. Osteonecrosis of the mandible is sometimes a complication of this treatment.

 

(SAE11OS.88) Which of the following factors is least likely to have an impact on fracture healing? 

 

  1. Smoking

  2. Obesity

  3. Vitamin D deficiency

  4. Use of bisphosphonates for osteoporosis treatment

  5. Use of nonsteroidal anti-inflammatory drugs (NSAIDs) PREFERRED RESPONSE 2

Although the effect of obesity on complication rates has been studied and it may increase wound complications, it has not been shown to increase nonunion rates. The negative impact of smoking on bone healing has been shown in animal and human clinical studies. NSAIDs interfere with the inflammatory phase of bone healing and bisphosphonates interfere with osteoclast function, negatively impacting the remodeling phase. Vitamin D deficiency has been identified in up to 70% of nonunion patients.

 

(SAE08UE.111) Which of the following antibiotics is contraindicated in children? 

 

  1. Penicillin

  2. Cephalexin

  3. Tetracycline

  4. Erythromycin

  5. Ampicillin

 

PREFERRED RESPONSE 3

 

The tetracycline family of medications can stain teeth and bone in skeletally immature patients and as a result should be avoided in those patients. The remaining antibiotics have no known specific contraindication to use in children

 

(SAE10BS.14) A hip compression screw is placed in a test jig and a bending load is applied to the tip of the screw. After the load is released, the screw returns completely to its original shape. What is this type of deformation called? 

 

  1. Plastic

  2. Elastic

  3. Fatigue

  4. Creep

  5. Torsion

 

PREFERRED RESPONSE 2

 

When an implant is loaded below the yield point, by definition, it undergoes elastic deformation, meaning that all of the deformation recovers when the load is removed. If it is loaded above the yield point, then plastic or permanent deformation occurs. Fatigue is the gradual accumulation and progression of cracks in the material, which, after many cycles of loading, can lead to gross failure. Creep is the gradual accumulation of permanent (plastic) deformation over time, which may occur with polymeric materials but is not typical of metals or ceramics. Torsion refers to a torque being applied about the long axis of a bone or an implant. In general, the materials and dimensions of an implant are chosen to avoid plastic deformation or fatigue failure during typical clinical use.

(SAE10BS.12) A patient with MRSA osteomyelitis experiences skin redness and flushing with IV administration of vancomycin. The patient is changed to IV linezolid therapy. Following the first dose of linezolid, the patient reports headache, diaphoresis, and nausea. Physical examination reveals unresponsive pupillary reaction, tachycardia, hypertension, tremor, and clonus. In addition to discontinuing linezolid, what is the next most appropriate step in management? Review Topic

 

  1. IV administration of benzodiazepines for control of serotonin syndrome

  2. IV administration of prednisone for abatement of allergic reaction

  3. Laboratory evaluation for examination of acute drug-induced thrombocytopenia

  4. Head CT for evaluation of possible acute cerebral hemorrhage

  5. Spiral chest CT for evaluation of possible pulmonary embolus PREFERRED RESPONSE 1

The patient is displaying signs and symptoms of serotonin syndrome which is an acute adverse drug reaction caused by excess of serotonin activity. This syndrome produces an array of autonomic and somatic effects including abdominal pain, headache, diarrhea, and clonus. Linezolid is a reversible nonselective monoamine oxidase inhibitor (MAOI) and can induce serotonin syndrome in combination with other medications, but most commonly with serotonin selective reuptake inhibitors (SSRIs). Many medications may produce this interaction including antidepressants, pain medications, and even herbal medicines. Patients should be monitored when this antibiotic is first administered. Benzodiazepines are given to abate the symptoms of serontonin syndrome, and in severe cases patients are given serotonin antagonists.

 

(SAE08OS.167) An orthopaedic surgeon frequently uses hip and knee prostheses from a specific manufacturer. The surgeon becomes acquainted with the manufacturer's representative who provides the support for these prostheses in the hospital. They develop a personal relationship outside of work through a common interest in sailing. Together they become interested in buying a sailboat. The manufacturer's representative suggests a partnership in a boat costing $200,000. The manufacturer's representative would purchase a 90% interest and the surgeon a 10% interest in the boat. There would be no restrictions on use of the boat by the surgeon. What should the orthopaedic surgeon do? 

 

  1. Accept the offer because it would allow the physician to have a boat and enjoy a hobby of sailing with a minimal financial outlay.

  2. Accept the offer because it would have nothing to do with the orthopaedic practice and because the physician feels strongly that it would have no influence on practice decisions.

  3. Reject the offer because it represents an unacceptable conflict of interest.

  4. Accept the offer but disclose the relationship to patients when using the representative's products.

  5. Accept the offer but only with an equal interest arrangement in the purchase and use of the boat.

 

PREFERRED RESPONSE 3

 

Rejecting this proposal is the only appropriate course of action. Accepting it would, in essence, be receiving a huge gift from industry in the form of a sailboat. Physicians frequently assert that they are not influenced by gifts and relationships with industry representatives, but evidence is to the contrary. Such an arrangement constitutes a tremendous incentive to use the manufacturer's products. The fact that the boat partnership seems completely outside of the orthopaedic business relationship does not excuse it. Conflicts of interest should always be resolved and in the best interest of patient care, and in this case the best course clearly is to avoid the conflict of interest totally. An equal interest in the boat does not eliminate the conflict of interest.

 

(OBQ13.147) It is recommended that invasive dental work be completed prior to the initiation of which of the following medications? 

 

  1. Glucosamine

  2. Cholecalciferol

  3. Levothyroxine

  4. Teriparatide

  5. Bisphosphonates

 

PREFERRED RESPONSE 5

 

Bisphosphonate therapy combined with invasive dental work increases the risk for development osteonecrosis of the jaw.

 

Bisphosphonates are a class of drugs that prevent bone mass loss by inhibiting osteoclast resorption. They are used in the treatment of vertebral compression fractures, non-vertebral fragility fractures, osteogenesis imperfecta, multiple myeloma, and avascular necrosis. Because bisphosphonates have been associated with osteonecrosis of the jaw, it is suggested that all invasive dental work be completed prior to initiation of treatment.

Pazianas et al. (2011) review the safety profile of bisphosphonates. Specifically, they cite gastrointestinal discomfort, atypical femur fractures, osteonecrosis of the jaw, ocular inflammation, and musculoskeletal pain as common side effects. They state there is limited evidence surrounding bisphosphonate's association with esophageal cancer and atrial fibrillation.

 

Pazianas et al. (2007) reviewed 11 publications that reported 26 cases of osteonecrosis of the jaw following initiation of bisphosphonate treatment. Age >60 years, female sex, and previous invasive dental treatment were the most common characteristics of those who developed ONJ.

 

Illustration A shows the various bisphosphonates and their mechanisms of action. Illustration B shows an example of osteonecrosis of the jaw, a side effect that has been linked to bisphonphonate treatment.

 

Incorrect Answers:

Answer 1: Glucosamine is a dietary supplement used in the management of osteoarthritis. It is not contraindicated prior to dental work. Answer 2: Cholecalciferol is Vitamin D3. Toxicity and side effects are rare with Vitamin D therapy. Answer 3: Levothyroxine is used in the treatment of hypothyroidism. It is not contraindicated prior to dental work. Answer 4: Teriparatide (Forteo) has been used in the treatment of osteonecrosis of the jaw caused by bisphosphonates.

 

(SAE13BS.24) Bacterial resistance to tetracycline is confirmed by ribosome protection, tetracycline modification, and 

 

  1. altered RNA polymerase.

  2. altered membrane binding protein.

  3. increased drug efflux.

  4. DNA gyrase mutation. PREFERRED RESPONSE 3

Mutations of bacterial DNA gyrase can decrease the effectiveness of quinolones. Altered membrane-binding protein is observed with resistance to ?-lactam antibiotics. Tetracyclines are antibiotics that inhibit bacterial growth by stopping protein synthesis. Three specific mechanisms of tetracycline resistance have been identified:

increased tetracycline efflux, ribosome protection, and tetracycline modification. Alteration of RNA polymerase is found in resistance to rifampin.

 

(SAE10BS.5) You are preparing cylindrical specimens of bone from the cortex of the femoral shaft of a human cadaver. These will be used in tensile tests in a materials testing machine to determine the mechanical properties of the cortical bone. Why is it important to consider the orientation of the specimens relative to the femoral shaft? 

 

  1. The hydroxyapatite structure varies by orientation.

  2. It is easier in this way to produce long, slender specimens suitable for tensile testing.

  3. The mechanical properties of bone vary with the orientation of the specimen within bone.

  4. It is easier to cut bone in certain orientations than others.

  5. The water content in bone is different in different orientations. PREFERRED RESPONSE 3

Because of the orientation of collagen fibers, cortical bone is anisotropic, meaning its mechanical properties are different in different directions. In general, specimens loaded in the direction of the long axis of the femur will have a higher stiffness and tensile strength than oblique or transverse specimens.

 

(SAE10BS.29) In X-linked hypophosphatemic rickets, the gene defect may be found in which of the following? 

 

  1. GNAS1

  2. CLCN7

  3. TGF-BR2

  4. PHEX

  5. COLIA1

 

PREFERRED RESPONSE 4

Mutations of PHEX are responsible for X-linked hypophosphatemic rickets. Mutations of CLCN7 and TC1RG1 cause osteopetrosis. COLIA1 or COLIA2 mutations cause osteogenesis imperfecta (OI) which has literally hundreds of genotypes causing a more limited number of phenotypes as described by Silence. GNAS1 is the gene mutation of fibrous dysplasia that results in an activating mutation of the GSalpha1 protein. TGF-BR2 or FBN1 cause Marfan syndrome.

 

(SAE10BS.75) A surgeon desires to use an implant that will be able to absorb as much energy as possible before it fails. The implant should be made of a material that has which of the following properties? 

 

  1. A high modulus of elasticity

  2. A low modulus of elasticity

  3. High intrinsic toughness

  4. A high yield stress

  5. Brittleness

 

PREFERRED RESPONSE 3

 

High intrinsic toughness is defined as the amount of energy per volume a material can absorb without breaking. A high modulus of elasticity and a low modulus of elasticity are incorrect because the modulus of elasticity refers to the stiffness of the material. A brittle material absorbs little energy before breaking. High yield stress is incorrect because it is only one of several factors that lead to a high toughness. While the yield stress is the point at which plastic deformation begins, if coupled with a brittle material, the energy absorbed would be low relative to a ductile material.

 

(SAE10BS.55) Two major pharmacologic classes of bisphosphonates exist: nitrogen-containing and non-nitrogen-containing compounds. The nitrogen-containing compounds work by which of the following actions? 

 

  1. COX inhibition

  2. ATPase formation

  3. GTPase inhibition

  4. DNA gyrase inhibition

  5. RNA polymerase inhibition

 

PREFERRED RESPONSE 3

 

Bisphosphonates represent the most clinically important class of antiresorptive agents available to treat diseases characterized by osteoclast-mediated bone resorption. Two classes of bisphosphonates exist: nitrogen-containing and non-nitrogen-containing compounds. The non-nitrogen-containing bisphosphonates work by metabolizing into cytotoxic ATP analogs. The nitrogen-containing bisphosphonates work via the mevalonate pathway by inhibiting GTPase formation, leading to loss of GTP prenylation and eventual induction of osteoclast apoptosis.

 

(OBQ15.199) In the treatment of rheumatoid arthritis, which medication is an antagonist of tumor necrosis factor-alpha? 

 

  1. Rituximab

  2. Etanercept

  3. Abatacept

  4. Methotrexate

  5. Leflunomide

 

PREFERRED RESPONSE 2

 

Etanercept is a biochemically designed tumor necrosis factor receptor immunoglobulin G fusion protein, which binds to TNF-alpha and is thus a TNF-alpha antagonist.

 

TNF-alpha is considered to be one of the major cytokines involved in rheumatoid arthritis pathology. As a result, many biologic agents used to treat rheumatoid arthritis (RA) are manufactured to block TNF-alpha or its receptors. This has been shown to reduce inflammation and stop disease progression. In the USA, Etanercept is approved to treat rheumatoid arthritis, juvenile rheumatoid arthritis and psoriatic arthritis, plaque psoriasis and ankylosing spondylitis. The route of administration is subcutaneous.

 

Bongartz et al. used a randomized control trial to asses the risk of infection and malignancy rates in RA treated with TNF-alpha antagonist. Overall, patients with RA appear to have an approximately 2-fold increased risk of serious infection compared to the general population and non-RA controls, irrespective of TNF-alpha antagonist use. The pooled odds ratio for malignancy was 3.3 (95% confidence interval [CI], 1.2-

9.1) and for serious infection was 2.0 (95% CI, 1.3-3.1) with use of TNF-alpha antagonist.

 

Howe et al. review the medical management of patients with RA who underwent orthopaedic procedures. They state that while there is conflicting information regarding TNF-alpha antagonists, they recommend holding them prior to major orthopaedic interventions.

 

Incorrect Answers:

Answer 1: Rituximab is a monoclonal antibody to CD20 antigen (inhibits B cells). It is often used with good clinical outcomes as monotherapy in patients who are intolerant of methotrexate or have contraindications to methotrexate or other DMARDs.

Answer 3: Abatacept is a selective costimulation modulator that binds to CD80 and CD86 (inhibits T cells). It is often prescribed for treatment of moderate to severe rheumatoid arthritis, or after failure of a disease-modifying anti-rheumatic agent (DMARD), like methotrexate but it can be used as first-line therapy. Answer 4: Methotrexate is a folic acid analogue. It binds dihydrofolate reductase and prevents synthesis of tetrahydrofolate. It is usually a first line treatment for moderate to severe rheumatoid arthritis. Answer 5: Leflunomide is an inhibitor of pyrimidine synthesis. It is approved to treat adult moderate to severe rheumatoid arthritis, usually as a monotherapy or failure of other DMARDs.

 

(SAE08OS.163) The area moment of inertia of a structure such as a bone is influenced most by the 

 

  1. density of the bone.

  2. bone's elastic modulus.

  3. cross-sectional volume of the bone.

  4. distance of the bone from its center of mass.

  5. cross-sectional shape of the bone. PREFERRED RESPONSE 4

The area moment of inertia is a function of the distance squared of the mass distribution from its center of mass. Density of a material and its elastic modulus do not affect the area moment of inertia. The cross-sectional volume and shape of the bone has some effect, but two objects with the same cross-sectional volume or shape can have very different area moments of inertia (ie, a cylinder and a tube).

 

(SAE09FA.78) A 42-year-old woman who observes traditional Muslim practices is seen in your office accompanied by her physician husband to discuss possible elective bunion correction. In considering the treatment of this patient, what is one of the most important considerations? 

 

  1. The role her husband will play in the decision to proceed with surgery

  2. Her role as primary caregiver in the household

  3. Dietary concerns during her hospitalization

  4. Daily cleansing rituals that may affect wound care

  5. The importance of maintaining modesty precautions during examination, surgery, and postoperative appointments

 

PREFERRED RESPONSE 5

 

In considering faith-based issues regarding treatment of this patient, the presence of her husband for the office visit would imply an agreement with her decision to have surgery. It also may facilitate her examination. Her role as caregiver, dietary concerns, and cleansing rituals are less important considerations with an outpatient-based procedure. Privacy concerns remain paramount to Muslim women, which include limited exposure during examination, during surgery, and in subsequent follow-up visits.

 

(SAE10BS.93) Other factors permitting, it is preferable to place a bone plate on the tension side of a fractured long bone because this location 

 

  1. interferes less with the blood supply.

  2. protects the fracture from shear loading.

  3. minimizes the compressive stress on the fracture.

  4. minimizes torsional loading on the bone.

  5. minimizes the bending stresses in the plate. PREFERRED RESPONSE 5

If a plate spans the fracture on the tension side of the bone, then the fracture will tend to close when the bone is loaded, such that much of the axial and bending loads are

transmitted through the bone. If the plate is placed on the compression side, the fracture will tend to open during loading, and the plate will be required to carry nearly all of the axial and bending loads, increasing the potential for fracturing the plate, or pulling out the screws.

 

(SAE11UE.107) A 66-year-old woman with known poorly controlled rheumatoid arthritis reports that for the past 4 weeks she has been unable to extend the metacarpophalangeal (MCP) joints of her right hand index, middle, ring and little fingers. She cannot hyperextend the thumb interphalangeal joint. Active wrist extension is possible, but shows radial deviation. Examination reveals mild synovitis at the wrist and MCP joints of the affected hand. There is no ulnar deviation at the MCP joints with normal alignment. When the MCP joints are passively extended, the patient is unable to maintain them in this position. There is no piano key sign at the distal ulna. Passive wrist motion shows a normal tenodesis effect. Which of the following would most likely confirm your diagnosis? Review Topic

 

  1. Radiographs of the hand

  2. Radiographs of the cervical spine

  3. Electrodiagnostic studies of the affected upper extremity

  4. Surgical exploration of the extensor tendon ruptures

  5. MRI of the elbow

 

PREFERRED RESPONSE 3

 

There are many causes of inability to extend the MCP joints in a patient with rheumatoid arthritis. The most common cause is rupture of the extensor tendons. An intact tenodesis test suggests that the extensor tendons are intact, thus surgical exploration is not indicated and would not confirm the diagnosis. The patient has normal alignment of the fingers without ulnar deviation, suggesting that there are no MCP dislocations to account for the inability to extend the MCP joints; therefore, radiographs would not confirm the diagnosis. The most likely cause of inability to extend the fingers in this patient is posterior interosseous nerve (PIN) palsy. Electrodiagnostic studies would confirm the presence of PIN palsy. An MRI of the elbow may show synovitis at the radiocapitellar joint, which can cause the PIN palsy. This finding however, is nonspecific and many patients without PIN palsy would also demonstrate synovitis at the radiocapitellar joint. Therefore, although an MRI would be helpful in localizing a potential cause of PIN compression, it would not in itself confirm the diagnosis.

 

(OBQ15.214) A researcher is working on Medication A, a drug FDA-approved for the treatment of osteoporosis in men and women. It is an anti-resorptive agent that inhibits the formation, function and survival of osteoclasts. It does not bind to calcium hydroxyapatite. At 1-year after the initial dose, tissue levels are non-detectable. It can be used in the presence of cancer metastases to bone. What is Medication A? Review Topic

 

  1. Denosumab

  2. Alendronate

  3. Abaloparatide

  4. Teriparatide

  5. Strontium ranelate

 

PREFERRED RESPONSE 1

 

Denosumab is FDA-approved for the treatment of osteoporosis in men and women. It inhibits the formation, function and survival of osteoclasts (OC). It does not bind to calcium hydroxyapatite. At 1-year after the initial dose, tissue levels are non-detectable.

 

Denosumab is a human monoclonal antibody against RANKL. By binding RANKL, it prevents interaction of RANKL with RANK (on OC and osteoclast precursors, OCP), and inhibits OC-mediated bone resorption, and the formation, function and survival of OC. In contrast, bisphosphonates bind to calcium hydroxyapatite in bone, and decrease resorption by decreasing function and survival (but not formation) of OC.

 

Vaananen et al. reviewed the cell biology of OC. During bone resorption, 3 membrane domains appear: ruffled border, sealing zone and functional secretory domain. The resorption cycle starts with migration, bone attachment, polarization (formation of membrane domains), dissolution of hydroxyapatite, degradation of organic matrix, removal of degradation products from resorption lacuna, and apoptosis of the OC or return to the non-resorbing stage.

 

Boyce et al. reviewed the regulation of osteoclasts and their functions. OCPs are held in bone marrow by chemokines e.g. stroma-derived factor-1 (SDF1) and attracted to blood by sphingosine-1 phosphate (S1P) (increased in synovial fluid of patients with RA). All aspects of osteoclast formation and functions are regulated by M-CSF and RANKL. More recent studies indicate that osteoclasts and their precursors regulate immune responses and osteoblast formation and functions by means of direct cell-cell contact through ligands and receptors, such as ephrins and Ephs, and semaphorins and plexins, and through expression of clastokines.

 

Warriner and Saag reviewed the diagnosis and treatment of osteoporosis. They defined osteoporosis as T-score of = -2.5 or a history of fragility fracture. Incident hip and vertebral fractures increase future risk of these fractures (hazard ratio 7.3 and 3.5, respectively).

Cummings et al. compared subcutaneous denosumab (60mg every 6mths) vs placebo in prevention of fractures in 7868 osteoporotic (T-score -2.5 to -4.0) postmenopausal women. They found that denosumab reduced risk of vertebral fracture by 68% (risk ratio, 0.32), hip fracture by 40% (hazard ratio 0.6), nonvertebral fracture by 20% (hazard ratio 0.8). There was no increased risk of cancer, infection, delayed fracture healing, cardiovascular disease, osteonecrosis of the jaw or adverse reactions. They concluded that it was useful for reduction of fractures in osteoporotic women.

 

The video shows the action of denosumab (prolia). Illustration A shows the different osteoclast zones.

 

Incorrect Answers:

Answers 2: Alendronate (and other bisphosphnates) inhibit resorption of bone, decrease function and survival of osteoclasts. Because of binding to calcium hydroxyapatite, they are detectable years after dosing. They reduce function and survival of OC, but do not affect the formation of osteoclasts. Answer 3: Abaloparatide is a PTH analog that has completed phase III trials for osteoporosis. As of mid-2016, it is not yet approved for treatment of osteoporosis. Answer 4: Teriparatide (recombinant PTH 1-34) is the only anabolic (not antiresorptive) agent approved for osteoporosis treatment. It is administered by daily subcutaneous injection. Osteosarcoma, cancer metastases to bone and Paget's disease are contraindications.

Answer 5: Strontium ranelate (marketed as Protelos or Protos) both increases deposition of new bone by osteoblasts and reduces the resorption of bone by osteoclasts ("dual action bone agent", DABA). It is not FDA approved for use in the United States. Increased risk of myocardial infarction has been detected.

 

(SAE08AN.36) A 48-year-old woman reports bilateral thigh pain that is limiting her function as a librarian. A radiograph and a bone scan are shown in Figures 23a and 23b. What is the most likely diagnosis? 

 

 

  1. Ankylosing spondylitis

  2. Arthrokatadysis

  3. Osteomalacia

  4. Rheumatoid arthritis

  5. Developmental dysplasia

 

PREFERRED RESPONSE 4

 

The radiograph reveals bilateral severe acetabular protrusio. The bone scan and history confirm involvement of multiple joints, including the knees and the hindfoot. Although the first four choices can all cause the acetabular protrusio, the associated multiple joint involvement suggests the diagnosis of rheumatoid arthritis. Arthrokatadysis, or primary protrusio acetabuli, is often associated with osteomalacia but not other joint disease. Developmental dysplasia is a common cause of bilateral hip pathology but does not have acetabular protrusio.

 

(SAE08OS.94) When planning a research study, the power of the study may be increased by 

  1. performing a retrospective study.

  2. performing a prospective study.

  3. using a case control design.

  4. decreasing the sample size.

  5. increasing the sample size.

 

PREFERRED RESPONSE 5

 

The power of a study refers to the researchers' ability to detect a true association when one exists. Power is defined as 1-beta, with beta being the probability of concluding an association does not exist when one actually does (type II error). Increasing the sample size will increase the power of a study. A power analysis can be performed for both retrospective and prospective studies and is independent of the sample population used.

 

(SAE13BS.36) A tendon repair is thought to be weakest during which phase of tendon healing? 

 

  1. Inflammatory

  2. Proliferation

  3. Maturation

  4. Remodeling

 

PREFERRED RESPONSE 1

 

Healing after a tendon repair or rupture has the following stages: inflammatory, cellular proliferation, and remodeling. During the inflammatory phase, neutrophils and macrophages migrate into the injury site and release chemotactic factors that recruit fibroblasts. A tendon is thought to be weakest 5 to 21 days after repair, which coincides with the inflammatory phase. During the proliferative phase, inflammatory cells secrete cytokines and growth factors (platelet-derived growth factor, insulin-like growth factor, bone morphogenetic protein (BMP)-12 and BMP 13, and transforming growth factor-beta) that promote differentiation of fibroblasts. Fibrosis and decreased cellularity are the hallmarks of the remodeling stage.

(OBQ14.151) A resident arrives to the operating room late for a LEFT carpal tunnel release procedure. The patient is prepped and draped under general anaesthesia, and the attending surgeon and assistant are about to make an incision to the RIGHT carpal tunnel. The resident mentions to the attending surgeon that surgery was booked for the opposite limb, but he ignores the residents confers by saying the surgical mark is under the drape. What would be the most appropriate course of action for the resident at this time? Review Topic

 

  1. Ask the nurse to call the medical director

  2. Ask the surgeon to revisit the surgical safety checklist

  3. Assume the booking form was incorrect and proceed with the operation

  4. Leave the operating room to ask a family member

  5. Read the last clinical note for clarification

 

PREFERRED RESPONSE 2

 

The most appropriate course of action for the resident at this time would be to perform a surgical timeout prior to the operation.

 

Wrong-site surgery is completely preventable by having the surgeon, in consultation with the patient when possible, place his or her initials on the operative site using a permanent marking pen and then operating through or adjacent to his or her initials. The intended site should be marked such that the mark will be visible after the patient has been prepped and draped. A "time-out" procedure should be done before the initiating of any surgical procedure to confirm the type of procedure, site, and side with all operating room personnel including residents.

 

Haynes et al. reviewed the effect of surgical safety checklists before surgery on the morbidity and mortality in a global population. They found the rate of death was 1.5% before the checklist was introduced and declined to 0.8% afterward (P=0.003). Inpatient complications occurred in 11.0% of patients at baseline and in 7.0% after introduction of the checklist (P<0.001).

 

Incorrect Answers:

Answers 1,3,4,5: The WHO safety checklist requires all personnel to be involved. Details of the checklist should include, correctly confirming the patients identity, surgical site, equipment, anticipated surgical events, anesthesia check list, and nursing checklist.

(SAE08OS.187) The risk for postoperative wound infection after lumbar spine surgery is reduced most by the use of 

 

  1. iodine-impregnated preparation solution.

  2. chlorhexidine-impregnated preparation solution.

  3. greater than 10 minutes of surgical scrub.

  4. intravenous antibiotic prophylaxis prior to skin incision.

  5. laminar-flow air circulation.

 

PREFERRED RESPONSE 4

 

By far, the most impact on lowering the incidence of perioperative site infections after lumbar surgery is achieved by the use of intravenous perioperative antibiotics that are infused prior to skin incision. Ideally, these are administered within 1 hour of incision time. The use of an iodine versus a chlorhexidine solution has not been demonstrated to influence infection after spinal surgery. While adequate hand washing is an important practice, scrubbing for more than 10 minutes is not critical. Laminar-flow air circulation has been demonstrated to be helpful in total joint arthroplasty; however, this has not been demonstrated in spinal surgery. Furthermore, it is not likely that its use would influence the rate of infection as much as perioperative antibiotic prophylaxis.

 

(SAE07HK.4) Which of the following treatment regimens for thromboembolic prophylaxis meets the American College of Chest Physicians Guidelines for 10-day treatment after total hip arthroplasty and total knee arthroplasty? 

 

  1. Low-molecular-weight heparin

  2. Adjusted dose unfractionated heparin

  3. Aspirin

  4. Warfarin, INR 1.5 to 2.0

  5. Elastic compressive stockings

 

PREFERRED RESPONSE 1

 

Only three thromboembolic treatment protocols have reached Grade 1A status for the American College of Chest Physicians Guidelines for thromboembolic prophylaxis after total hip arthroplasty and total knee arthroplasty. Grade 1A evidence shows a clear benefit/risk improvement with supportive data from randomized clinical trials, which are strongly applicable in most clinical circumstances. Warfarin is recommended but at an INR level of 2 to 3. Low-molecular-weight heparin and

fondaparinox are also acceptable treatment options. Aspirin, adjusted dose unfractionated heparin, and elastic compressive stockings are not recommended as stand-alone options.

 

(SAE10BS.92) A patient with a history of chronic low back pain for several years reports decreased pain visual analog scores with the home use of a transcutaneous electrical neuromuscular stimulation (TENS) unit. This pain relief is most likely due to which of the following? 

 

  1. Improved skeletal muscle strength and secondary spinal support

  2. Neuromodulation via presynaptic inhibition in the dorsal horn of the spinal cord

  3. Distraction sensory input

  4. Enhancement of muscle metabolic activity with improved lactic acid excretion

  5. Placebo effect

 

PREFERRED RESPONSE 2

 

TENS units deliver superficial electrical stimulation. This electrical stimulation induces analgesia via inhibitory effects at the spinal cord level. The stimulation of small myelinated afferent fibers produces a presynaptic inhibition of the nociceptive transmission via unmyelinated C fibers, thus decreasing the transmission of pain stimuli. Additional benefit may come from the endogenous release of endorphins in the stimulated tissues.

 

(SAE08PA.22) A 17-year-old girl who initially presented as a child with multiple skeletal lesions, café-au-lait spots, and precocious puberty now has bone pain. A recent bone scan reveals multiple areas of increased scintigraphic uptake, including bilateral proximal femurs. A radiograph is shown in Figure 19. Besides activity modification, what is the next best line of treatment for decreasing her pain? Review Topic

 

 

  1. Bisphosphonates

  2. Calcitonin

     

  3. Parathyroid hormone

     

  4. Vitamin D and calcium

     

  5. Methotrexate

 

PREFERRED RESPONSE 1

 

McCune-Albright syndrome is the combination of polyostotic fibrous dysplasia, café-au-lait lesions, and endocrine dysfunction. The most common endocrine presentation is precocious development of secondary sexual characteristics. Compared with bone lesions in patients without polyostotic disease, the skeletal lesions in patients with the syndrome tend to be larger, more persistent, and associated with more complications. Bisphosphonate therapy has been shown in several studies to decrease the pain associated with the skeletal lesions of fibrous dysplasia.

 

(SAE13BS.78) What molecules have been shown to promote fibrosis during muscle injury? 

 

  1. Insulin growth factor 1 (IGF-1)

     

  2. Basic fibroblast growth factor (bFGF)

     

  3. Transforming growth factor beta 1 (TGF-ß1)

  4. Bone morphogenetic protein

 

PREFERRED RESPONSE 3

 

A muscle's response to injury can be divided into 4 phases: necrosis, inflammation, repair, and fibrosis. Necrosis involves the degeneration of the muscle fibrils and death. The inflammatory cells then phagocytose the debris and secrete cytokines that promote vascularity. Muscle regeneration does not occur until phagocytic cells remove the debris. Consequently, anti-inflammatory drugs may have negative effects on muscle healing by inhibiting macrophage-induced phagocytosis. Muscle fibrosis occurs at the same time as muscle regeneration and has been shown to involve TGF-ß1. IGF-1 and bFGF are important trophic factors in muscle regeneration. Bone morphogenetic protein has several functions including bone and cartilage regeneration.

 

(OBQ15.227) The difference between vitamin D-dependent rickets type I (VDDR I) and vitamin D-dependent rickets type II (VDDR II) is 

 

  1. VDDR I is caused by an inactivating mutation of the receptor for 1,25 (OH)2 vitamin D3. VDDR II is a deficiency of an enzyme predominantly found in the kidney.

     

  2. VDDR I is caused by an activating mutation of the receptor for 1,25 (OH)2 vitamin D3. VDDR II is a deficiency of an enzyme predominantly found in the kidney.

  3. VDDR I is a deficiency of an enzyme predominantly found in the kidney. VDDR II is caused by an inactivating mutation of the receptor for 1,25 (OH)2 vitamin D3.

     

  4. VDDR I is a deficiency of an enzyme predominantly found in the kidney. VDDR II is caused by an activating mutation of the receptor for 1,25 (OH)2 vitamin D3.

     

  5. VDDR I is a deficiency of an enzyme predominantly found in the liver. VDDR II is caused by an inactivating mutation of the receptor for 1,25 (OH)2 vitamin D3.

 

PREFERRED RESPONSE 3

 

VDDR I is a deficiency of an enzyme predominantly found in the kidney. VDDR II is caused by an inactivating mutation of the receptor for 1,25 (OH)2 vitamin D3.

 

VDDR I is a deficiency of 1a-hydroxylase [converts 25(OH)D to 1a,25(OH)2D3].

Lab tests show hypocalcemia, secondary hyperparathyroidism, elevated alkaline phosphatase (ALP) and low or undetectable calcitriol in the presence of adequate 25(OH)D levels. VDDR II or hereditary vitamin D resistant rickets (HVDRR) (autosomal recessive) is an inactivating mutation in the vitamin D receptor (VDR). Lab tests show low serum calcium and phosphate, elevated ALP and secondary hyperparathyroidism. Serum 25(OH)D values are normal and the 1,25(OH)2D levels are elevated (key difference from VDDR I).

 

Malloy et al. reviewed genetic disorders in vitamin D action. They state that VDDR I is an inborn error of vitamin D metabolism coded by the gene CYP27B1. Children with VDDR I present with joint pain/deformity, hypotonia, muscle weakness, growth failure, and hypocalcemic seizures or fractures in early infancy. Treatment is with calcitriol or 1a-hydroxyvitamin D (NOT cholecalciferol). Children with VDDR II present with bone pain, muscle weakness, hypotonia, hypocalcemic convulsions, growth retardation, severe dental caries or teeth hypoplasia. Affected children are resistant to therapy and supra-physiologic doses of all forms of vitamin D.

Illustration A shows the differences between VDDR I and VDDR II. Incorrect Answers

Answers 1, 2, 4, 5: VDDR I is a deficiency of 1a-hydroxylase (predominantly found

in the kidney). The liver enzyme vitamin D 25-hydroxylase (found in hepatocytes) is not responsible for VDDR. VDDR II is caused by an inactivating mutation (rather than an activating mutation).

 

 

(SAE13HK.9) A 68-year-old woman had advanced right knee arthritis and total knee replacement was planned. She learned she had primary biliary cirrhosis at age 41 and now has advancing liver failure. Preoperative coagulation tests show a baseline International Normalized Ratio (INR) of 1.36. Appropriate methods to prevent thromboembolic disease as recommended by the 2011 AAOS Clinical Practice Guideline, Preventing Venous Thromboembolic Disease in Patients Undergoing Elective Hip and Knee Arthroplasty, include 

 

  1. use of mechanical prophylaxis (eg, pneumatic calf compressors) while in the hospital.

     

  2. oral warfarin with a goal INR between 2.0 and 3.0.

     

  3. low-dose warfarin for 3 weeks postsurgically beginning 48 hours after surgery.

     

  4. no prophylaxis because this patient already is partially anticoagulated secondary to her liver disease.

PREFERRED RESPONSE 1

 

The 2011 AAOS Clinical Practice Guideline, Preventing Venous Thromboembolic Disease in Patients Undergoing Elective Hip and Knee Arthroplasty, recommends the use of mechanical prophylaxis for patients at increased risk for bleeding (including those with liver disease or hemophilia). This recommendation is the consensus of the workgroup that established these guidelines because there was insufficient evidence to justify a stronger recommendation in this clinical scenario. The other responses use no prophylaxis or pharmacological prophylaxis. Pharmacological prophylaxis is not recommended in patients who are at increased risk for bleeding.

 

(OBQ14.207) Type I collagen fibers in peripheral nerves are primarily responsible for which of the following? 

 

  1. Conduction strength

     

  2. Tensile strength

     

  3. Conduction velocity

     

  4. Compressive strength

     

  5. Cross-linkage

 

PREFERRED RESPONSE 2

 

Type I collagen fibers are most responsible for the tensile strength of a peripheral nerve. Type I collagen is the most abundant collagen of the human body which forms large, eosinophilic fibers known as collagen fibers. It is present in scar tissue, the end product when tissue heals by repair, as well as tendons, ligaments, the endomysium of myofibrils, the organic part of bone, the dermis, the dentin and organ capsules.

 

The COL1A1 gene produces a component of type I collagen, called the pro-alpha1(I) chain. This chain combines with another pro-alpha1(I) chain and also with a pro-alpha2(I) chain (produced by the COL1A2 gene) to make a molecule of type I procollagen. These triple-stranded, rope-like procollagen molecules must be processed by enzymes outside the cell. Once these molecules are processed, they arrange themselves into long, thin fibrils that cross-link to one another in the spaces around cells. The cross-links result in the formation of very strong mature type I collagen fibers.

Wong et al. provide a review of the basic science behind nerve healing and the recovery after nerve repair. They note the importance of minimizing additional surgical insult and careful handling of nerve tissue during repair to optimize outcomes.

 

Pertici et al. noted that autologous nerve implantation to bridge a long nerve gap presents the greatest regenerative performance in spite of substantial drawbacks. They were able to show improved nerve guided regrowth with a type I collagen matrix conduit as compared to a conduit made of a mix of type I and type III collagen.

 

Illustration A shows a diagram of type I collagen, showing the rope-like characteristics behind the tensile strength.

 

Incorrect Answers:

Answer 1: Strength of nerve conduction is not dependant on collagen, but is polyfactorial, with contributions from patient age, sex, nerve temperature, and myelin sheath condition.

Answer 3: Type I collagen does not significantly affect the velocity of nerve conduction, as this is also polyfactorial, as explained in Answer I immediately above. Answer 4: Type I collagen can be compressed, but is more responsible for the tensile strength through the cross-linkages. Answer 5: Peripheral nerves do not have cross-linkages.

 

(SAE10BS.66) In normal adult articular cartilage, what percentage of the total volume is occupied by chondrocytes? 

 

1 2%

 

2 15%

 

3 30%

 

4 75%

 

5 90%

 

PREFERRED RESPONSE 1

 

Chondrocytes occupy approximately 2% of the total volume of normal adult articular cartilage. Sixty-five percent to 80% of the weight of articular cartilage is composed of water, depending on the load status at the surface. Chondrocytes are the only cells

present within the cartilage matrix, which is composed largely of collagen, noncollagenous proteins, and proteoglycans. Chondrocytes are responsible for the synthesis and degradation of the cartilage matrix during embryonic development and maintenance of the adult cartilage.

 

(SAE10BS.27) Bone morphogenetic proteins transduce intracellular signal through what class of cell surface receptor? 

 

  1. Mitogen-activated protein kinase

     

  2. Tyrosine kinase

     

  3. Serine-threonine kinase

     

  4. Aurora kinase

     

  5. Glycogen synthase kinase 3

 

PREFERRED RESPONSE 3

 

Bone morphogenetic proteins (BMPs) are extracellular proteins belonging to the TGF-beta superfamily of molecules. Members of this family include BMPs, growth and differentiation factors (GDFs), anti-mnllerian hormone (AMH), activin, Nodal, and TGF-beta. These proteins exert their action by binding to cell surface receptors of the serine-threonine kinase class to activate intracellular signaling pathways. The other kinase participate in various cell signaling functions, but are not associated with BMP.

 

(SAE11OS.133) Currently, what is the most common clinical study type in the orthopaedic literature? 

 

  1. Level 1 (prospective, randomized trial)

     

  2. Level 2 (cohort trial)

  3. Level 3 (retrospective case control)

     

  4. Level 4 (retrospective case series)

     

  5. Level 5 (expert opinion)

 

PREFERRED RESPONSE 4

 

Although a recent push for prospective, randomized trials has been advocated by multiple orthopaedic journals, many studies published continue to be of Level 4 evidence (retrospective case series). Case series represented 64% of all studies reviewed by Freedman and associates in 2001 from the British and American volumes of Journal of Bone and Joint Surgery and from Clinical Orthopaedics and Related Research. Obremskey and associates published that 58.1% of all studies from nine orthopaedic journals were Level 4 evidence. Further investigation of more current trends is likely warranted with the current emphasis on publishing higher level-of-evidence studies in orthopaedic journals.

 

(SAE13HK.7) What antithrombotic agent is a selective factor I0a inhibitor? Review Topic

 

  1. Warfarin

     

  2. Low-molecular-weight heparin

     

  3. Rivaroxaban

     

  4. Aspirin

 

PREFERRED RESPONSE 3

 

Rivaroxaban is a selective factor I0a inhibitor. Aspirin is a cyclooxygenase inhibitor. Low-molecular-weight heparin is a nonspecific anticoagulant. Warfarin is a vitamin K antagonist and reduces production of clotting factors II, VII, IX, and X.

(SAE13BS.72) You have been waiting to enroll a patient into your recent institutional review board (IRB)-approved clinical research protocol. A patient is admitted who, upon initial screening, meets all inclusion criteria. However, he has late-stage dementia, and you are told by your research coordinator that this condition is an exclusionary criterion. This scenario demonstrates the need for which aspect of proper informed consent? 

 

  1. Provision of information

     

  2. Competency

     

  3. Understanding

     

  4. Jurisdiction

 

PREFERRED RESPONSE 2

 

Proper informed consent for research protocols includes the same elements as consent for invasive procedures. Jurisdiction is not one of the 5 elements, although it must be recognized that the process of obtaining proper informed consent for participation in a study protocol is mandated by the local IRB. The 5 elements of informed consent are:

1) The potential study subject must be able to act voluntarily, free of coercion and unfair persuasion. 2) There must be a provision of information through which the subject is informed of risks and anticipated benefits of the intervention. A discussion concerning available alternatives as well as the consequences of no treatment must also ensue. The explanation must be made in simple lay terms. 3) The participant must have the mental competency to comprehend the information. 4) There should be reasonable steps to ascertain that the subject understands all aspects of the discussion.

5) The subject actually makes a decision regarding participation.

 

(SAE11OS.47) Spindled cells that are surrounded in mature osteoid that connect to other similar cells via canaliculi are best described as which of the following? Review Topic

 

  1. Osteoblasts

     

  2. Osteoclasts

  3. Osteocytes

     

  4. Histiocytes

     

  5. Megakaryocytes

 

PREFERRED RESPONSE 3

 

Osteocyte cell processes travel through canaliculi to interconnect with other osteocytes and cells on the bone surfaces. Osteoblasts are cells that produce bone matrix and are seen rimming immature bone. Osteoclasts are large multinucleated cells that resorb bone and are found in Howship's lacunae. Megakaryocytes and histiocytes are found in marrow but not mature bone cortex.

 

(SAE08UE.88) In a patient with rheumatoid arthritis of the wrist, which of the following extensor tendons is most at risk of rupture? 

 

  1. Extensor digiti quinti

     

  2. Abductor pollicis longus

     

  3. Extensor pollicis longus

     

  4. Extensor carpi radialis brevis

     

  5. Extensor carpi ulnaris

 

PREFERRED RESPONSE 1

 

The tendon most prone to rupture in a patient with rheumatoid arthritis of the wrist is the extensor digiti quinti. It can be a silent injury since the extensor digitorum communis can provide extension to the fifth finger. The extensor digiti quinti is at high risk since it is overlying the ulnar head where it is prone to attritional rupture (Vaughan-Jackson syndrome).

(SAE11OS.90) An orthopaedic surgeon in his first year of practice is negotiating with a private for-profit hospital to be their employed trauma specialist. The state of employment is known to have a high rate of malpractice claims because of a favorable plaintiff legal environment. During the course of negotiations, malpractice insurance is being discussed. The surgeon should ask the hospital to provide which type of malpractice insurance policy? Review Topic

 

  1. Claims made with "nose" coverage

     

  2. Claims made without tail coverage

     

  3. No policy because of employed status and sovereign immunity

     

  4. Occurrence coverage

     

  5. Occurrence coverage with "nose" coverage

 

PREFERRED RESPONSE 4

 

An occurrence policy provides coverage for all claims made during employment irrespective of when it is filed (during or postemployment) and therefore is the best option. Claims made policy only covers suits for the time employed. A prepurchased "tail" is needed to provide coverage for cases that occurred during employment but filed postemployment. Nose coverage is applicable if the surgeon was previously employed and did not have tail coverage from previous employment, but this surgeon just emerged from training where it is not applicable. Claims made without tail coverage is unwise because the surgeon would be unprotected or have to purchase his own policy postemployment. Only in certain situations does sovereign immunity exist, and generally not in a for-profit system. Occurrence coverage with nose coverage is incorrect because it does not apply to this surgeon with no previous employment or claims policy lacking tail coverage.

 

(SAE10BS.62) The swelling pressure in cartilage is predominantly due to the association of exchangeable water with which of the following substances? Review Topic

 

  1. Biglycan

     

  2. Type II collagen

  3. Type IX collagen

     

  4. Aggrecan

     

  5. Versican

 

PREFERRED RESPONSE 4

 

Biglycan and versican are two of the small proteoglycans in cartilage. Biglycan plays a role in molecular association between cartilage and other molecules. Versican is more associated with the cell surface and plays multiple roles. Aggrecan has a longer core protein with multiple keratin sulfate and chondroitin sulfate chains. The molecular weight is around 1,000,000. Aggrecan aggregates on hyaluronic acid with link proteins. Aggrecan can associate with 50 times its weight in water. The resulting swelling pressure is retained by 20% by type II collagen. This results in the physiologic properties of articular cartilage. Type IX collage plays a role in matrix molecule association with type II collagen.

 

(SAE10BS.87) Which of the following is not a cause of childhood osteomalacia (rickets)? 

 

  1. Vitamin D deficiency

     

  2. Renal osteodystrophy

     

  3. X-linked hypophosphatemia

     

  4. Vitamin D malabsorption

     

  5. Vitamin K deficiency

 

PREFERRED RESPONSE 5

 

Rickets is caused by a decrease in calcium and phosphate that affects mineralization at the physes of the long bones. This disease is characterized by brittle bones; physeal widening is seen on radiographs with long-bone bowing, Looser's lines, enlargement of costal cartilages, and dorsal kyphosis. All of the above-mentioned choices are known causes of osteomalacia or rickets except vitamin K deficiency.

 

(SAE10BS.46) The World Health Organization (WHO) developed specific criteria for osteoporosis in 1994 based on the T-score obtained by dual-energy absorptiometry (DEXA). The T-score represents the number of standard deviations that the individual's bone mineral density differs from the normal peak bone mass in young adults. Osteoporosis is defined as a T-score of 

 

1 +2.5 or below.

 

2 -1.0 to -2.0.

 

  1. -2.5 or below.

     

  2. -3.0 or below.

 

5 +1.0 to +2.0.

 

PREFERRED RESPONSE 3

 

Osteoporosis is defined as a chronic, progressive disease characterized by low bone mass and decreased bone strength. Risk factors for osteoporosis include increasing age, postmenopausal women, long-term calcium deficiency, and excessive steroid or alcohol abuse. T-scores are defined as standard deviations from normal peak bone mass (young adults), whereas Z-scores compare bone density of the same age and size. Osteoporosis is defined as a T-score of -2.5 or below as defined by the WHO, and osteopenia is defined as a T-score between -1.0 and -2.5. Z-scores are not used to stratify patients into categories of "osteopenia" or "osteoporosis" in elderly patinets. They are used to aid in the diagnosis of metabolic bone disease in young patients.

 

(SAE11OS.116) A workers' compensation carrier for a local manufacturing company requests a second opinion on a 59-year-old man who sustained a crush injury to his foot and leg at work 6 months ago. His leg and foot were pinned between a forklift and a wall when an employee he was supervising lost control of the forklift. The employer suspects that the injured worker is malingering because the treating physician released him to work, but he has not returned to work. Which of the following elements of your history will best help you determine that the injured

worker does not want to return to work out of fear of a confrontation with the employee he was supervising? 

 

  1. Formality

     

  2. Empathy

     

  3. Yes-no questions

     

  4. Taking copious notes

     

  5. Sitting leaning back in a chair

 

PREFERRED RESPONSE 2

 

Empathy during the interview demonstrates compassion and earns the patient's trust; which, in turn, enables the patient to discuss any agenda or concerns he or she may otherwise feel uncomfortable revealing. It is also important to engage the patient to establish a trusting relationship and thus understand all the factors impacting the patient. A formal attitude toward the patient makes it difficult to engage the patient to be "drawn in." An engaged patient is more comfortable, reliable, and thorough when providing a history. Closed-end, yes-no questions do not allow the patient to detail all of the subtle nuances of their condition and its effect on their life. Taking copious notes likewise prevents engagement of the patient and the distraction of taking notes may cause the physician to miss an important detail. It is better to lean forward in a chair when interviewing a patient because this suggests the physician is genuinely interested, whereas leaning back in a chair suggests the physician is simply waiting for the patient to finish talking. Avoid interrupting the patient when talking.

 

(SAE13HK.97) What infection-control measure has been shown to have the most notable impact in reducing surgical-site infections? 

 

  1. Intravenous antibiotic administration within 1 hour of surgical incision

     

  2. Screening and decolonization of patients colonized with methicillin-resistant

    Staphylococcus aureus

     

  3. Horizontal laminar flow

     

  4. Use of enclosed body exhaust suits

PREFERRED RESPONSE 1

 

Timely administration of prophylactic antibiotics is the most important factor shown to decrease surgical-site infections. The use of horizontal laminar flow and body exhaust suits has not been shown to significantly affect infection rates.

 

(SAE08OS.143) Articular cartilage is divided in zones with specific arrangements of the collagen framework and proteoglycan content. The superficial zone is characterized by collagen oriented 

 

  1. randomly, and low proteoglycan content.

     

  2. randomly, and high proteoglycan content.

     

  3. parallel to the surface, and low proteoglycan content.

     

  4. parallel to the surface, and high proteoglycan content.

     

  5. perpendicular to the surface, and low proteoglycan content.

 

PREFERRED RESPONSE 3

 

Understanding collagen orientation is important to understand articular cartilage damage and disease. Articular cartilage acts as a fiber-reinforced composite matrix able to withstand and distribute physiologic loads without mechanical failure. Collagen fibers form a complex framework with a distinctive cross-sectional architecture. The superficial zone is characterized by collagen fibers oriented parallel to the joint surface and a relatively low concentration of proteoglycans. The transitional zone is characterized by larger diameter collagen fibers oriented in a more random manner, with a higher concentration of proteoglycans. The deep zone has the largest collagen fibers with a vertical arrangement. It has the highest proteoglycan concentration. Calcification of the matrix is seen in the zone of calcified cartilage.

(SAE08OS.152) Performance parameters developed by the Centers for Medicare and Medicaid Services and the Centers for Disease Control and Prevention regarding prophylactic antibiotics include 

 

  1. initiation within 1 hour of incision, use of an appropriate antibiotic, and continuation of antibiotics until all surgical drains are removed.

     

  2. initiation within 1 hour of incision, use of an appropriate antibiotic, and continuation of antibiotics for 48 hours postoperatively for all joint arthroplasty patients.

     

  3. initiation within 1 hour of incision, use of vancomycin for all patients having hardware inserted, and continuation of antibiotics for 24 hours postoperatively.

     

  4. initiation within 1 hour of incision, use of an appropriate antibiotic, and continuation of antibiotics for 24 hours postoperatively.

     

  5. initiation within 1 hour of incision, use of vancomycin for all patients having hardware inserted, and continuation of antibiotics until all surgical drains are removed.

 

PREFERRED RESPONSE 4

 

There are over 500,000 surgical site infections each year in the U.S. Patients with a surgical site infection are 60% more likely to spend time in an intensive care unit, and are twice as likely to die during the perioperative period. In 2002, the Centers for Medicare and Medicaid Services and the Centers for Disease Control and Prevention initiated the National Surgical Infection Prevention Project to decrease morbidity and mortality in surgical site infections by promoting the appropriate use of perioperative antibiotics. The three performance parameters they developed are initiation of parenteral antibiotics within 1 hour of surgical incision, selection of an appropriate antibiotic, and discontinuation within 24 hours. The entire dose of antibiotics should be administered prior to tourniquet inflation. For total joint arthroplasty, cefazolin or cefuroxime is the preferred prophylactic antibiotic. Vancomycin or clindamycin is indicated for patients with severe allergies or adverse reactions to beta-lactams. Vancomycin can also be used for prophylaxis in hospitals with a "high" frequency of methicillin-resistant Staphylococcus aureus. There is no evidence that the continuation of antibiotics until surgical drains have been removed provides any additional protection against surgical site infection.

(OBQ14.170) Which of the following is a function of siRNA (small interfering RNA)? 

 

  1. Identifies proteins

     

  2. Blocks transcription of DNA

     

  3. Separates DNA based on size

     

  4. Blocks translation of mRNA

     

  5. Identifies DNA sequences

 

PREFERRED RESPONSE 4

 

siRNA functions by causing mRNA to be broken down after transcriptions, resulting in an inability to undergo translation.

 

siRNA are short (usually 20 to 24-bp) double-stranded RNA (dsRNA) sequences with phosphorylated 5' ends and hydroxylated 3' ends. Because of their ability to block a gene of interest, they have been generating interest in the treatment of disease processes that involve gene expression.

 

Noh et al. study the affects of PD98059, an extracellular signal-regulated kinase 1/2 (ERK1/2) inhibitor, on osteosarcoma. They found that blocking the ERK1/2 pathway with PD98059 induces osteosarcoma cell death by inhibiting a potential drug-resistance mechanism.

Illustration A shows how siRNA works to block translation of mRNA. Incorrect Answers:

Answer 1: This is the function of a Western blot.

Answer 2: siRNA have not been shown to block transcription of DNA. Answer 3: This is the function of a gel electrophoresis. Answer 5: This is the function of a Southern blot.

 

(SAE13BS.39) Tendon fibroblasts detect applied strain through what mechanism? 

 

  1. Induced apoptosis from tendon loading

  2. Induced cell elongation from tendon loading

     

  3. Tendon loading-mediated cell pressurization

     

  4. Deflection of cell-cilia from tendon loading

 

PREFERRED RESPONSE 4

 

The primary cilium is a cellular extension present in nearly every cell of the body, and has been shown to have critical importance in numerous functions. Emerging evidence in tendon mechanobiology suggests that mechanotransduction signaling is also mediated by the deflection of the primary cilium in response to tendon loading. "Induced apoptosis from tendon loading" would infer that each time a tendon was loaded and a portion of the tenocytes apoptosed, there would be fewer cells left, and no hope for a hypertrophy response to the loads applied. Tendon loading-mediated cell pressurization and cell elongation were previously suggested as possible means for a mechanism to mediate mechanotransduction.

 

(SAE12TR.28) Of the following variables, which has the strongest influence on external fixator stiffness? 

 

  1. Pin diameter

     

  2. Pin spread

     

  3. Bone quality

     

  4. Stacking a second fixator bar

     

  5. Distance from bone to fixator bar

 

PREFERRED RESPONSE 1

 

Whereas all of the factors will have an impact on frame rigidity and stability, the single biggest factor is the pin diameter because it has an exponential effect.

 

(SAE13BS.44) Tendons should have what ratio of matrix protein? 

 

  1. 65% collagen type I and 35% proteoglycans

     

  2. 80% collagen type I and 20% collagen type VI

     

  3. 95% collagen type I and less than 5% collagen type III

     

  4. 95% proteglycans and 5% tenascin

 

PREFERRED RESPONSE 3

 

Tendons consist of mainly type I collagen (95%); a small amount of collagen types III, V, VI; and proteoglycans (< 5%). Proteoglycans have highly charged glycosaminoglycan side chains that attract water and help keep tendons well hydrated. Decorin is the most common proteoglycan in tendons and has been shown to bind to collagen. Tenascin-C is a glycoprotein upregulated in tendinopathy.

 

(SAE12SN.39) What complication is associated with the use of epidural morphine and steroid paste after laminectomy? 

 

  1. Surgical site infection

     

  2. Arachnoiditis

     

  3. Urinary retention

     

  4. Disk space infections

     

  5. Nerve irritation

 

PREFERRED RESPONSE 1

 

Kramer and associates conducted a retrospective review during an "epidemic" period to identify the risk factors associated with a sudden increase in the rate of surgical site infections. They found in a multivariate analysis that the use of morphine nerve paste

resulted in a 7.6-fold increase in postoperative surgical wound debridement, and an 11% rate of surgical site complications. There is no evidence in the literature verifying the incidence of postoperative urinary retention and arachnoiditis.

 

(SAE11OS.72) In a diagnostic test, the proportion of individuals who are truly free of a designated disorder identified by the test is known as 

 

  1. specificity.

     

  2. sensitivity.

     

  3. accuracy.

     

  4. positive predictive value.

     

  5. negative predictive value.

 

PREFERRED RESPONSE 1

 

Specificity refers to the proportion of individuals who are truly free of the designated disorder who are so identified by the test. Sensitivity refers to the proportion of individuals who truly have the disorder who are so identified by the test. Positive predictive value refers to the proportion of individuals with a positive test who have the disorder. Negative predictive value refers to the proportion of individuals with a negative test who are free of the disorder. Accuracy is the overall ability to identify patients with the disorder (true positives) and without the disorder (true negatives) in the study population.

 

(OBQ16.8) Which of the following is most likely to decrease surgical mortality and inpatient morbidity while simultaneously increasing surgical team adherence to life-saving steps in operating room crisis situations? 

 

  1. An intra-operative surgical team leader

  2. A surgical checklist

     

  3. The presence of a senior surgeon

     

  4. Magnet certification of nursing staff

     

  5. Exclusion of surgical residents from the operating room team

 

PREFERRED RESPONSE 2

 

The utilization of a surgical safety checklist has demonstrated substantial reductions in surgical morbidity and mortality. Checklist use has also demonstrated efficacy in increasing team adherence to life-saving care plans in operating room crises.

 

The World Health Organization concept of a surgical safety checklist concept was validated in 2009 as a way of improving surgical outcomes. Successful implementation of a surgical checklist relies upon surgeon leadership to educate staff on its rationale and the practical components of implementation in the operating room.

 

Haynes et al. evaluated a 19-item surgical safety checklist and evaluated its ability to reduce complications and deaths associated with surgery in a global population. Across 8 diverse hospitals in 8 cities around the world, their study demonstrated that implementation of a pre-surgical checklist resulted in a 50% reduction in mortality (1.5% to 0.8%) and a 37% reduction in inpatient complications (11% to 7%). The patients in the study were over the age of 16 and undergoing non-cardiac surgery.

 

Conley et al. evaluated the effectiveness of implementation of surgical safety checklists in five hospitals using a series of interviews conducted with surgeons. Analysis of the survey results demonstrated that effective implementation relied upon surgeon leadership to explain the necessity of the checklist and how to effectively implement its use. When surgeon leadership failed in these functions, hospital staff failed to comprehend the utility of the checklist and were not able to appropriately use it. These failures eventually led to institutional abandonment of the safety checklist.

 

Arriaga et al. evaluated the utility of checklists to guide the surgical team through intraoperative crisis situations (e.g., massive hemorrhage, cardiac arrest). Their study of 17 surgical teams undergoing 106 simulated surgical-crises demonstrated that use of a crisis checklist led to greater adherence to life-saving steps of a care plan. 97% of participants agreed that they desired a checklist to be present if a crisis were to occur while they themselves were undergoing surgery.

 

Incorrect answers:

Answers 1, 3-5: None of these have consistently demonstrated substantive improvement in surgical morbidity and mortality.

 

(OBQ15.166) A postmenopausal patient comes to your office for follow-up after a dual-energy x-ray absorptiometry (DEXA) test. The T-score is -0.7SD. The Z score is

-0.4SD. By World Health Organization (WHO) criteria, these DEXA findings would merit a diagnosis of 

 

  1. Normal

     

  2. Osteopenia based on Z-score findings

     

  3. Osteopenia based on T-score findings

     

  4. Osteopenia based on T- and Z-score findings

     

  5. Osteoporosis

 

PREFERRED RESPONSE 1

 

By WHO classification, her findings (T-score of -0.7SD) would merit a diagnosis of Normal bone mineral density (BMD).

 

The WHO classifies bone density in postmenopausal women based on T-scores. Patients are classified based on the lowest T score of the spine, femoral neck, trochanter, or total hip. The classification should not be used with peripheral measurements. Z-scores are used for premenopausal women, younger men, and in children. The Z score compares a patient with age-, sex-, and race-matched norms.

 

Blake and Fogelman reviewed the role of central DEXA in treatment of osteoporosis, compared with quantitative CT, peripheral DEXA and quantitative ultrasound. The advantages of central DEXA include results that can be interpreted using WHO T-score definitions, ability to predict fracture risk, and effectiveness at targeting antifracture treatments.

 

Templeton reviewed secondary osteoporosis. In women with osteoporosis, the most common causes of secondary osteoporosis include hypercalciuria, malabsorption, hyperparathyroidism, vitamin D deficiency, and exogenous hyperthyroidism. In men with osteoporosis, the most common causes of secondary osteoporosis include hypogonadism, corticosteroid use and alcoholism.

 

Unnanuntana et al. reviewed the assessment of fracture risk. The FRAX (World Health Organization Fracture Risk Assessment Tool) calculates 10-year risk of fracture based on the following variables: age, sex, race, height, weight, BMI, history of fragility fracture, parental history of hip fracture, use of oral glucocorticoids, secondary osteoporosis and alcohol use to calculate 10-year risk of fracture. They also

discussed biochemical markers of bone formation and resorption, which are useful for monitoring the efficacy of antiresorptive / anabolic therapy, and may help identify patients at high risk for fracture.

 

Illustration A shows the WHO classification. Illustration B shows a comparison between central DEXA and other methods of BMD measurement.

 

Incorrect Answers:

Answers 2, 3, 4: In a postmenopausal woman, only the T-score is used for classification. Osteopenia is defined as T score of between 1 and 2.5 standard deviations below the norm. Answer 5: Osteoporosis is defined as a T-score of more than 2.5 standard deviations below the norm.

 

(OBQ13.169) Immunological testing of anti-cyclic citrullinated peptide antibodies (anti-CCP) is most commonly used for the diagnosis and prognosis of which immunological condition? 

 

  1. Ankylosis spondylitis

     

  2. Rheumatoid arthritis

     

  3. Psoriatic arthritis

     

  4. Systemic lupus erythematosus

     

  5. Reiter's syndrome

 

PREFERRED RESPONSE 2

 

Anti-cyclic citrullinated peptide antibodies (anti-CCP) are commonly used as a marker for the diagnosis and prognosis of rheumatoid arthritis (RA).

 

Immunological studies are commonly performed to investigate cases of suspected rheumatoid arthritis. Rheumatoid factor has historically been used as a primary marker for RA. However, in more recent years, the use of anti-CCP antibodies has shown to be as sensitive as, and more specific than, rheumatoid factor (RF) in early and fully established disease. In general, anti-CCP assays equate to a sensitivity of 50-75% and a specificity of 90-95%. High levels of anti-CCP have been shown to be indicative of a more erosive disease process and may be detected before the onset of arthritis.

Gardner and Kadel reviewed the laboratory studies most commonly used in rhuematologic diseases. Standard ordering for clinically suspected RA include Rf, anti-CCP, ESR/CRP as well as other markers of autoimmune diseases such as antinuclear antibodies, anticardiolipin antibodies and lupus anticoagulant, HLA-B27, and uric acid levels.

 

Illustration A shows the sensitivity and specificity of anti-CCP vs. RF in a variety of autoimmune diseases.

 

Incorrect Answers:

Answers 1,3-5: Anti-CCP is not routinely used to diagnose and monitor these conditions.

 

(OBQ11.182) Which of the following factors has NOT been found to be a significant independent predictor of moderate or severe pain 6 months after musculoskeletal injury? 

 

  1. Failure to complete high school

     

  2. Self-reported preinjury pain-related disability

     

  3. Disability compensation

     

  4. Male gender

     

  5. Moderate or severe pain at discharge from the acute hospital

 

PREFERRED RESPONSE 4

 

Age and gender have not been found to be significant independent predictors of moderate or severe pain 6 months after musculoskeletal injury.

 

Williamson et al. performed a prospective cohort study of 1290 trauma patients in 2 Australian hospitals using a self-rated pain scale and a SF-12. They found that the prevalence of moderate or severe pain was 48% at discharge and 30% at 6 months post-injury. Failure to complete high school, self-reported preinjury pain-related disability, eligibility for compensation (payment for medical treatment, rehabilitation services, disability services, and income assistance), and moderate or severe pain at discharge from the acute hospital were found to be independent predictors of moderate or severe pain at 6 months post-injury.

Vranceanu et al. published a Level 5 review reporting that psychosocial factors are important and treatable correlates of disabling musculoskeletal pain. They encourage orthopaedic surgeons to diminish pain intensity and pain-related disability by teaming up with psychologists and other health-care providers in multidisciplinary teams to address cognitive, affective, behavioral, and social aspects of pain.

 

Incorrect Answers:

Answer 1, 2, 3, 5: Failure to complete high school, self-reported preinjury pain-related disability, disability compensation, and moderate or severe pain at discharge from the acute hospital are all factors that have been found to be significant independent predictors of moderate or severe pain 6 months after musculoskeletal injury.

 

(SAE13BS.71) Trabecular bone is remodeled through the formation of 

 

  1. cutting cones.

     

  2. Haversian canals.

     

  3. Volkmann canals.

     

  4. Howship lacunae.

 

PREFERRED RESPONSE 4

 

Trabecular bone is remodeled through osteoclast activation that creates a resorption pit known as a Howship lacuna. After the pit is formed, osteoclasts are replaced by osteoblasts that form new bone matrix. The cement line separates new bone formation from resorption. Cutting cones are created in cortical bone remodeling. Haversian canals carry nerves and blood vessels longitudinally in bone, while Volkmann canals connect different Haversian canals.

(SAE10HK.45) The American Academy of Orthopaedic Surgeons thrombophlebitis prophylaxis guidelines for patients undergoing total joint arthroplasty include which of the following? 

 

  1. General as opposed to regional anesthesia

     

  2. Preoperative assessment for risk of thromboembolic disease as well as bleeding

     

  3. Routine use of inferior vena cava filters

     

  4. Warfarin with a goal International Normalized Ratio (INR) of 2-3 for patients with standard risk of bleeding and thrombophlebitis

     

  5. Thromboembolic compression stockings (TEDs)

 

PREFERRED RESPONSE 2

 

The 2007 AAOS guidelines for thrombophlebitis prophylaxis for patients undergoing total hip and knee arthroplasty includes preoperative risk assesment for deep venous thrombosis, pulmonary embolism, and bleeding. Regional anesthesia when appropriate is suggested. Inferior vena cava filters may be appropriate in selected patients. When warfarin is used as a chemoprophylactic agent, the goal INR is less than or equal to 2 to minimize the risk of bleeding. This is in contrast to the 2004 ACCP guidelines for warfarin with a goal INR of 2-3.

 

(OBQ13.139) Which of the following situations is most likely to decrease sentinel event errors? 

 

  1. Physician and nurse training is lengthened by 20%

     

  2. Resident hours are decreased to 55 hours per week

     

  3. An environment is created where all members of the healthcare team feel empowered to express their concerns and beliefs

     

  4. Holding individuals responsible for errors in clinical judgement

     

  5. Physicians and nurses are assigned to a smaller number of patients

 

PREFERRED RESPONSE 3

Creating an environment where all members of the healthcare team feel empowered to express their beliefs increases communication, the key element in decreasing sentinel events.

 

Research has shown that 70% of sentinel event errors are caused by improper communication. Specific ways to improve communication include effective clinical handover between shifts and breaking down the "hierarchy" so that all members of the team can discuss their expectations and concerns. Barriers to effective communication include distractions, cultural differences, power distance relationships, time pressures, and lack of organization.

 

Leonard et al. describe specific clinical experiences in the application of surgical briefings, properties of high reliability perinatal care, the value of critical event training and simulation, and benefits of a standardised communication process in the care of patients transferred from hospitals to skilled nursing facilities. They recommend embedding standardized tools and behaviors to bridge differences in communications styles between clinicians.

 

Incorrect Answers:

Answer 1: Increasing training time is unlikely to decrease sentinel event errors if effective communication is not established. Answer 2: Some studies have shown increased error rates with decreased resident physician work hours. Answer 4: Holding individuals responsible for errors in clinical judgement has not specifically been cited as a way to decrease errors within a healthcare setting. Answer 5: While this may decrease sentinel event errors, this has not been to be as effective as improvements in communication.

 

(OBQ13.223) Which of the following is included in safety checks on the World Health Organization Safe Surgery Guidelines Checklist? 

 

  1. Check if antibiotics has been given within 60 minutes prior to skin incision

     

  2. Check if blood products are on standby

     

  3. Check if venous thromboembolism prophylaxis has been ordered

     

  4. Check if the implant representative is either present, or has been notified

     

  5. Check if all members of the surgical and anesthetic team have been properly credentialed

PREFERRED RESPONSE 1

 

The WHO Safe Surgery guidelines checklist requires that antibiotic prophylaxis be given within 60 minutes prior to skin incision. This check is performed between nurse, anesthetist and surgeon.

 

Preoperative checks are necessary for patient safety. On the WHO checklist, critical preoperative check points for the surgeon include: site marking, patient consent, allergies, blood loss, antibiotic prophylaxis, critical and non-routine steps, case duration, and whether preoperative imaging is readily visible.

 

The WHO Surgical Safety Checklist (see below) has 3 checkpoints: before induction of anesthesia, before skin incision, and before leaving the operating room.

 

Haynes et al. prospectively collected data on the outcomes of 3733 patients before, and 3955 patients after introduction of the Surgical Safety Checklist in 8 hospitals in 8 cities. They found an improvement in death rate (1.5% before, 0.8% after) and inpatient complication rate (11.0% before, 7.0% after). They concluded that implementation of the checklist led to reduction in death and complication rates.

Illustration A shows the WHO Surgical Safety Checklist. Incorrect Answers:

Answer 2: The checklist does not check if blood products are on standby. Answer 3: The checklist does not check if VTE prophylaxis has been ordered. Answer 4: The checklist does not concern implant representatives.

Answer 5: The checklist does not check credentialing of team members

 

(OBQ15.52) Decorin is a major controlling molecule for: 

 

  1. Myelin diameter

     

  2. Non-mineralized bone matrix density

     

  3. Tendon collagen fiber size

     

  4. Elastic cartilage thickness

     

  5. Axon branching pattern

PREFERRED RESPONSE 3

 

Decorin is a proteoglycan molecule that regulates the assembly of collagen fibrils in tendons.

 

Tendons are composed organized bundles of uniaxially arranged collagen fibrils, which are assembled together to generate force from muscles to bone. However, the variability in tendon morphology is multifactorial. At the microscopic level, decorin is an important regulator of tendon structure as it acts as a mediator of fibril growth. Variations in expression of decorin has shown to correlate with both size and density of collagen fibrils and organisation.

 

Robinson et al. measured the mechanical properties of multiple tendon tissues in mice with knock-out models of decorin. They found that the loss of decorin affected the posterior tibialis causing an increase in modulus and stress relaxation, but had little effect on the flexor digitorium longus. They concluded that tendons likely are uniquely tailored to their specific location and function at the microscopic level.

 

Zhang et al. performed a biomechanical study on tendon function. They showed that decorin-deficient mice demonstrate altered fibril structure and significantly reduced strength and stiffness. They concluded that decorin is a key regulatory molecule in tendon development.

 

Incorrect Answer:

Answer 1: Myelin diameter is organized by Schwann cells Answer 2: Non-mineralized bone matrix is formed by osteoblasts. This includes alkaline phosphatase, osteonectin, osteocalcin, etc. Answer 3: Elastic cartilage is formed from elastic microfibril, which consist of numerous proteins such as microfibrillar-associated glycoproteins, fibrillin, fibullin, and elastin.

Answer 5: Laminin, fibronectin, tenascin, and perlecan are molecules shown to be associated with axonal growth and assembly. Decorin in not a major controlling molecule in the branching pattern.

 

(SAE08OS.11) Hybrid locked plating for distal femoral fractures refers to the use of nonlocked and locked screws in the same construct. The advantages of using the combination of nonlocked and locked screws in both the proximal and distal fragments are that nonlocked screws 

 

  1. placed prior to locked screws allow use of the plate as a reduction aid and locked screws provide fixed angle support to resist varus collapse.

  2. placed after locked screws allow use of the plate as a reduction aid and locked screws provide improved fixation in osteoporotic bone.

     

  3. provide bicortical fixation and locked screws provide improved fixation in osteoporotic bone.

     

  4. provide bicortical fixation and locked screws provide fixed angle support to resist varus collapse.

     

  5. provide better fixation in the diaphysis and locked screws provide better fixation in the distal fragment.

 

PREFERRED RESPONSE 1

 

Hybrid locked plating refers to the use of nonlocked and locked screws in the same fixation construct. Hybrid plating offers the advantages of both traditional plating and locked plating. Nonlocked screws are inserted first to "lag" the bone to the plate, thereby using the plate as a reduction tool. After fixation with nonlocked screws in both the proximal and distal fragments, locked screws can be added. Locked screws in the distal fragment create a fixed angle device that is resistant to varus collapse. Locked screws in the diaphyseal fragment are indicated when there is associated osteoporosis.

 

(SAE10BS.60) A well-healed bulk proximal tibia osteoarticular allograft is removed 10 years after implantation due to arthropathy. Histologic examination of the host allograft junction site will most likely reveal 

 

  1. bridging external callus along the allograft and perpendicular callus at the junction site.

     

  2. direct osteonal penetration of the allograft with haversian remodeling.

     

  3. complete incorporation of the allograft with obliteration of the host allograft junction site.

     

  4. fibrovascular invasion with absence of callus formation.

     

  5. gap formation with resorption of the allograft surface.

 

PREFERRED RESPONSE 1

Retrieval studies of well-fixed bulk allografts reveal that the junction site heals with bridging external callus and there is persistence of callus perpendicular to the junction site. External callus is annealed to the surface of the allograft. There is very little penetration of the allograft and the bone graft is not remodeled. Direct osteonal penetration of the allograft with haversian remodeling defines primary bone healing seen in fractures, which does not occur with allografts. Fibrovascular tissue is seen early in the healing phase of the cancellous portion of the allografts.

 

(SAE08OS.103) Deep venous thromboses are primarily composed of red blood cells and fibrin. What is fibrin? 

 

  1. Platelet cytoskeletons

     

  2. Fragmented collagen

     

  3. A type of thromboplastin

     

  4. A product of the coagulation cascade

     

  5. A product of the fibrinolytic system

 

PREFERRED RESPONSE 4

 

Fibrin is the end product of the coagulation cascade that results from the cleavage of fibrinogen by thrombin (factor IIa). Platelets are a key component in arterial thromboses. Collagen initiates the intrinsic arm of the coagulation cascade. Thromboplastin is a procoagulant substance that triggers the extrinsic arm of the coagulation cascade. The fibrinolytic system is the regulatory pathway that breaks down cross-linked fibrin.

 

(SAE13BS.51) What proteinaceous compound binds to hyaluronic acid to function as an effective boundary molecular layer in articular cartilage? 

  1. Lubricin

     

  2. Vitronectin

     

  3. Aggrecan

     

  4. Chondroitin sulfate

 

PREFERRED RESPONSE 1

 

Hyaluronic acid (HA) is abundant in cartilage and synovial fluid and is thought to be integral to joint lubrication, although its role is not clearly understood. HA binds to lubricin, a glycoprotein, creating a cross-linked network. Boundary lubrication occurs when the fluid film has been depleted and the contacting bearing surfaces are separated only by a boundary lubricant of molecular thickness, which prevents excessive bearing friction and wear. In articular cartilage, this monolayer of glycoprotein is adsorbed on each of the opposing articular surfaces. Friction experiments in a porcine model have shown that with compression, HA diffuses out of the cartilage and becomes physically trapped and constricted by the collagen network at the interface. This in effect creates a "boundary lubricant." Vitronectin is a glycoprotein similar in the N and C terminal to lubricin. It does not bind to HA. Aggrecan is the second-most-common protein by dry weight of cartilage extracellular matrix. Aggrecan interacts with HA and link proteins to create a proteoglycan aggregate that attracts water to cartilage and gives the tissue its viscoelastic properties. Chondroitin sulfate contributes to matrix proteoglycan structure rather than boundary lubrication.

 

(SAE10SM.88) What is the primary goal of the initial (acute) rehabilitation phase of an overhead athlete's shoulder? 

 

  1. Improve flexibility

     

  2. Strengthen muscles

     

  3. Enhance power and endurance

     

  4. Regain neuromuscular control

     

  5. Perform functional drills

PREFERRED RESPONSE 1

 

The goal in the initial phase of shoulder rehabilitation is to improve flexibility, reestablish baseline dynamic stability, normalize muscle balance, and restore proprioception. In the advanced strengthening and final phase, the goals are to initiate aggressive strengthening drills, enhance power and endurance, perform functional drills, and to gradually initiate throwing activities.

 

(OBQ13.18) Which of the following statements is correct regarding Vitamin D? 

 

  1. 1,25-dihydrocholecalciferol is the best laboratory study to determine a Vitamin D deficiency

     

  2. 25-hydroxycholecalciferol is the active form of Vitamin D

     

  3. 24,25-dihydroxycholecalciferol is an inactive form of Vitamin D

     

  4. 1,25-dihydrocholecalciferol is converted to 25-hydroxycholecalciferol in the kidney

     

  5. The half-life of 1,25-dihydrocholecalciferol is longer than 25-hydroxycholecalciferol

 

PREFERRED RESPONSE 3

 

24,25-dihydroxycholecalciferol in an inactive form of Vitamin D. High levels of 1,25-dihydroxyvitamin D stimulate the enzymatic production of 24,25-dihydroxyvitamin D, the inactive form of vitamin D, thereby self-regulating the action of 1,25-dihydroxyvitamin D.

 

Vitamin D is paramount to proper calcium homeostasis and has important clinical implications in the orthopaedic patient. Vitamin D3 is synthesized in the skin and is converted to 25-hydroxycholecalciferol in the liver. 25-hydroxycholecalciferol is then converted in the kidney into 1,25-dihydroxycholecalciferol, the active form of vitamin

D. The best test to determine Vitamin D deficiency is the measurement of 25-hydroxycholecalciferol, as it has a longer half-life and circulating levels are 1,000x more than 1,25-dihydrocholecalciferol.

 

Patton et al. review the importance of Vitamin D in the orthopaedic patient. They discuss the implications of Vitamin D deficiency, and urge orthopaedic surgeons to be proficient in both the diagnosis and treatment of the condition.

Bogunovic et al. measured the levels of 25-hydroxycholecalciferol in 723 patients who were to undergo orthopaedic surgery. 40% of these patients were noted to be deficient in Vitamin D, with the highest rates in patients scheduled to undergo trauma and sports surgery.

 

Illustration A reviews Vitamin D metabolism. 24,25-dihydroxycholecalciferol is referred to as pre-Vitamin D.

 

Incorrect Answers:

Answer 1: 25-hydroxycholecalciferol is the best laboratory study to determine Vitamin D deficiency due to its long half-life and high circulating levels. Answer 2: 1,25-dihydroxycholecalciferol is the active form of Vitamin D. Answer 4: 25-hydroxycholecalciferol is then converted in the kidney into 1,25-dihydroxycholecalciferol.

Answer 5: The half-life of 25-hydroxycholecalciferol is 2-3 weeks, while the half-life of 1,25-dihydrocholecalciferol is only 4-6 hours.

 

(OBQ15.203) Cortical bone demonstrates viscoelastic behavior as its mechanical properties are sensitive to strain rate and duration of applied load. Regarding longitudinal strain in cortical bone, which of the following statements regarding this characteristic is true? 

 

  1. As strain rate increases, both elastic modulus and ultimate strength increase

     

  2. As strain rate increases, elastic modulus remains unchanged but ultimate strength increases

     

  3. As strain rate increases, elastic modulus increases but ultimate strength decreases

     

  4. As strain rate increases, both elastic modulus and ultimate strength decrease

     

  5. As strain rate increases, elastic modulus increases but ultimate strength remains unchanged

 

PREFERRED RESPONSE 1

 

As strain rate increases, both elastic modulus and ultimate strength increase.

 

For LOW strain rates typical of normal activity (physiological strain rates of <0.1/s), bone is ELASTIC and DUCTILE (increasing ultimate strain with increasing strain rate). There is a ductile-to-brittle transition with increasing strain rate from normal to

supranormal rates. For EXTREMELY HIGH supranormal strain rates (>0.1/s, high impact trauma), bone is VISCOELASTIC and BRITTLE (low ultimate strain with increasing strain rate). Bone also becomes stronger and stiffer (higher modulus, steeper slope of stress-strain plot) as strain rate increases. This viscoelastic property helps in damping muscle contracture.

 

Natali and Meroi reviewed studies examining mechanical properties of bone. Mechanical properties are correlated with moisture, deformation rate, density and region of bone. Mechanical adaptation of bone is affected by strain rate (rate at which bone is deformed), strain mode (tension, compression, shear), strain direction (direction of strain relative to bone surface), strain frequency (cycles/second), stimulus duration (period over which deformation cycles are applied), strain distribution (pattern of strain magnitude across bone section) and strain energy (energy stored during deformation).

Illustration A shows the mechanical properties of bone with increasing strain rates. Incorrect Answers:

Answers 2, 3, 4, 5: As strain rate increases, elastic modulus and ultimate strength

increase. During normal activity, as strain rate increases, bone is more ductile. With high impact trauma, bone is more brittle.

 

(SAE13BS.54) Amphotericin exerts antifungal activity by 

 

  1. inhibiting DNA-dependent RNA polymerase.

     

  2. blocking folic acid synthesis.

     

  3. binding to sterols and disrupting the cell membrane.

     

  4. binding to cellular ribosomes and inhibiting protein synthesis.

 

PREFERRED RESPONSE 3

 

Antifungals such as amphotericin and nystatin bind to sterols in the cell membrane and disrupt its integrity, allowing diffusion of macromolecules and causing cell death. Sulfonamides and trimethopim mimic the metabolite substrate and block synthesis of metabolites such as folic acid. Rifampin inhibits bacterial RNA polymerase, blocking synthesis of RNA. Tetracycline, chloramphenicol, and clindamycin bind to ribosomes and block bacterial growth by inhibiting protein synthesis.

 

(SAE10BS.84) Osteoarthritis (OA) affects the matrix of articular cartilage, which includes changes in collagen type and content. During early arthritis, which collagen type is found to be increased? 

 

  1. I

     

  2. IX

     

  3. X

     

  4. XI

     

  5. XII

 

PREFERRED RESPONSE 3

 

Type X collagen is seen in articular cartilage in the region of the interface with subchondral bone in most OA patients. The threshold of chondrocyte sensitivity to inducing type X collagen mRNA production is more than 2.5% local strain. Increased local strains above the threshold results in an increase of Col X mRNA expression. Such quantitative analysis has important implications for our understanding of mechanosensitivity of cartilage and mechanical regulation of chondrocyte gene expression. None of the other collagen types are shown to increase in osteoarthritic articular cartilage.

 

(OBQ15.127) Metastatic disease of several cancers create lytic lesions because these cancers: 

 

  1. directly produce osteoprotegerin

     

  2. stimulate osteoblasts to produce osteoprotegerin

     

  3. directly produce receptor activator of nuclear factor kappa beta ligand (RANKL)

  4. stimulate osteoclasts to produce receptor activator of nuclear factor kappa beta ligand (RANKL)

     

  5. directly produce an analog to calcitonin

 

PREFERRED RESPONSE 3

 

Several cancers directly produce receptor activator of nuclear factor kappa beta ligand (RANKL) which leads to bone resorption and the lytic lesions seen on radiographs.

 

Cancers that originate or metastasize to bone can be blastic, lytic, or mixed when viewed on radiographs. Lytic lesions appear radiolucent due to bone resorption that occurs around the cancer cells. This is secondary to direct production and release of RANKL by the cancer cells themselves. RANKL then stimulates osteoclastogenesis and an increase in local bone resorption. Denosumab is a monoclonal antibody against RANKL that has been shown to decrease rates of pathologic fractures.

 

Lynch et al. review the progression of metastatic disease. They note that cancer cells can stimulate both osteogenesis and osteolysis, and that this pathologic increase in bone matrix turnover is what allows the cancer to progressively expand or metastasize to distant sites. They discuss the role of matrix metalloproteinases as primary regulators of this process.

 

Illustration A is a diagram illustrating how the binding of RANKL to receptors on the surface of osteoclast precursors activates them and begins the process of bone resorption. Cancer cells can directly produce RANKL thus increasing the rate of local bone resorption.

 

Incorrect Answers:

Answers 1 and 2- Osteoprotegerin acts by sequestering RANKL as a decoy receptor and causes a decrease in bone resorption. Answer 4- Osteoclasts do not secrete RANK-ligand, rather they express the receptor (RANK) for the ligand. Answer 5- Calcitonin acts directly on osteoclasts to inhibit bone resorption.

 

(SAE10HK.11) Which of the following best describes the legal definition of standard of care? 

 

  1. Conforming to a majority norm

     

  2. Providing the most minimally acceptable care

  3. Providing average care

     

  4. Providing the care you would expect for yourself or a loved one

     

  5. Reasonable treatment that exhibits knowledge, skill, diligence, and care

 

PREFERRED RESPONSE 5

 

The standard of care is a legal concept that is elusive and amorphous, although the term is used widely by physicians to mean different things. Different state courts across the United States have also applied different meanings to the term "standard of care." Most commonly, the standard of care is that which a reasonable physician would have done under similar circumstances. Expert testimony from other physicians is often required to educate a jury in a medical malpractice trial about the applicable standard of care. As a general rule, treatment that exhibits knowledge, skill, diligence, and care on the part of the physician is likely to fall within the standard of care, regardless of variations in the definition of this term.

 

(SAE10SM.86) An otherwise healthy 25-year-old man underwent a right anterior cruciate ligament reconstruction with a bone-patellar tendon-bone allograft. Routine preimplantation cultures of the allograft taken by the surgeon were positive for coagulase-negative Staphylococcus 5 days postoperatively. The patient has exhibited no evidence of clinical infection and his postoperative course has been uncomplicated during this time. What is the ideal management of this patient? 

 

  1. Observation

     

  2. Oral antibiotics for 6 weeks

     

  3. IV antibiotics for 6 weeks

     

  4. Arthroscopic irrigation and debridement with graft retention

     

  5. Arthroscopic irrigation and debridement with graft removal

 

PREFERRED RESPONSE 1

 

The incidence of preimplantation positive cultures of musculoskeletal allografts used for anterior cruciate ligament reconstruction has varied between 4.8% and 13.3%. Interestingly, in none of the studies evaluating this issue did any of the patients

implanted with a "contaminated" graft develop a clinical infection. The results of the current literature suggest that the treatment of low-virulence organisms is unnecessary if no evidence of clinical infection exists. Preimplantation cultures do not appear to correlate with clinical infection. Therefore, the routine culture of allograft tissue is not recommended.

 

(OBQ13.126) A prospective, randomized controlled trial of 150 patients undergoing total hip arthroplasty is performed to test whether repair of the capsule during a posterior approach reduces post-operative dislocations in the first three months. The study found no difference in dislocation rate if the capsule was repaired versus not repaired (p = .34). Subsequently, a multicenter follow-up study of 2000 patients showed that repairing the capsule led to a decreased dislocation rate in the first three months (p = .03). Assuming the second study reflects reality, which of the following errors occurred in the first study? 

 

  1. Observer bias

     

  2. Type-II error

     

  3. Alpha error

     

  4. Type-I error

     

  5. Confounding error

 

PREFERRED RESPONSE 2

 

In this situation, the null hypothesis was accepted when it should have been rejected. This is a type-II error.

 

A study can have two types of errors. Type-I errors, or alpha errors, occur when the null hypothesis is rejected when it should have been accepted. The alpha level refers to the probability of a type-I error. By convention, the alpha level of significance is set at 0.05, which means that we accept the finding of a significant association if there is less than a one in twenty chance that the observed association was due to chance alone. Type-II errors, or beta errors, occur when the null hypothesis is accepted when it should be rejected. This often occurs when studies are underpowered. In the example above, the null hypothesis is that repair of the capsule does not reduce dislocations within the first three months. Since the first study did not show a

statistically significant difference, the null hypothesis was accepted. Since a more powered study showed that repair of the capsule does reduce dislocations, the null hypothesis should have been rejected in the initial study (if it was adequately powered).

 

Fosgate et al. review the importance of sample size calculations when performing research. They state that sample size ensures statistical significance if the subsequent data collection is perfectly consistent with the assumptions made for the sample size calculation (assuming power was set as 50% or greater).

 

Illustration A shows the difference between type-I and type-II errors. Video V is a lecture discussing statistical definition review of PPV, NPV, sensitivity and specificity.

 

Incorrect Answers:

Answer 1: Observer bias is when the observer (usually the investigator) influences the results of an experiment as a result of their own bias. Answer 3: Alpha errors are the same as type I error (see below). Answer 4: A type-I error would reject the null hypothesis when it is true. Answer 5: A confounder is a variable that has associations with both the dependent and independent variables, potentially distorting their relationship. Confounders are not technically considered "errors," but instead are variables that properly constructed studies attempt to avoid.

 

(SAE10BS.54) Fluoroquinolones work by inhibiting what enzyme? 

 

  1. DNA gyrase

     

  2. RNA gyrase

     

  3. DNA polymerase

     

  4. Ribosomal transferase

     

  5. DNA telomerase

 

PREFERRED RESPONSE 1

 

Fluoroquinolones are commonly used antibiotics used to treat orthopaedic infections and are popular because of their equivalency in IV or oral formulations and their gram-positive and gram-negative antibacterial activity. It is part of a larger class of

antibiotics called quinolones that interfere with DNA metabolism. Fluoroquinolones are synthetic derivatives of nalidixic acid. Its mechanism of action involves interfering with the function of DNA gyrase. DNA gyrase ia a type-II topoisomerase, and "unwinds" DNA to allow replication.

 

(SAE10BS.64) Moderate distance running has what effect on knee articular cartilage in asymptomatic distance runners? 

 

  1. Increased loss of collagen tensile integrity in the superficial zone

     

  2. Increased production of glycosaminoglycans

     

  3. Decreased collagen in the middle zone

     

  4. Loss of cartilage thickness that correlates with the total distance run during the course of adult life

     

  5. Collagen loss in the medial compartment and collagen increase in the lateral compartment

 

PREFERRED RESPONSE 2

 

Articular cartilage in the knee adapts to exercise by increasing the glycosaminoglycan (GAG) content. Increased GAG content is important to cartilage health. There is a higher glycosaminoglycan content in the lateral femoral cartilage compared to the medial femoral cartilage. Using T1 gadolinium enhanced MRI, the GAG content is seen to be increased in moderate distance runners. Moderate exercise may be a good treatment, not only to improve joint symptoms and function, but also to improve the knee cartilage GAG content in patients at high risk of developing osteoarthritis.

 

(SAE07PE.74) The husband of a 22-year-old woman has hypophosphatemic rickets. The woman has no orthopaedic abnormalities, but she is concerned about her chances of having a child with the same disease. What should they be told regarding this disorder? 

  1. Their sons will have a 50% chance of having this X-linked dominant disorder.

     

  2. All of their daughters will be carriers or will have this disorder.

     

  3. They should be advised to not have any children as the risk of having boys with the disorder and girls who will be carriers is too hard for any parent.

     

  4. As long as the woman does not carry the trait, the children will not be affected because the husband has the disease and this is an X-linked dominant disorder.

     

  5. Their sons or daughters may be born with this disorder, but males are more severely affected.

 

PREFERRED RESPONSE 2

 

Hypophosphatemia is a rare genetic disease usually inherited as an X-linked dominant trait. The fact that the woman has no skeletal manifestations would indicate that the husband has the X-linked mutation. The disease is more severe in boys than it is in girls. The husband will not transmit the disease to his sons. However, all of their daughters will be affected either with the disease or as carriers. If the woman has the disease or the trait, there is a 50% chance that her sons will inherit the disease and a 50% chance that her daughters will be carriers or have a milder form of the disease. Parents should be advised to have genetic counseling so they can be informed when deciding whether to have children.

 

(SAE13BS.94) What is the recommended optimal timing of presurgical antibiotic administration to prevent infection in patients undergoing total joint replacement surgery? 

 

  1. Within 1 hour before incision

     

  2. Within 2 hours before incision

     

  3. Immediately after incision

     

  4. Within 1 hour after incision

     

    PREFERRED RESPONSE 1

    The current recommendation for antibiotic prophylaxis for major orthopaedic surgical procedures is to administer intravenous antibiotics within 1 hour of surgical incision. Redosing of antibiotics should occur 3 to 4 hours after the initial dose for procedures that extend beyond 3 to 4 hours. Little evidence supports postsurgical antibiotic use beyond 24 hours. As you move beyond 1 hour from time of administration of antibiotics, risk for infection increases and rates of bacterial cell death decline. It is not acceptable to administer presurgical antibiotics after incision.

     

    (SAE10BS.6) Massive cortical structural allografts are commonly used in oncologic and arthroplasty surgery. What percent of cortical structural allografts fracture due to insufficiency? 

     

    1 0% to 5%

     

    2 15% to 30%

     

    3 50% to 60%

     

    4 70% to 80%

     

  5. 80% to 100%

 

PREFERRED RESPONSE 2

 

Allograft is available in particulate and structural forms. Particulate allograft has a higher rate of incorporation than structural but adds little structural support. Cortical allograft incorporation occurs slowly and the bulk of the graft fails to remodel and remains devascularized. Stress fractures eventually occur in approximately 25% of structural grafts used in tumor surgery.

 

(SAE08AN.101) You are interested in learning a new technique for minimally invasive total knee arthroplasty. The Keyhole Genuflex system seems appealing to

you because the instrumentation comes with wireless controls. Which of the following represents an acceptable arrangement? 

 

  1. The local Keyhole representative has invited you and your spouse out to dinner at a local restaurant to discuss your interest in their new minimally invasive total knee system, the Keyhole Genuflex knee.

     

  2. Keyhole has offered to pay your tuition to attend a CME course sponsored by the American Association of Hip & Knee Surgeons where both the Genuflex and the competing Styph total knee are discussed and demonstrated.

     

  3. Keyhole will pay your expenses to attend a workshop, in Phoenix at their company headquarters, to learn how to implant the Genuflex knee and to see how the implant is manufactured and tested.

     

  4. Keyhole will pay you $500 for each knee that you implant if you switch from your current total knee system.

  5. After you have implanted 25 Genuflex knees, Keyhole will list you on their website as a consultant, pay you a consulting fee of $5,000 per year, and invite you to a golf tournament for their consultants at a resort.

 

PREFERRED RESPONSE 3

 

Both the AAOS and AdvaMed, the medical device manufacturer's trade organization, have written guidelines that address potential conflicts of interest regarding interactions between physicians and manufacturer's representatives when it comes to patients' best interest. The AAOS feels that the orthopaedic profession exists for the primary purpose of caring for the patient and that the physician-patient relationship is the central focus of all ethical concerns. When an orthopaedic surgeon receives anything of significant value from industry, a potential conflict of interest exists. The AAOS believes that it is acceptable for industry to provide financial and other support to orthopaedic surgeons if such support has significant educational value and has the purpose of improving patient care. All dealings between orthopaedic surgeons and industry should benefit the patient and be able to withstand public scrutiny. A gift of any kind from industry should in no way influence the orthopaedic surgeon in determining the most appropriate treatment for his or her patient. Orthopaedic surgeons should not accept gifts or other financial support with conditions attached. Subsidies by industry to underwrite the costs of educational events where CME credits are provided can contribute to the improvement of patient care and are acceptable. A corporate subsidy received by the conference's sponsor is acceptable; however, direct industry reimbursement for an orthopaedic surgeon to attend a CME educational event is not appropriate. Special circumstances may arise in which orthopaedic surgeons may be required to learn new surgical techniques demonstrated by an expert or to review new implants or other devices on-site. In these circumstances, reimbursement for expenses may be appropriate.

 

(OBQ15.226) Which of the following materials best approximates the Young's modulus of elasticity of cortical bone? 

 

  1. Titanium

     

  2. Cobalt-chrome alloy

     

  3. Alumina

     

  4. Zirconia

     

  5. Stainless steel

 

PREFERRED RESPONSE 1

 

Of the materials listed titanium (100GPa) has an elastic modulus closest to cortical bone (approximately 18GPa) as well as cancellous bone (approximately 2GPa).

 

Titanium is a material that is light, highly ductile, strong and corrosion resistant. However, titanium has poor wear resistance and is notch sensitive. It is commonly used as an orthopaedic implant materials because it has torsional and axial stiffness (moduli) that most closely mimics bone. Young’s modulus is constant and different for each material and represents the material's ability to maintain shape under external loading.

 

Rho et al found that the average Young's modulus for trabecular bone measured ultrasonically and mechanically was 14.8 GPa (S.D. 1.4) and 10.4 (S.D. 3.5), respectively. The average Young's modulus of microspecimens of cortical bone measured ultrasonically and mechanically was 20.7 GPa (S.D. 1.9) and 18.6 GPa (S.D. 3.5), respectively.

 

Illustration A depicts a stress vs. strain curve. The slope of the line in the elastic zone represents the Young Modulus of Elasticity.

 

Incorrect Answers:

Answer 2: Cobalt-chrome alloy is approximately 240 GPa Answer 3: Alumina is approximately 340 GPa Answer 4: Zirconia (Ceramic) = 248 GPa Answer 5: Stainless steel is approximately 240 GPa

 

(OBQ14.177) A patient undergoing joint arthroplasty is put on a drug that competitively inhibits the activation of an enzyme that breaks down Factor Ia. The drug is 

 

  1. Dabigatran

     

  2. Tranexamic acid

     

  3. Rivaroxaban

     

  4. Fondaparinux

     

  5. Heparin

 

PREFERRED RESPONSE 2

 

Factor Ia is fibrin. The enzyme that breaks down fibrin is plasmin. Tranexamic acid (TXA) is an antifibrinolytic that prevents the activation of plasmin from the inactive zymogen plasminogen.

 

Tranexamic acid competitively inhibits the activation of plasminogen to plasmin by binding to specific sites on both plasminogen and plasmin. Tranexamic acid has roughly eight times the antifibrinolytic activity of an older analogue, e-aminocaproic acid. It is used during joint replacement surgery to reduce blood loss and the need for transfusion.

 

Watts et al. review strategies for minimizing blood loss and transfusion. They recommend 1g of TXA prior to incision, and 1g at wound closure. They also recommend giving fluids for symptoms of anemia, rather than transfusion, as even high risk patients do well with sufficient intravascular volume even with low hemoglobin levels.

 

Imai et al. evaluated TXA in 107 patients undergoing THA. They found that intraoperative blood loss after preoperative TXA administration was lower than both control and postoperative TXA administration groups. They recommend using 1 g of TXA 10 minutes before surgery and 6 hours after the first administration to best reduce blood loss during THA.

 

Gillette et al. retrospectively reviewed 2046 patients receiving TXA for THA or TKA together with either aspirin, warfarin or dalteparin. They found that the rates of symptomatic DVT (0.35%, 0.15%, and 0.52%, respectively) and nonfatal PE were similar (0.17%, 0.43%, and 0.26%, respectively) for the 3 drugs respectively. They recommend TXA to decrease blood loss and transfusion.

 

Illustration A shows the role of tranexamic acid in the fibrinolytic cycle and the

clotting cascade.

 

Incorrect Answers:

Answer 1: Dabigatran is an oral direct thrombin inhibitor. Answer 3: Rivaroxaban is an oral direct factor Xa inhibitor. It is FDA approved for hip and joint arthroplasty DVT prophylaxis. Answer 4: Fondaparinux is an indirect factor Xa inhibitor that works through antithrombin III. Unlike heparin, it is selective for factor X and does not have effect on thrombin.

Answer 5: Heparin is an anticoagulant that binds and activates antithrombin III. Activated antithrombin III then inactivates thrombin and factor Xa.

 

(OBQ15.115) A researcher studies growth factors that have positive effects on cartilage healing. In vivo and in vitro experiments are performed with Growth Factor

A. The properties of Growth Factor A include (1) it is the most widely investigated growth factor in cartilage repair, (2) it increases extracellular matrix synthesis in cartilage and mesenchymal stem cells, and (3) it also triggers synovial proliferation and fibrosis. Which of the following is most likely to be Growth Factor A? Review Topic

 

  1. Interleukin-1 (IL-1)

     

  2. Tumor necrosis factor-alpha (TNF-alpha)

     

  3. Fibroblast growth factor (FGF)

     

  4. Transforming growth factor-beta 1 (TGF-beta1)

     

  5. Platelet-derived growth factor (PDGF)

 

PREFERRED RESPONSE 4

 

TGF-beta 1 stimulates the synthesis of extracellular matrix (ECM) and causes synovial proliferation and fibrosis.

 

TGF-beta is the most thoroughly investigated member of the TGF-beta superfamily. This group includes TGF-beta1, BMP-2, and BMP-7. Besides the above activities, TGF-beta1 also stimulates chondrocyte synthetic activity and decreases the catabolic activity of IL-1.

 

Fortier et al. reviewed the role of growth factors in cartilage repair and modification

of osteoarthritis. They found that members of the TGF-beta superfamily, FGF family, IGF1, and PDGF have all been investigated as possible treatment augments in the management of chondral injuries and early arthritis. They concluded that more research was necessary before routine application.

 

Illustration A shows a summary of the different growth factors and their effects on cartilage.

 

Incorrect Answers:

Answer 1: IL-1 is catabolic and leads to cartilage breakdown rather than synthesis. Answer 2: TNF-alpha is a proinflammatory cytokine. It does not lead to cartilage synthesis.

Answer 3: Although FGF-2 increases proteoglycan synthesis and cell proliferation, it also increases inflammation and osteophyte formation and does not aid in healing of cartilage defects. FGF-18 is less well studied. Answer 5: PDGF has no adverse or positive effects on cartilage when used in vivo. Much evidence on its role is extrapolated from the effects of PRP.

 

(SAE11OS.103) Results of a study demonstrating no difference between treatments when a difference truly exists is an example of which of the following? 

 

  1. Statistical insignificance

     

  2. Type I error

     

  3. Type II error

     

  4. Fragile p-values

     

  5. Negative predictive value

 

PREFERRED RESPONSE 3

 

A type II error (also known as a beta error) occurs when results demonstrate that two groups are similar when, in reality, they are different (with regard to the statistic being measured). Type I errors show that a difference exists when, in reality, no difference exists. A statistically insignificant result may lead an investigator to conclude that no difference exists between two groups; this may be correct (and therefore not a type II error). The concept of `fragile` p-values is that small sample sizes may result in wide variability of p-values with only one change in a data point for a given group. This singular change could be a chance occurrence, but it still can affect the statistical

significance of the outcomes analysis. Fragility of p-values is limited by increasing sample sizes. Negative predictive value is the proportion of patients with negative test results who are correctly diagnosed.

 

 

(SAE13HK.51) A 72-year-old man was scheduled for left total knee replacement. He has a history of hypertension and deep venous thrombosis (DVT) in his right lower extremity after an ankle fracture 2 years ago that was treated nonsurgically. The patient asked about the recommended types of DVT prophylaxis or investigations. Based on the 2011 AAOS Clinical Practice Guideline, Preventing Venous Thromboembolic Disease in Patients Undergoing Elective Hip and Knee Arthroplasty, what is an acceptable option? 

 

  1. Six weeks of acetylsalicylic acid postsurgically beginning the evening of surgery

     

  2. Six weeks of low-molecular-weight heparin beginning the morning after surgery

     

  3. Routine duplex scans of both lower extremities before hospital discharge to ensure the patient has not developed another DVT

     

  4. Use of pneumatic calf compressors on both lower extremities while in the hospital and 4 weeks of warfarin starting the evening of surgery

 

PREFERRED RESPONSE 4

 

The 2011 AAOS guideline, Preventing Venous Thromboembolic Disease in Patients Undergoing Elective Hip and Knee Arthroplasty, recommends the combined use of mechanical and pharmacological prophylaxis in patients who have a history of previous thromboembolism. This recommendation is a consensus opinion of the work group that established these guidelines because there is no other reliable evidence for this clinical scenario. There is strong evidence against the use of routine duplex scans in patients undergoing hip and knee replacement. The remaining two responses are less appropriate because they do not include the use of mechanical prophylaxis immediately after surgery.

(SAE13BS.84) You are studying a single continuous variable after administration of a defined treatment intervention. Your statistician informs you the data are not normally distributed. What is the best test to analyze the data? 

 

  1. Analysis of variance (ANOVA)

     

  2. Regression analysis

     

  3. Student t test

     

  4. Mann-Whitney U test

 

PREFERRED RESPONSE 4

 

The Mann-Whitney U test is used when data are nonparametric, meaning either not normally distributed or variances are not equal among groups. Both the Student t test and ANOVA are used with parametric, normally distributed data. A regression analysis is a statistical model that allows for control of potentially confounding variables. It is used to assess the relationship between a dependent variable and (usually) multiple independent variables.

 

(SAE10BS.15) Phosphate administration decreases urinary calcium excretion through which of the following actions? 

 

  1. It inhibits PTH secretion thus enhancing calcium reabsorption.

     

  2. It creates a complex with calcium in the intestine to decrease available calcium for absorption.

     

  3. It competes with calcium in the bone and soft tissue, thus increasing the filtered load of calcium.

     

  4. It has a direct effect on the proximal tubule of the kidney.

     

  5. It increases ionized calcium through binding to serum proteins.

 

PREFERRED RESPONSE 2

 

The kidneys are responsible for monitoring and regulating calcium homeostasis as well as for controlling levels of phosphate, magnesium, and other minerals. The

kidneys act both as target organs for parathyroid hormone (PTH) and for excreting it. The proximal convoluted tubules of the kidneys are the site of production of 1,25-dihydroxycholecalciferol (the active form of vitamin D following hydroxylation of 25-hydroxycholecalciferol catalyzed by 1-hydroxylase), the foremost regulator of intestinal calcium absorption. This hormone also promotes osteoclastic resorption of bone and the feedback inhibition of PTH synthesis. Modest changes in the efficacy of renal excretion dramatically alter the body's ability to maintain mineral homeostasis. Phosphate reduces urine calcium excretion through several extrarenal mechanisms. An increase in phosphate will directly stimulate PTH secretion and can reduce ionized calcium, also enhancing PTH secretion. The increased PTH will enhance calcium resorption. Phosphate will complex with calcium in the intestine, decreasing the amount of calcium for absorption. It can also complex with calcium in the bone and soft tissues, decreasing the filtered load of calcium. Decreased phosphate will result in hypercalcuria. There appears to be a direct effect of phosphate to decreased calcium reabsorption in the distal nephron.

 

(SAE10SM.11) A college athlete on a scholarship has a medical condition that you feel presents a life-threatening risk to him with participation in athletics. Because of the gravity of this decision and the potential effect it can have on the student/athlete's future, the college asks for your guidance. As the team physician for the college, what is your ethical obligation? 

 

  1. Ban the athlete from sports participation.

     

  2. Allow the athlete to participate as it is his constitutional right to do so.

     

  3. Advise the college to revoke the athlete's college scholarship.

     

  4. Offer no opinion as it is a matter strictly between the college and the athlete.

     

  5. Recuse yourself from all decision making and advise the athlete to get an opinion from a third-party physician who is not employed by the college or university.

 

PREFERRED RESPONSE 1

 

There is legal precedent for banning a scholarship athlete from participation in college athletics if the physician feels that it presents a significant physical risk to the athlete. The courts have decided that the athlete has no constitutional right to participate in NCAA sports, and as a team physician you must advise your athlete and the school as to the best course of action. The athlete must be allowed to keep his or her college scholarship.

 

(SAE10BS.21) Which of the following therapies has a direct anabolic effect on bone? 

 

  1. Bisphosphonates

     

  2. Tumor necrosis factor-a (TNF-a) inhibitors

     

  3. Calcitonin

     

  4. Intermittent PTH

     

  5. Corticosteroids

 

PREFERRED RESPONSE 4

 

Bisphosphonates are antiresorptive agents that act by targeting osteoclatic bone resorption. Calcitonin also inhibits osteoclastic bone resorption. Excessive cortisol decreases intestinal calcium absorption, increases calcium loss from the kidney, inhibits bone matrix formation, and causes secondary hyperparathyroidism. TNF-a inhibits osteoblastic activity and enhances osteoclastic bone resorption. Intermittent administration of PTH isoforms is anabolic, whereas persistent elevated levels of PTH promotes bone resorption.

 

(SAE07SM.96) Closed-chain exercise differs from open-chain exercise in which of the following ways? 

 

  1. Distal portion of the extremity is free during exercise

     

  2. More commonly used in upper extremity exercise

     

  3. Predictable movement is produced by co-contraction of muscles

     

  4. Joint compression is decreased

  5. Usually involves a single joint

 

PREFERRED RESPONSE 3

 

Closed-chain exercise requires the distal portion of the extremity to be fixed. It is more commonly used in lower extremity exercise, and movement is produced by co-contraction of muscles. Joint compression is increased, and multiple joints are involved with closed-chain exercise. In open-chain exercise, the distal portion of the extremity is free.

 

(SAE08UE.117) A patient with rheumatoid arthritis has a rupture of the extensor digitorum communis to 4 and 5. You are planning to perform an extensor indicis proprius (EIP) tendon transfer. What effect will this have on index finger extension? 

 

  1. No effect

     

  2. Index finger weakness

     

  3. Index metacarpophalangeal hyperextension

     

  4. Index metacarpophalangeal hyperflexion

     

  5. Index metacarpophalangeal ulnar deviation

 

PREFERRED RESPONSE 1

 

EIP transfer results in no functional deficit. If the tendon is cut proximal to the sagittal band, there will be no extensor deficit.

 

(SAE11OS.106) A patient with a transverse femur fracture undergoes statically locked antegrade intramedullary nailing. Postoperatively, the patient appears to have a

rotational deformity of greater than 25 degrees. The surgeon informs the patient, who chooses to undergo corrective treatment with removal of distal interlocking screws, rotational correction, and relocking of the screws. The patient goes on to heal but has persistent hip pain and a limp that does not improve completely after extensive rehabilitation. There is complete healing, no evidence of infection, no hardware issues, no ectopic bone, and rotational studies indicate less than 2 degrees of malrotation. Functional capacity testing reveals the affected abductor and quadriceps function to be about 85% of the uninjured side and the patient returns to work and most of his recreational activities except rock climbing. Two days before the statute of limitations, the patient files a malpractice suit alleging negligence of surgery, loss of function, consortium, and pain and suffering due to the surgeon's efforts. What action should the surgeon and the defense team take? Review Topic

 

  1. Settle the case because the surgeon made an error that resulted in unnecessary surgery, and thus the case is indefensible.

     

  2. Settle the case because they are likely to lose the case, and it would be cheaper to settle than to defend.

     

  3. Defend the case alleging that there was no error, and no damages, and that the patient is malingering.

     

  4. Defend the case because despite there being an error, the error was corrected and there were little or no damages compared with expected outcomes.

  5. Contact the patient directly to discuss why he is suing and attempt an amicable resolution.

 

PREFERRED RESPONSE 4

 

To establish negligence, certain criteria must be met. 1) A duty was owed by the surgeon (in this case, yes, a relationship was established). 2) The duty was breached, where the provider failed to meet the standard of care (there was a technical error, but it was corrected). 3) The breach caused an injury. In this case, the patient had an outcome that was very acceptable, as documented with outcome studies, for femur fractures. Also, the rotational error and locking distally would have had little impact on the hip, whereas antegrade nailing itself is expected to result in some objective impairment of the hip in some patients. 4) Damages were incurred as a result. In this case, the patient returned to work and could not rock climb which could be reasonably expected with a femur fracture in some patients, and cannot be causally linked to the corrective surgery. For all practical purposes, the patient had a very acceptable outcome. Thus, settling the case for an error would be rather permissive and the important issue is that the surgeon recognized the problem, addressed it, and fulfilled his or her postoperative responsibility. The case is very defendable, and thus it is unlikely to be lost. Defending the case and alleging no error is incorrect because there was an error. The surgeon should never function outside of his or her legal counsel once a suit is filed.

 

(SAE13BS.91) When making a comparison to autograft incorporation, the inflammatory process in allograft tissue anterior cruciate ligament (ACL) reconstruction 

 

  1. occurs earlier.

     

  2. occurs later.

     

  3. is prolonged.

     

  4. is shortened.

 

PREFERRED RESPONSE 3

 

Compared to similar autograft, allograft tissue demonstrates a prolonged inflammatory response, slower rate of biological incorporation and remodeling, and a higher proportion of large-diameter collagen fibrils. Native ACL inserts into bone through a transition of 4 distinct zones: tendon, unmineralized fibrocartilage, mineralized fibrocartilage, and bone. This transition is not reproduced with tendon grafts, which instead heal with interposed fibrovascular scar at the graft-tunnel interface. The scar rapidly remodels to form perpendicular fibers resembling Sharpey fibers and, eventually, mature bone growth into the outer portion of the graft. The intra-articular portion of allograft undergoes an initial phase of necrosis followed by repopulation by host synovial cells into the acellular collagen scaffold. Revascularization and maturation complete the ligamentization of graft tissue.

 

(SAE10BS.13) As a diaphyseal fracture heals, peripheral callus forms about the shaft axis, creating a structure with a substantially larger diameter than the original diaphyseal shaft. What biomechanical properties does this callus impart to the healing fracture site? 

 

  1. Callus decreases torsional stability and stiffness at the fracture site

     

  2. Callus formation is random and unstructured and does not affect the local biomechanical properties

  3. The callus decreases peak torque to failure with time

     

  4. The callus increases the moment of inertia, resulting in less strain at the fracture site

     

  5. The callus decreases the moment of inertia, increasing stress at the fracture site

 

PREFERRED RESPONSE 4

 

Callus formation is biomechanically benefecial because it increases the outer diameter of the bone, leading to an increase in stiffness, torsional strength, moment of inertia, and decreases resultant interfragmentary strain at the fracture site.

 

The biomechanical role of the peripheral callus is to provide initial stability to the fracture and to act as a scaffold for gradual mineralization. Because the bending stiffness of a structure is proportional to the 4th power of the diameter, a peripherally located callus provides substantial stability to the fracture, despite the relatively low stiffness and strength of callus. For example, doubling the diameter of the callus increases the resistance to bending by a factor of 16. As mineralization progresses, the bending stiffness and strength of the healed fracture eventually may be substantially greater than that of the original, intact bone.

 

Augat et al. review the mechanical and biological aspects of fracture healing. They report that increased diameter of periosteal callus formation benefits healing by enlarging the cross-sectional area of area of the bridging tissue and reducing interfragmentary motion. Patients with osteoporosis are known to have decreased callus mineralization and biomechanical properties.

 

Incorrect Answers:

Answer 1: Callus will increase torsional stability and stiffness, not decrease Answer 2: While callus formation may be randomly arranged initially, it quickly becomes orderly as the fracture heals and remodels according to Wolff's law. As stated above, callus formation influences the local biomechanical properties. Answer 3: Callus formation increases peak torque to failure Answer 5: Callus will actually increase the moment of inertia and distribute stress at the fracture site.

 

(SAE11OS.109) You design a research study in which you ask patients who have a nonunion of the tibia to fill out a questionnaire in which they report on a variety of medical conditions and social/behavioral practices. You compare these findings to a

similar group who did not develop a nonunion in order to identify medical and/or social conditions that might be risk factors for the development of tibial nonunions. This would be an example of what type of study? 

 

  1. Case series

     

  2. Meta-analysis

     

  3. Case control study

     

  4. Retrospective cohort study

     

  5. Prospective cohort study

 

PREFERRED RESPONSE 3

 

A case control series starts with the occurrence of a specific disease or observation, and then compares data on those individuals to a similar group without the disease (control group) in order to identify potential risk factors for the development of the disorder. A case series is an observational study in which an investigator follows a series of patients who received a specific treatment, recording the results and outcomes of that treatment. A meta-analysis is the combination of several separate studies that look at similar hypotheses in an effort to create a larger patient population for analysis. A cohort study looks for the incidence of a specific outcome in two groups (cohorts) of patients who are similar with the exception of a particular research variable (risk factor).

 

(SAE10BS.43) Which of the following enzymes is used to resorb bone by mature osteoclasts? 

 

  1. Alkaline phosphatase

     

  2. Cathepsin K

     

  3. Tissue inhibitor of metalloproteinase (TIMP)

     

  4. Osteonectin

     

  5. Osteopontin

PREFERRED RESPONSE 2

 

Osteoclasts resorb bone. Osteoclasts are derived from monocytes and macrophages that fuse to form multinucleated cells in response to receptor activator of nuclear factor kappa b (RANKL) and macrophage colony stimulating factor (MCSF). Osteoclasts attach to bone surfaces through a specific cell attachment receptor called an integrin (av¯3 or vitronectin receptor). The ruffled border refers to multiple invaginations of plasma membrane against the bone matrix surface. Lysosomes move to the ruffled border of osteoclasts and discharge lysosomal enzymes into the resorption region. These enzymes include cathepsin K, matrix metalloproteinase, and carbonic anhydrase (CA II).

 

(OBQ14.155) What term in statistics defines rejecting the null hypothesis when it is in fact true? 

 

  1. Type-I error

     

  2. Type-II error

     

  3. Confounding error

     

  4. Variance

     

  5. Negative likelihood ratio

 

PREFERRED RESPONSE 1

 

Rejecting the null hypothesis when it is true is an example of a type-I error.

 

Type-I errors, to put it simply, detect an effect that is not present. In contrast, a type-II error fails to detect an effect that is present. In simple studies, the rate of a type-I error is denoted by a (alpha). For a 95% confidence level, the value of alpha is 0.05. This means that there is a 5% probability that we will reject a true null hypothesis.

Illustration A shows the difference between type-I and type-II errors. Incorrect Answers;

Answer 2: A type-II error accepts the null hypothesis when it should be rejected. Answer 3: A confounder is a variable that has associations with both the dependent and independent variables, potentially distorting their relationship. Confounders are not technically considered "errors," but instead are variables that properly constructed

studies attempt to avoid. Answer 4: Variance is an estimate of the variability of each individual data point from the mean.

Answer 5: Negative likelihood ratio describes how the likelihood of a disease is changed by a negative test result.

 

(OBQ14.160) As an orthopaedic surgery resident, you arrive late to a medial patellofemoral ligament reconstruction procedure in your institution's outpatient surgical center. It is standard practice in your residency program to miss the start of the first case because of didactic requirements on Tuesdays. The surgical team has your gown and gloves ready, and following scrubbing and gowning, you join the surgical team and assist the attending with the procedure.

 

Which of the following represents a violation of standard surgical safety checklists? 

 

  1. Joining an ongoing surgery without repeating a surgical timeout

     

  2. Arriving late to an elective surgery as a member of the surgical team

     

  3. Wearing the same set of scrubs under your surgical scrubs and gown that you wore attire during your didactic session

  4. Using an alcohol-based gel rather than a formal surgical scrub for the first case of the day

     

  5. Operating on a patient without personally marking the surgical site in the pre-operative holding area

 

PREFERRED RESPONSE 1

 

When a surgeon arrives late to the operating room, the timeout should be repeated to confirm the patient and procedure as well as introduce all members of the team and their respective roles.

 

The World Health Organization and nearly every professional surgical subspecialty group advocates for safety checklists prior to operations. Although there are institutional variations to individual items on the checklists, it is important to include a system for marking the surgical site, verifying the patient's identity and procedure planned, identifying all team members in the room and agreement regarding the type of anesthesia, antibiotic prophylaxis, instrumentation used. Simply put, safety checklists have been shown to drastically reduce costly errors in other high-risk

industries, and they have been universally adopted by hospitals and surgery centers and the organizations that inspect and validate healthcare settings.

 

The Joint Commission Guidelines and the WHO surgical safety checklists are examples of governing bodies of healthcare implementing safety checklists in procedural settings.

Illustration A is the WHO surgical safety checklist. Incorrect Answers:

Answer 2: It is acceptable to arrive late to a surgery, but recommended by the WHO to repeat a universal timeout Answers 3 and 4: While some hospitals have institutional guidelines that these answer choices may violate, these are not part of standard patient safety checklists Answer 5: The surgical site should always be marked ahead of time when applicable sites are operated on - however, each surgical team member does not have to personally do so prior to the onset of a procedure

 

(SAE13BS.9) A study is proposed in which 2 groups of patients are randomized to treatment with bisphosphonates or placebo. This is an example of what study type? 

 

  1. Crossover design trial

     

  2. Parallel design trial

     

  3. Cohort study

     

  4. Case series

 

PREFERRED RESPONSE 2

 

In a parallel design trial, participants are randomized to 2 or more groups, each of which receives a different treatment or intervention. For example, Group A receives the drug and Group B receives the placebo. This type of design allows for comparison between groups. In a crossover design clinical trial, both groups receive both interventions over a defined time period. For example, Groups A and B both receive the drug as well as the placebo. This allows for within-participant comparisons. In a cohort study, patient groups are followed over time on the basis of having or not having received an exposure. Cohort studies are not randomized. In a case series, patients often receive a particular treatment and the outcomes are then examined.

 

(SAE11UE.88) An otherwise healthy 30-year-old man undergoes right shoulder arthroscopic Bankart repair under regional anesthesia using an interscalene brachial plexus block. In the recovery room, he reports mild difficulty breathing and his chest radiograph shows a high riding diaphragm on the right side. His peripheral oxygenation is 97% on 2 liters of oxygen by nasal cannula. What is the most appropriate management? 

 

  1. Continued observation and monitoring

     

  2. Obtain arterial blood gas measurements

     

  3. Obtain emergent spiral CT scan to assess for pulmonary embolism

     

  4. Insertion of a chest tube

     

  5. Airway control and, if necessary, endotracheal intubation

 

PREFERRED RESPONSE 1

 

Because the phrenic nerve lies in close proximity to the site of anesthetic injection, temporary hemidiaphragmatic paresis is a very common side effect of interscalene brachial plexus block. Pulmonary function and chest wall mechanics may be slightly compromised, but can easily be compensated in a healthy patient. Therefore, with sufficient oxygenation, aggressive assessments or treatments such as arterial blood gas measurements, emergent spiral CT scans, chest tube insertions, or endotracheal intubation are not warranted. For this stable patient, continued monitoring with gradual withdrawal of oxygen is the most appropriate treatment.

 

(SAE08AN.15) Figure 9 shows the AP radiograph of a 65-year-old man who has knee pain and swelling. What is the most likely diagnosis? 

 

 

  1. Gout

  2. Chondrocalcinosis (pseudogout)

     

  3. Hemochromatosis

     

  4. Rheumatoid arthritis

     

  5. Ochronosis

 

PREFERRED RESPONSE 2

 

Although all the choices are known causes of joint degeneration (secondary osteoarthritis), only chondrocalcinosis shows distinct linear calcification of the cartilage due to deposition of calcium pyrophosphate crystals. Gout is a recurrent acute arthritis resulting from the deposition of monosodium urate from supersaturated hyperuricemic body fluids. Hemochromotosis is characterized by focal or generalized deposition of iron within body tissues. Arthritis may be present but is less common than other manifestations such as liver cirrhosis, skin pigmentation, diabetes mellitus, and cardiac disease. Rheumatoid arthritis is a nonspecific, usually symmetric inflammation of peripheral joints resulting in destruction of articular and periarticular structures. Ochronosis is a hereditary enzyme deficiency (homogentisic acid oxidase) resulting in deposition of homogentisic acid polymers in articular cartilage.

 

(SAE10BS.79) Which of the following zones of articular cartilage has the highest concentration of proteoglycans? 

 

  1. Superficial

  2. Transitional

     

  3. Deep

     

  4. Calcified

     

  5. Tidemark

 

PREFERRED RESPONSE 3

 

The fundamental structure of normal adult articular cartilage is divided into four different zones: superficial, transitional, deep, and calcified. These layers vary in chondrocyte morphology, size and orientation of collagen bundles, and water and proteoglycan content. The deep zone has the highest concentration of proteoglycans and the lowest concentration of water. The tidemark is a boundary between the calcified and uncalcified layers of articular cartilage.

 

(SAE12SN.75) Smoking has been associated with lower fusion rates in both cervical and lumbar fusion. Which of the following statements best describes an explanation for these findings? 

 

  1. Nicotine impairs osteoblast activity, thus interfering with bone remodeling.

     

  2. The effects of smoking on bone healing are multifactorial and not yet fully understood.

     

  3. The vasoconstrictive and platelet-activating properties of nicotine inhibit fracture healing.

     

  4. Nicotine inhibits the function of fibroblasts, red blood cells, and macrophages.

     

  5. Hydrogen cyanide inhibits oxidative metabolism at the cellular level.

 

PREFERRED RESPONSE 2

 

Tobacco smoking is now the leading avoidable cause of morbidity and mortality in the United States. The musculoskeletal effects of smoking have been implicated in osteoporosis, low back pain, degenerative disk disease, poor wound healing, and delayed fusion and fracture healing. A number of studies have demonstrated the relationship between smoking and development of pseudarthrosis. Numerous studies

have been performed to offer an explanation of the mechanism mediating this effect. Whereas all of the above have been postulated as explanations, more recent studies have demonstrated that nicotine delivered via a transdermal patch significantly enhanced posterior spinal fusion in rabbits. Thus it appears that the effects of smoking on fracture healing are multifactorial and not yet fully understood.

 

(SAE10SM.59) Figure 59 shows properties of a material being tested for use as an implant. What is represented by the portion of the stress-strain curve from point A to point B? 

 

 

 

  1. Elastic limit

  2. Nonproportional behavior

     

  3. Plastic behavior

     

  4. Elastic behavior

     

  5. Fracture point

 

PREFERRED RESPONSE 4

 

The figure is a stress-strain diagram representing specific metal subjected to increasing tensile stress. The portion of the curve from A to B is a straight line demonstrating a proportional increase in strain for each increase in tensile stress. If the stress is removed at any point between A and C, the material will return to its original shape, returning back along the original curve without permanent deformation. This is termed elastic behavior. If the applied stress causes strain beyond

point C, then permanent deformation occurs and returns along a different path to a different zero stress point. This is termed plastic behavior. The point C at which the material stops behaving in an elastic manner and begins behaving in a plastic manner is the elastic limit or yield point. Point D represents a point on the curve of plastic deformation. Point E is the fracture point when the stress on the material creates enough strain that the material fractures.

 

(SAE09TR.28) What is the most common cause of errors that harm patients? Review Topic

 

  1. Communication breakdown

     

  2. Equipment breakdown

     

  3. Nursing competence

     

  4. Patient noncompliance

     

  5. Physician competence

 

PREFERRED RESPONSE 1

 

The AMA report identified communication breakdown as the most common cause of errors that harm patients. It is extremely important to learn to communicate effectively with your patients. Understanding cultural and language differences helps avoid communication errors.

 

(SAE13BS.57) Etanercept modifies the natural history of inflammatory arthropathies through what mechanism? 

 

  1. Antagonism of the Interleukin-1 (IL-1) receptor

     

  2. Suppression of prostaglandin production through selective inhibition of cyclooxygenase (COX)-2

  3. Selective costimulation modulator inhibition of T lymphocyte activation

     

  4. Inhibitory binding to tumor necrosis factor alpha (TNF&# 945;)

 

PREFERRED RESPONSE 4

 

TNFα has been implicated in the pathogenesis of many chronic inflammatory diseases. Selective blockade with agents such as etanercept decreases the activation of mesenchymal cells, thereby reducing pannus formation, cartilage destruction, and osteoclastic bone resorption. IL-1 production in response to inflammatory stimulus contributes to the rapid loss of proteoglycans, leading to cartilage destruction and osteoclastic bone resorption. Recombinant forms of IL-1 antagonists such as the drug anakinra effectively block IL-1 by competitively binding to the IL-1 type I receptor. Nonsteroidal anti-inflammatory drugs inhibit the enzymes COX-1 and COX-2, which are necessary for the production of prostaglandins. Abatacept is a selective costimulation modulator that inhibits T lymphocyte activation implicated in pathogenesis of juvenile idiopathic arthritis. Methotrexate is an effective agent in the treatment of rheumatoid arthritis. The mechanism of action of this drug has not been fully elucidated. Proposed actions include decreasing cytokine production through promotion of adenosine release and inhibition of transmethylation reactions that otherwise result in accumulation of toxic compounds (spermine and spermidine).

 

(SAE12FA.96) A 31-year-old woman underwent a left Kidner procedure 3 months ago. She now has pain overlying the medial column of the foot. She withdraws the foot when touching of the medial foot is attempted. Examination reveals allodynia, pain, hyperalgesia, and edema of the medial foot. What is the most likely diagnosis? 

 

  1. Shingles

     

  2. Cellulitis

     

  3. Charcot foot

     

  4. Osteomyelitis

     

  5. Reflex sympathetic dystrophy

 

PREFERRED RESPONSE 5

Patients with reflex sympathetic dystrophy (RSD) have a history of trauma, minor rather than major (eg, Colles fracture), in about 50% to 65% of cases. The condition may also follow a surgical procedure. Patients usually have symptoms and signs of RSD including: pain, described as burning, throbbing, shooting, or aching; hyperalgesia; allodynia; and hyperpathia. There are trophic changes within 10 days of onset of RSD in 30% of the extremities affected, including stiffness and edema and atrophy of hair, nails, and/or skin. Finally there can be autonomic dysfunction, such as abnormal sweating, either in excess or anhydrosis, heat and cold insensitivity, or redness or bluish discoloration of the extremities. Shingles, also called herpes zoster or zoster, is a painful skin rash caused by the varicella zoster virus (VZV). VZV is the same virus that causes chickenpox. After a person recovers from chickenpox, the virus stays in the body. Usually the virus does not cause any problems; however, the virus can reappear years later, causing shingles. Charcot arthropathy is a progressive condition of the musculoskeletal system that is characterized by joint dislocations, pathologic fractures, and debilitating deformities. This disorder results in progressive destruction of bone and soft tissues at weight-bearing joints; in its most severe form, it may cause significant disruption of the bony architecture. In patients with diabetes, the incidence of acute Charcot arthropathy of the foot and ankle ranges from 0.15% to 2.5%. Acute Charcot arthropathy almost always appears with signs of inflammation. Profound unilateral swelling, an increase in local skin temperature (generally, an increase of 3° to 7° above the nonaffected foot's skin temperature), erythema, joint effusion, and bone resorption in an insensate foot are present. These characteristics, in the presence of intact skin and a loss of protective sensation, are often pathognomonic of acute Charcot arthropathy. Cellulitis is an infection of the skin. Examination would reveal erythema, edema, and pain. Osteomyelitis is an infection of the bone. Examination may reveal edema, drainage, and pain.

 

(SAE09TR.59) In the setting of a proximal tibial plateau fracture and its repair, which of the following materials is an isotropic material? 

 

  1. Proximal tibial cortical bone

     

  2. Articular cartilage of the tibial plateau

     

  3. Stainless steel tibial plate

     

  4. Medial or lateral meniscus

     

  5. Calcium phosphate cement

 

PREFERRED RESPONSE 3

An isotropic material is one that has similar mechanical properties regardless of the orientation of the material. Examples of isotropic materials include metals, plastics, and methacrylate. Most biologic tissues are anisotropic, meaning their mechanical properties alter depending on the materials’ orientation to the applied stress.

 

(SAE10BS.26) Porous hydroxyapatite is placed into a bone defect. Incorporation of this bone graft substitute is expected to follow which of the following patterns? 

 

  1. Neovascularization and appositional new bone growth

     

  2. Creeping substitution

     

  3. Inflammatory response with sequestration

     

  4. Dissolution of the graft material and osteoblastic bone formation

     

  5. Bridging bone formation without incorporation of the graft

 

PREFERRED RESPONSE 1

 

Porous hydroxyapatite is created via a hydrothermal chemical exchange with phosphate of the calcium carbonate exoskeleton of ocean corals. This process converts the original exoskeleton into an inorganic replica of hydroxyapatite. The porous structure allows neovascularization and new bone is deposited on the macrostructure via appositional bone deposition. The hydroxyapatite does not dissolve and is not removed via creeping substitution. Creeping substitution relies on osteoclastic resorption creating a cutting cone followed by osteoblastic bone formation. The macrostructure of porous hydroxyapatite allows full penetration of osteoblasts and vascularization, not just to the periphery. Inorganic hydroxyapatite does not induce an inflammatory response.

(SAE13BS.49) The ability of compressed cortical bone to resist greater applied force in the longitudinal plane than in the transverse plane is an illustration of what material property? 

 

  1. Yield strength

     

  2. Elastic modulus

     

  3. Viscoelasticity

     

  4. Anisotropy

 

PREFERRED RESPONSE 4

 

Material properties characterize mechanical functional limits of a material independent of the size or shape of that material. Anisotropic materials are those for which properties behave differently dependent on the direction of applied force. Yield strength is the load at which permanent plastic deformation begins to occur. Elastic modulus is the mathematical description of the tendency of a material to be deformed elastically in response to an applied force. The elastic modulus of a material is defined as the slope of its stress-strain curve in the elastic deformation region. Viscoelastic materials such as bone exhibit time-rate-dependent stress-strain behavior as a function of internal friction. The modulus of viscoelastic materials increase as the strain rate increases.

 

(SAE11OS.172) Randomized controlled trials can be designed in several ways. Which of the following study designs refers to a randomized controlled trial in which two interventions are compared within the same study group? 

 

  1. Parallel

     

  2. Case control

     

  3. Case series

     

  4. Factorial

     

  5. Crossover

 

PREFERRED RESPONSE 4

A factorial randomized control trial design is more easily represented in a two by two table. Practically, patients are randomized to either treatment A and B, treatment A or control, treatment B or control, or no treatment. The strength of this trial design is that two interventions can be assessed with the same study population. Also, any interaction between the treatments can be determined (for example, does treatment A work differentially when combined with treatment B). The parallel design trial is the simplest and most classic design for a randomized controlled trial. In this trial design, participants are randomized to two or more groups of different treatments and each group is exposed to a different intervention and only that intervention. In the crossover design trial, both groups receive both interventions over a randomly allocated time period. Group A can receive the treatment, and after a suitable washout period, can receive the placebo. Group B can receive the placebo and later can receive the treatment; this produces within-participant comparisons. The crossover trial design has a limited role in surgical interventions because it is difficult or impossible for patients to receive both treatment interventions, such as plate and nail fixation, or a cemented versus a cementless total hip arthroplasty. Case control and case series are not randomized trials, but observational studies.

 

(SAE10BS.33) In a hypothetical study, an investigator uses pedometers to determine that the 20 women in his activity study averaged 2.1 million steps per year, whereas the 20 men averaged 1.8 million. The p-value for this difference was 0.09. Which of the following is a correct interpretation of the outcome of this study? 

 

  1. Since p > 0.05, the activity levels of women and men are statistically equal.

     

  2. If, in the general population, there is no difference between the average activity levels of women and men, the difference observed between the women and men in this study would occur by chance selection in nine out of 100 times that the study was performed.

     

  3. The sample size was too small for this study, so the p-value has no meaning.

     

  4. Though a difference was measured between the two groups, the findings are not clinically significant, since p > 0.05.

  5. There is a 9% difference between the two groups because p = 0.09.

 

PREFERRED RESPONSE 2

The p-value should be interpreted only as an indication of the level of uncertainty of the results observed in this study. That is, the p-value answers the specific question, "If, in general, there actually is no difference between the average activity levels of women and men, how often would one expect to obtain by chance a difference as large (or larger) than was observed in the present study?" If the p-value is very small, it is relatively unlikely that the observed difference occurred by chance. However, it is critical to realize that, because of its definition, a large p-value is not an indication that there probably is no difference in general. Therefore, it is not true that the study has shown that there is "statistically no difference" between the activity levels of women and men. Rather, a large p-value indicates a relative lack of certainty of whether the difference between the activity levels of women and men in general is much smaller or much larger than was observed in the present study. Furthermore, no matter how large the p-value, in the absence of other data (other studies), the difference observed between two randomly selected groups of subjects is the most reliable estimate of the magnitude of the actual difference between the full populations. In a study such as this, if the p-value is sufficiently small, the investigators may be relatively confident in concluding that the observed difference holds in general. In contrast, if the p-value is very large (say, 0.8), then the investigators are relatively uncertain about any conclusion - they are not highly certain that there is no difference in general. Put simply, contrary to the common misconception, observed differences are not shown to be real or false depending on whether the p-value is less than or greater than 0.05, or any other arbitrary value.

 

(SAE10SM.95) What allograft has the highest antigenicity when used for ligament reconstruction about the knee? 

 

  1. Tibialis anterior used for anterior cruciate ligament (ACL) reconstruction

     

  2. Tibialis anterior used for posterolateral reconstruction

     

  3. Bone-patellar tendon-bone used for ACL reconstruction

     

  4. Semitendinosus used for posterior cruciate ligament reconstruction

     

  5. Semitendinosus used for medial collateral ligament reconstruction

 

PREFERRED RESPONSE 3

Although theoretically the intra-articular environment is slightly more immune privileged, the role of immunogenicity is related more to bone than soft tissue. Therefore, the bone-patellar tendon-bone used for ACL reconstruction would have the highest risk of immunogenicity if storage techniques and harvest techniques were similar. This also is true for bone plugs associated with meniscal allografts.

 

(OBQ15.168) You are staffing the prison clinic in a large public hospital when a 55-year-old African American male presents complaining of severe right hip pain. His pain has been ongoing for the past five years and limits his ambulation. He has never used medications for pain control or physical therapy. A radiograph is shown in figure

A. When formulating his treatment plan, it is important to: 

 

 

 

  1. Guarantee the success of total hip arthroplasty

  2. Recommend simultaneous bilateral total hip arthroplasty

     

  3. Understand the role of implicit bias as a determinant of health care delivery disparity

     

  4. Request the patient reveal the reason for his incarceration

     

  5. Suggest referral to a pain management clinic

 

PREFERRED RESPONSE 3

 

The patient is an African American male prisoner with symptomatic right hip osteoarthritis. When formulating a treatment plan, it is important to understand the role of physician implicit bias in delivery of care and in creating disparities in

healthcare delivery.

 

Physician bias, prejudice, discrimination, and clinical uncertainty are all factors that contribute to health care disparities in the United States. Implicit and explicit attitudes are cognitive traits that influence physician delivery of care, and sometimes these attitudes do not perfectly correspond. It is important for a physician to understand that his implicit attitudes about a patient may unintentionally influence care despite his explicit attitudes. Physicians should be aware of their implicit biases in order to provide more effective decision-making and quality of care.

 

Stone et al. write about the issue of culturally competent delivery of care and the avoidance of unconscious bias in medical decision making. They argue that because unconscious stereotypes and prejudices can trigger biased medical decisions against specific groups, leading to the creation of differential diagnoses, disparities in treatment, and causing minorities to feel uncomfortable with seeking or complying with treatment plans. The authors suggest the integration of cultural competency training into medical education in order to help understand the perspective of the minority group patient.

 

Sabin et. al. compared the implicit and explicit biases of physicians with respect to race, gender, and age. They found that medical doctors showed an implicit bias of preferentially caring for White Americans relative to Black Americans, independent of the doctors’ self-report (explicit biases). Doctors'implicit biases exceeded their explicit biases in all race groups studied, except for African American physicians, who did not show an implicit bias toward patients.

 

Figure A demonstrates an AP pelvis x-ray with severe arthrosis of the right hip. The left hip demonstrates moderate disease.

 

Incorrect Answers:

Answer 1: While total hip arthroplasty has a high rate of success for osteoarthritis, it is important to avoid making guarantees of success. Answer 2: The patient has symptomatic right hip osteoarthritis. Radiographically he has severe right hip osteoarthritis with mild to moderate disease in the left hip. Left hip arthroplasty is not indicated in an asymptomatic hip. Answer 4: Requesting that the patient reveal the reason for incarceration may reveal or contribute to the physician's implicit biases. Care should be taken during this conversation in order to uphold a high quality of care. Answer 5: The patient has known osteoarthritis, which has a surgical solution as well as several non-operative modalities that have not yet been attempted. Referral to pain management at this point in the discussion is not appropriate at this time.

(SAE08OS.37) You are interested in learning a new technique for minimally invasive total knee arthroplasty. The Keyhole Genuflex system seems appealing to you because the instrumentation comes with wireless controls. What is an acceptable arrangement to learn more about this system? 

 

  1. You and your spouse attend dinner at a local restaurant with the local representative to discuss the Keyhole Genuflex knee.

     

  2. Keyhole pays your tuition to attend a CME course sponsored by the American Association of Hip & Knee Surgeons where both the Genuflex and the competing Styph total knee are discussed and demonstrated.

     

  3. Keyhole will pay your expenses to attend a workshop at their company headquarters, to learn how to implant the Genuflex knee and to see how the implant is manufactured and tested.

     

  4. Keyhole will pay you $500 for each knee that you implant if you switch from your current total knee system.

     

  5. After you have implanted 25 Genuflex knees, Keyhole will list you on their website as a consultant, pay you a consulting fee of $5,000 per year, and invite you to a golf tournament for their consultants at a resort.

 

PREFERRED RESPONSE 3

 

Both the American Academy of Orthopaedic Surgeons (AAOS) and AdvaMed, the medical device manufacturer's trade organization, have written guidelines that address potential conflicts of interest regarding interactions between physicians and manufacturer's representatives when it comes to patients' best interest. The AAOS feels that the orthopaedic profession exists for the primary purpose of caring for the patient and that the physician-patient relationship is the central focus of all ethical concerns. When an orthopaedic surgeon receives anything of significant value from industry, a potential conflict of interest exists. The AAOS believes that it is acceptable for industry to provide financial and other support to orthopaedic surgeons if such support has significant educational value and has the purpose of improving patient care. All dealings between orthopaedic surgeons and industry should benefit the patient and be able to withstand public scrutiny. A gift of any kind from industry should in no way influence the orthopaedic surgeon in determining the most appropriate treatment for his or her patient. Orthopaedic surgeons should not accept gifts or other financial support with conditions attached. Subsidies by industry to underwrite the costs of educational events where CME credits are provided can contribute to the improvement of patient care and are acceptable. A corporate subsidy received by the conference's sponsor is acceptable; however, direct industry reimbursement for an orthopaedic surgeon to attend a CME educational event is not appropriate. Special circumstances may arise in which orthopaedic surgeons may be required to learn new surgical techniques demonstrated by an expert or to review new implants or other devices on-site. In these circumstances, reimbursement for expenses may be appropriate.

 

(SAE08OS.192) Which of the following alternatives to autogenous bone grafting functions through osteoinduction? 

 

  1. Allograft

     

  2. Tricalcium phosphate

     

  3. Calcium phosphate

     

  4. Calcium sulfate

     

  5. Bone morphogenetic protein

 

PREFERRED RESPONSE 5

 

Allograft bone is one of the most common bone graft substitutes, and is frequently used as a bone graft extender. The sterilization process kills all cells but their three-dimensional structure is retained, offering an osteoconductive scaffold. Tricalcium phosphate, calcium phosphate, and calcium sulfate are also osteoconductive materials with a three-dimensional scaffold similar to native cancellous bone. Bone morphogenetic protein is an inductive protein, which stimulates osteoprogenitor cells to differentiate along a bone-forming lineage.

 

(SAE08OS.122) A 23-year-old man is injured in a motorcycle accident and has a Glasgow Coma Scale (GCS) score of 10. His fiance arrives shortly after he does. He has an open, IIIc tibial fracture. The patient's parents are on the way but are not expected to arrive for some time. Who should be asked to provide informed consent? 

 

  1. The patient

     

  2. The fiance

  3. The surgeon performing the procedure along with another physician with similar experience and knowledge

     

  4. The surgeon performing the procedure and a nurse taking care of the patient

     

  5. Wait for the patient's parents to arrive and have them provide informed consent

 

PREFERRED RESPONSE 3

 

With a GCS of 10, the patient is not capable of providing consent nor does the fiancTe have legal standing to do so. This is an emergency and waiting for the parents is not acceptable. Two surgeons of similar knowledge and experience may confirm the necessity of the procedure.

 

(SAE08OS.25) Which of the following is considered a limitation of the Short Form

36 (SF-36) general health status instrument when applied to musculoskeletal conditions? 

 

  1. Potential for "ceiling" and "floor" effects

     

  2. Normative population data have not been attained

     

  3. Patient's perspective is marginalized or ignored

     

  4. Must be performed on-site by investigator

     

  5. Better upper than lower extremity application

 

PREFERRED RESPONSE 1

 

Health outcome surveys are often either general or condition specific. The SF-36 (a general health-based survey) is the most widely applied general health status instrument. It measures three aspects of health: functional ability, well being, and overall health. Eight domains of quality of life are measured to quantify these aspects. It is designed to be self-administered by the patient. Because it is a patient-derived assessment, with patient-derived outcome measures, the patient's perspective is integrated and objectivity enhanced. The inherent bias of surgeon-driven formats is thus avoided. The SF-36 has been validated and normative population data obtained.

Multilingual validated translations have been produced in addition to international population data sets. The survey can be patient self-administered via office visit, mail, or telephone. A bias of lower over upper extremity function with regard to outcome measures employing the SF-36 has been demonstrated. Limits on the detection of certain changes in quality of life status may impose ceiling and floor effects on analysis and interpretation.

 

(OBQ14.87) The Arg-Gly-Asp (RGD) sequence of extracellular bone proteins directly allows which of the following? 

 

  1. Interaction with integrins

     

  2. Chemoattraction of osteoclasts

     

  3. Binding to vinculin protein

     

  4. Attachment site of Sharpey's fibers

     

  5. Decrease in bone stiffness

 

PREFERRED RESPONSE 1

 

The Arg-Gly-Asp (RGD) sequence of extracellular bone proteins such as fibronectin and vibronectin allow binding of integrins on the surface of osteoclasts to enable bone resorption.

 

Bone homeostasis involves molecular regulation that involves osteoblasts, osteoclasts, and bone. Osteoblasts upregulate and down regulate osteoclasts that absorb bone at the ruffled borders. Integrins, on the surface of osteoclasts bind to extracellular proteins on bone, such as vibronectin, to facilitate bone resorption at the ruffled border. The Arg-Gly-Asp (RGD) sequence plays an important roll in the binding of integrins and extraceullar proteins such as vibronectin and fibronectin. After binding, the intracellular side of these proteins binds to intracellular proteins to allow for change in the intracellular structures and function.

 

Bosseti et al. review the extracellular matrix interactions involved with bone induction and conduction mechanisms. They note that extracellular binding leads to intracellular protein alteration, which causes different intracellular effects depending on the specific ligand that binds.

 

Illustration A shows the binding of integrin and vibronectin.

Incorrect Answers:

Answers 2-5: The Arg-Gly-Asp (RGD) sequence does not involve these options.

 

(SBQ12SP.45) Which of the following lists these materials in order of increasing modulus of elasticity?: 

 

  1. Cortical bone; Titanium; Cobalt-chrome; Stainless steel; Ceramic

     

  2. Titanium; Cortical bone; Ceramic; Cobalt-chrome; Stainless steel

     

  3. Cortical bone; Titanium; Stainless steel; Cobalt-chrome; Ceramic

     

  4. Stainless steel; Titanium; Cortical bone; Ceramic; Cobalt Chrome

     

  5. Cortical bone; Stainless steel; Titanium; Cobalt-chrome; Ceramic

 

PREFERRED RESPONSE 3

 

Cortical bone has the lowest modulus of elasticity of the materials listed, followed by titanium, stainless steel, cobalt-chrome alloy, then ceramic.

 

Young's modulus of elasticity is the ratio of stress to strain, and represents the stiffness of a material and its ability to resist deformation when placed under tension. Of the materials listed, titanium has the stiffness closest to cortical bone. Ceramic has the highest modulus of elasticity, making it the most stiff of the materials listed.

 

Illustration A (from Miller's Review) shows the relative stiffnesses of various orthopaedically relevant materials. Young's modulus is the slope of the lines shown. Illustration B (Google images) charts their Young's Modulus.

 

Incorrect answers:

Answers 1, 2, 4, 5: Cortical bone is the least stiff of the materials listed. Ceramic is the stiffest. The intermediate materials listed are in the relative order listed in answer 3.

(SAE08UE.76) Outcome measures should have established psychometric properties of reliability, validity, and responsiveness. Reliability refers to which of the following? 

 

  1. The amount of change in the score over time

     

  2. Sensitivity of the measure in evaluating a problem

     

  3. The ability of the instruments to actually measure what it intends to measure

     

  4. The measure of change over the course of treatment

     

  5. The reproducibility of the measurements either between repeated tests or between observers

 

PREFERRED RESPONSE 5

 

The recent JBJS article by Kocher and associates defines the different psychometric properties that are used in outcome measures. Reliability is a measure of how reproducible a test is. This can be interobserver reliability (ie, reliability between people), or intraobserver reliability (ie, reliability for the same person doing the outcome measure at different occasions).

 

(OBQ14.22) Which of the following statements is true regarding articular cartilage? 

 

  1. The hydrophilicity of type II collagen molecules gives cartilage its viscoelastic properties

     

  2. Water accounts for approximately 25% of the mass of cartilage

     

  3. Multiple aggrecan molecules link together to form glycosaminoglycans

     

  4. The positive charge formed by aggrecan molecules helps to create the high osmotic swelling pressure of cartilage

     

  5. Type II collagen helps prevent swelling of articular cartilage which would otherwise occur due to high osmotic pressures

PREFERRED RESPONSE 5

 

The one role of collagen in articular cartilage is to provide the structural framework to resist swelling under high osmotic tissue pressures created by aggrecan. Type II collagen is the predominant type in articular cartilage.

 

Proteoglycans, the most common of which is aggrecan, are produced by chondrocytes and give articular cartilage its hydrophilic properties. Multiple glycosaminoglycans (GAGs), such as chondroitin and keratin can attach to core proteins to form aggrecans. Link proteins then help aggrecans interact with hyaluronic acid. The negative charge of this complex helps create a strong osmotic gradient, which attracts water and increases tissue pressures. Normal aging involves a decrease in the water content of the extracellular matrix while osteoarthritis is associated with increased water content, which leads to loss of strength and elasticity.

 

Chen et al. evaluated the strain and depth related properties of articular cartilage in bovine models. They found that the zero-strain permeability, zero-strain equilibrium confined compression modulus, and deformation dependence constant differed among the layers of cartilage. They suggest that the complex strain-dependent properties of articular cartilage of different thickness and location have clinical implications for tissue engineering.

Illustration A is a diagram depicting the extracellular matrix of articular cartilage. Incorrect Answers:

Answer 1: The hydrophilicity of aggrecan molecules contributes to the viscoelastic properties of cartilage. Answer 2: Water accounts for 65%-80% of cartilage mass Answer 3: Multiple GAGs link to form aggrecan molecules Answer 4: Aggrecan molecules have a negative charge that contributes to the high osmotic gradient

 

(OBQ15.170) Which of the following genetic disorders has an X-linked recessive inheritance pattern? 

 

  1. Gaucher disease

     

  2. Prader-Willi Syndrome

     

  3. Diastrophic Dysplasia

     

  4. Hemophilia A

  5. Hypophosphatemic rickets

 

PREFERRED RESPONSE 4

 

Hemophilia A is inherited in an X-linked recessive fashion.

 

  1. linked recessive disorders occur when a mutation occurs on the X chromosome that causes the phenotype to be expressed in males and in females who are homozygous for the gene mutation. These disorders are much more common in males since they only have one X chromosome. Other X-linked recessive disorders include: Duchenne muscular dystrophy, Becker's muscular dystrophy, Hunter's syndrome, and spondyloepiphyseal dysplasia (SED) tarda.

     

    Vanderhave et al. reviewed the orthopaedic consideration in patients with hemophilia. Amongst other things, they discuss arthroplasty in patients with this condition. While they are at higher risk for stiffness and acute hemarthrosis following total knee arthroplasty, ~90-95% of patients have good or excellent results.

     

    Illustration A shows how hemophilia is inherited through a punnett square. Illustration B shows how a weak clotting mechanism causes increased bleeding in patients with hemophilia.

     

    Incorrect Answers:

    Answer 1: Gaucher disease is an autosomal recessive condition. Answer 2: Prader-Willi Syndrome is inherited through imprinting. Imprinting is a genetic phenomenon by which certain genes are expressed in a parent-of-origin-specific manner.

    Answer 3: Diastrophic dysplasia is an autosomal recessive condition. Answer 5: Hypophosphatemic rickets is a sex-linked dominant condition.

     

    (OBQ14.5) A 58-year-old patient suddenly develops cardiac arrest while undergoing a routine total knee arthoplasty. He is resuscitated with 20% lipid emulsion. What was the most likely causative agent for cardiac arrest? 

     

    1. Epinephrine

       

    2. Bisoprolol

       

    3. Bupivacaine

       

    4. Fentanyl

    5. Rocuronium

 

PREFERRED RESPONSE 3

 

This patients cardiac arrest was likely caused by an intravascular bolus of Bupivacaine.

 

Bupivacaine is a long acting local anesthetic. It acts on intracellular voltage-gated sodium channels to block sodium influx into nerve cells, which prevents depolarization. If administered systemically, bupivacaine can cause serious complications to the cardiovascular system. Side effects include hypotension, arrhythmia, bradycardia, heart block, and cardiac arrest. Treatment should involve urgent administration of intravascular lipid emulsion alongside standard ACLS protocols.

 

Rosenblatt et al. describe the successful use of a 20% lipid emulsion to resuscitate a patient after a presumed bupivacaine-related cardiac arrest. Resuscitation involved a lipid emulsion bolus of 1 mL/kg given immediately, which was followed by a continuous infusion until the patient stabilized.

 

Corman et al. evaluated the use of lipid emulsion for reversal of local anesthetic-induced toxicity. They suggest that lipid emulsion may reverse local anesthetic toxicity by extracting lipophilic local anesthetics from aqueous plasma or tissues or by counteracting local anesthetic inhibition of myocardial fatty acid oxygenation.

 

Illustration A shows an ECG of a patient with clinical deterioration after systemic administration of bupivicaine. The ECG shows complete heart block with multifocal ventricular beats. The patient progressed to asystole.

 

Inncorrect Answers:

Answer 1: Intravascular epinephrine would cause hypertension and tachycardia. There is no specific antidote for epinephrine intoxication and treatment is primarily supportive.

Answer 2: Intravascular overdose of bisoprolol would cause hypotension, bradycardia and possible cardiac arrest. Treatment includes treating bradycardia with atropine, isoproterenol or cardiac pacing. Hypotension should be treated with IV fluids. Continuously monitoring is required. Answer 4: High doses of fentanyl usually have little significance intra-operatively under the care of the anesthesiologist. The most significant is respiratory depression. Pure opioid antagonists, such as naloxone, are specific antidotes if required. Answer 5: High doses of Rocuronium will cause respiratory depression, tachycardia and hypertension. Sugammadex (tradename Bridion) is an agent for reversal of neuromuscular blockade by the agent rocuronium in general anaesthesia.

(SAE10BS.51) Arthritic change in cartilage is characterized by which of the following findings? 

 

  1. Proteoglycan loss in the cartilage matrix

     

  2. Increased compressive modulus compared to normal cartilage

     

  3. Increased tensile modulus

     

  4. Decreased water content

     

  5. Increase in the shear modulus

 

PREFERRED RESPONSE 1

 

Experimental models of late-stage arthritis in animals demonstrated lower compressive modulus, higher permeability, and higher water content. There is proteoglycan loss within the matrix. A significant and progressive decrease in the tensile and shear modulus has been observed.

 

(SAE10BS.63) An osteoconductive bone graft material has which of the following properties? 

 

  1. It contains osteoblastic cells capable of direct bone formation.

     

  2. It has the ability to stimulate recruitment and differentiation of mesenchymal stem cells.

     

  3. It has the ability to recruit progenitor cells that can differentiate into osteoblastic cells.

     

  4. It has the ability to permit ingrowth and sprouting of capillaries and perivascular tissue.

     

  5. It prevents creeping substitution to allow graft incorporation.

 

PREFERRED RESPONSE 4

 

Osteoconductive bone graft material is described as the scaffold that allows new bone growth. This new bone is created via sprouting capillaries, perivascular tissue, and osteoprogenitor cells on the three-dimensional graft structure, permitting creeping

substitution, which allows for graft incorporation. An osteoinductive graft will stimulate the recruitment of progenitor cells that can differentiate into osteoblastic cells. Specific BMPs are osteoinductive. Osteogenic material contains viable cells with the ability to form bone. The ability to provide osteoprogenitor cells is only seen in fresh autograft. Other grafts rely on the recruitment of host progenitor cells to differentiate.

 

(OBQ04.180) During fracture healing, granulation tissue tolerates the greatest strain before failure so that mature bone can eventually bridge the fracture gap during healing. What is the definition of strain? 

 

  1. Amount of force an object can withstand until plastic deformation is lost

     

  2. Change in length / original length of an object due to an external force

     

  3. Relationship of stiffness to time-dependent loading

     

  4. Force intensity / volume

     

  5. Force intensity / cross sectional area

 

PREFERRED RESPONSE 2

 

Strain is defined as the change in length/original length (L) and is created by a deformation of a material from an applied force.

 

The mechanical environment at the fracture site has a major influence on fracture healing. Granulation tissue can withstand higher strain, which stabilizes the mechanical environment and forms a scaffold on which cartilage and bone eventually form; this occurs after strain decreases incrementally. Optimal healing, however, depends on duration, rate, timing and type of mechanical influence. Bone is formed by osteoblasts that are adapted to the very low strains of over 1% change in length. Osteoblast synthesis and proliferation is stimulated at uniaxial strain of between 0.3% and 2.8%. It is known that limited inter-fragmentary movement of 0.2 mm to 1 mm is optimal for fracture healing, resulting in promotion of callus and increase in rigidity. Excessive movement, on the other hand, prolongs fracture healing. Researchers have identified that tissue strain of 2% is suitable for primary bone healing and secondary bone healing takes place at tissue strain of 2-10%. Strain of 10-100% results in fibrous tissue formation and 100% strain to non-union. This is known as Perren's theory.

Stokes published a review article on the effects of stress on bone healing and growth, and notes the importance of the 'Hueter-Volkmann Law' (growth is retarded by increased mechanical compression, and accelerated by reduced loading in comparison with normal values) in bone growth. Stokes also notes that sustained compression of physiological magnitude inhibits growth by 40% or more, while distraction increases growth rate by a much smaller amount.

 

Illustration A shows an example of a stress-strain curve, with several key definitions labeled on the diagram.

 

Incorrect Answers:

Answer 1: This is the elastic limit or yield point. Answer 3: The time-dependent deformations caused by creep and shrinkage have an Answer effect on the stiffness or modulus of an object. Answer 4: This is the definition of force density. Answer 5: This is the definition of stress.

 

(OBQ06.144) All of the following are characteristic of synovium affected by rheumatoid arthritis (RA) EXCEPT: 

 

  1. Prominent intimal hyperplasia

     

  2. Decreased apoptosis

     

  3. Increased angiogenesis

     

  4. Disruption of the basement membrane

     

  5. Abundant lymphocytes

 

PREFERRED RESPONSE 4

 

The basement membrane is not disrupted in rheumatoid arthritis-affected synovial tissue, as synovium lacks a true basement membrane.

 

Normal synovium consists of two layers, the intimal and the sublining, and two types of cells, type A and B. In RA, the following changes to the synovium are seen: 1) hyperplasia, with the intimal lining increasing from two cell layers to 10-20 layers 2) decreased apoptosis of the lining 3) increased angiogenesis and 4) abundant lymphocytes around vessels, forming lymphoid follicles.

The synovial pannus is invasive granulation tissue that contains fibroblast-like synoviocytes, but few inflammatory cells. The synovial cells in the pannus have anchorage-independent growth and invasive capabilities, allowing them to directly attack and destroy articular cartilage.

 

Koch et al. note that the possible contributions of angiogenesis to the proliferation of the inflammatory synovial pannus and the ingress of inflammatory leukocytes into the synovial tissue in RA have been extensively studied. Relevant angiogenesis inducers seen in RA include FGF-2, VEGF, TGFß, TNFa, IL-1, IL-8, VCAM-1, among others. Modulation of angiogenesis may be a viable therapeutic option for RA in the future.

 

Incorrect Answers:

Answer 1, 2, 3, 5: Synovium affected by rheumatoid arthritis has all of these characteristics.

 

(SAE08AN.60) Figure 35 shows the radiograph of a 44-year-old woman with rheumatoid arthritis who reports neck pain. Below what threshold number is surgical stabilization warranted for the interval shown by the arrow? 

 

 

 

  1. 8 mm

  2. 10 mm

     

  3. 12 mm

     

  4. 14 mm

     

  5. 16 mm

PREFERRED RESPONSE 4

 

The posterior atlanto-dens interval represents the space available for the spinal cord and a distance of less than 14 mm is predictive of neurologic progression, thus warranting consideration for fusion, even in the absence of symptoms.

 

(OBQ14.67) The Chi-square test is most appropriate for which of the following? 

 

  1. Comparing means of parametric numeric data between two groups

     

  2. Comparing means of non-parametric numeric data between two groups

     

  3. Comparing means of parametric numeric data between 3 or more groups

     

  4. Comparing categorical data between groups

     

  5. Comparing categorical data between groups with less than 5 samples or data points per group

 

PREFERRED RESPONSE 4

 

The Chi-square test is most appropriately used for comparing proportions of categorical or ordinal data.

 

When considering which statistical test to use the investigator must consider how many groups are being compared, and if the data collected is numerical or categorical, and parametric or non-parametric. When the study is comparing proportions of categorical data the appropriate test will be either a Chi-square test, or a Fisher exact test. The Fisher exact test is preferred when there is less than 5 data points in any group being compared.

 

Illustration A is a flow chart demonstrating how to determine which statistical test is most appropriate.

 

Incorrect answers:

Answer 1: Comparing means of parametric data between 2 groups is done with a t-test or paired t-test Answer 2: Comparing means of non-parametric data between 2 groups is best done

with the Mann-Whitney U test Answer 3: Comparing means of parametric data between 3 or more groups is best done with Analysis of variance (ANOVA) Answer 5: Comparing proportions of categorical data between groups when at least one group has less than 5 samples or data points is best done with the Fisher exact test

 

(OBQ15.64) A researcher experimenting with limb patterning removes some tissue from 1 part of the limb bud (which we shall call Site A) and transplants it along the anteroposterior (AP) axis to create a mirror-hand duplication. Which of the following is true? 

 

  1. Site A is the apical ectodermal ridge (AER). Site A tissue expresses Shh protein.

     

  2. Site A is the AER. Site A tissue expresses FGF8.

     

  3. Site A is the zone of polarizing activity (ZPA). Site A tissue expresses Shh protein.

     

  4. Site A is the zone of polarizing activity (ZPA). Site A tissue expresses FGF8 protein.

     

  5. Site A is non-AER ectoderm. Site A tissue expresses WNT7a.

 

PREFERRED RESPONSE 3

 

The ZPA is located on the posterior (ulnar) margin of the limb bud. It expresses Shh protein. When tissue from ZPA is added to the anterior (radial) margin of the limb bud, ulnar dimelia, or mirror hand duplication, occurs.

 

The ZPA controls AP (radioulnar) growth. The signaling molecule is Shh, which is dose dependent. Higher Shh doses lead to posterior (ulnar) digits ulnar sided polydactyly. The extent of duplication is dose dependent (higher dose = more replication). Reduced Shh leads to loss of digits. Posterior elements (little finger/ulna) are formed EARLY prior to anterior elements which are formed LATE (radius/thumb). Disruption of AP patterning will result in loss of later forming elements (radius/thumb).

 

Al-Qattan et al. reviewed embryology of the upper limb. They summarized that embryology of the upper limb can be viewed in 2 distinct ways: the steps of limb development and the way that the limb is patterned along its 3 spatial axes. Cell signaling plays a major role in regulating growth and patterning of the vertebrate limbs. Signaling cell dysfunction results in congenital differences according to the affected signaling axis.

Illustration A shows an experiment to create ulnar dimelia by adding ZPA tissue to the anterior limb bud. The video shows development of the limb.

 

Incorrect Answers:

Answers 1 and 2: Site A is the ZPA. The AER controls proximal to distal patterning. Answer 4: The ZPA expresses Shh. The AER expresses FGF8. Answer 5: Non-AER ectoderm controls dorsoventral patterning. Dorsal tissue express WNT7a and ventral tissue express en-1 protein.

 

(SAE12SN.88) A 78-year-old woman has a history of chronic low back pain. She denies any extremity problems. Her pain is worse in the morning, and gets better, although it does not go away, as the day goes on. An MRI scan of the lumbar spine is shown in Figure 88. She denies any acute worsening of her symptoms, although in general, her symptoms are slowly worsening. She takes nonsteroidal anti-inflammatory drugs as needed for her pain, but otherwise takes no other medications. What is the next most appropriate step in management? Review Topic

 

 

 

  1. DEXA scan

  2. Brace treatment with a Jewett hyperextension brace

     

  3. Anterior lumbar corpectomy and arthrodesis with instrumentation

     

  4. Posterior lumbar decompression and fusion

     

  5. Vertebral cement augmentation

PREFERRED RESPONSE 1

 

The patient has MRI findings throughout her lumbar spine consistent with old compression fractures. Given the imaging findings and advanced age, she is at high risk for osteoporosis and subsequent fragility fractures. Management should consist of a DEXA scan to evaluate her degree of osteoporosis and begin medical treatment as appropriate. Because acute fracture is unlikely, and she has no neurologic compromise, neither bracing nor surgical treatment is indicated.

 

(OBQ13.77) Communication breakdown is the leading cause of which of the following? 

 

  1. Delayed diagnoses

     

  2. Medication errors

     

  3. Surgical site infections

     

  4. 1 and 2

     

  5. All of the above

 

PREFERRED RESPONSE 4

 

Communication failures are the leading cause of wrong side surgeries, medication errors and diagnostic delays.

 

Poor communication sets up environments in which medical errors can take place. Per the Joint Commission, medical errors may be the among the top 10 causes of death in the United States. Establishing open lines of communication is critical to reduce the risk of error and enhance patient safety.

 

Gandhi et al. designed a framework to study missed or delayed diagnoses and their causes. The most significant factors contributing to errors were poor handoffs, failures in judgment, failures in memory and failures in knowledge.

 

O’Daniel et al. review the importance of professional communication and collaborative team efforts. They note that patient safety is at risk when poor communication is in place. The leading cause for medication errors, treatment delays and wrong-site surgeries is communication failure.

Illustration A shows the leading causes of death in the United States. This includes “preventable errors” as a cause.

 

Incorrect Answers:

Answers 1, 2: Communication failures can lead to delays in diagnosis and treatment, medication errors and wrong side surgery Answers 3, 5: Communication failure is not a direct contributor to surgical site infection

 

(OBQ13.116) A prospective randomized trial is conducted to test the efficacy of Vitamin C versus placebo in treating patients who develop chronic regional pain syndrome (CRPS) after distal radius fractures. At first follow-up, the rates of CRPS are 1% and 9% in the study and placebo group, respectively. Which statistical test is most appropriate to determine significance? 

 

  1. Single factor analysis of variance

     

  2. Chi-square test

     

  3. Student t-test

     

  4. Mann-Whitney rank sum test

     

  5. Wilcoxon rank sum test

 

PREFERRED RESPONSE 2

 

In the study provided, we need to determine whether distributions of categorical variables differ from one another. The appropriate study is the chi-square test.

 

Data can be classified as numerical (continuous) or categorical (proportional). Responses to such questions as "What is your major?" or Do you own a car?" are categorical because they yield data such as "biology" or "no." In contrast, responses to such questions as "How tall are you?" or "What is your G.P.A.?" are numerical. When comparing two independent means from numeric data, a t-test is performed. However, if categorical data is being compared, the chi-square test will determine if the proportions are really different.

 

Kocher et al. review basic clinical epidemiology and biostatistics relevant to orthopaedic surgery. Amongst other things, they describe that data can be summarized in terms of measures of central tendency, such as mean, median, and mode, and in

terms of measures of dispersion, such as range, standard deviation, and percentiles. Illustration A shows an algorithm for determining which test to use for varying data.

Incorrect Answers:

Answer 1: Analysis of variance (ANOVA) is used to compare means of three or more independent groups in which the data are normally distributed. Answer 3: Student t-test is used for comparing means of continuous data that is normally distributed.

Answer 4: The Mann-Whitney and Wilcoxon rank sum tests are used for comparing means of non-continuous data. Answer 5: The Mann-Whitney and Wilcoxon rank sum tests are used for comparing means of non-continuous data.

 

(OBQ13.250) A 38-year-old female with a grade IIIB open tibia fracture is scheduled to undergo definitive fixation and subsequent flap coverage with the orthopaedic and plastic surgery teams. She is met in the pre-operative area by the surgical intern and paperwork is completed per institutional protocol. She is then brought back to the operating room. Which of the following is true regarding the pre-surgical timeout? 

 

  1. The surgical intern must be present because he brought the patient to the operating room

     

  2. The timeout cannot begin without the implant representative

     

  3. If both the orthopaedic and plastic surgical teams are present, a single timeout is sufficient for the entire procedure

     

  4. The timeout may be completed as long as the attending is in an adjacent operating room

     

  5. The pre-surgical timeout has not been shown to decrease complication rates

 

PREFERRED RESPONSE 3

 

If both teams are present, a single timeout is sufficient for the entire procedure. If one team is absent, a second timeout needs to be completed prior to start of the second part of the procedure.

 

The WHO pre-surgical safety checklist involves assessment at three points during an operative procedure: before induction of anesthesia, before skin incision, and before the patient leaves the operating room. Some components of the checklist include

confirming IV access and allergies, administration of antibiotic prophylaxis, and surgical site verification. The surgeon has been shown to be the most effective team member at reducing complications when using the checklist.

 

Haynes et al. evaluated the effects of the WHO pre-surgical checklist on perioperative complication rates at eight international sites. They found a significant reduction in rates of complications and death in patients over the age of 16 undergoing noncardiac procedures after implementation of the checklist.

Illustration A shows the WHO pre-surgical checklist Incorrect Answers:

Answer 1: The surgical intern will not be involved in the case and does not need to

present for the timeout. Answer 2: The implant representative is not necessary for the timeout. Answer 4: The attending surgeon needs to be present in the same OR for the timeout. Answer 5: The timeout has been shown to decrease rates of complication and death.

 

(SAE08UE.110) The radiograph shown in Figure 54 reveals that the plate on the second metacarpal is acting in what manner? 

 

 

 

  1. Compression plate

  2. Tension band plate

     

  3. Bridge plate

     

  4. Buttress plate

     

  5. Spring plate

PREFERRED RESPONSE 3

 

There are four ways in which a plate acts: compression, tension bend, bridge or spanning, and buttress. Since there is no cortical contact with the large span of comminution, this plate is acting as a bridge plate. A bridge plate is defined as when the plate is used as an extramedullary splint attached to the two main fragments, leaving the comminution untouched.

 

(SAE13BS.100) A man who weighs 75 kg (165 pounds) is scheduled for elective total hip arthroplasty. He reports a history consistent with anaphylaxis from penicillin. Within 1 hour of the incision the patient should receive 1000 mg of 

 

  1. cefazolin.

     

  2. cefuroxime.

     

  3. vancomycin.

     

  4. clindamycin.

 

PREFERRED RESPONSE 3

 

The recommended antimicrobial prophylaxis for total hip arthroplasty is cefazolin or cefuroxime unless the patient has an allergy to beta-lactam antibiotics. Patients with an allergy should be given vancomycin 10 to 15 mg/kg or clindamycin 600 to 900 mg. Because this patient has an allergy to penicillin, he should be given 1000 mg of vancomycin within 1 hour of the skin incision.

 

(SAE11OS.147) Figure 147 is an MRI scan of a 72-year-old woman admitted to the hospital 7 days ago with persistent and worsening back pain. A repeat vertebral augmentation was performed at L2 three days ago. Today she became diaphoretic,

reported severe dyspnea, and collapsed during physical therapy. Examination reveals a pulse of 128/min, blood pressure of 98/55 mm Hg, and temperature of 100 degrees F (37.7 degrees C). Jugular venous distention is noted. What is the most likely complication? 

 

 

 

  1. Spinal shock

  2. Neurogenic shock

     

  3. Hemorrhagic shock

     

  4. Pulmonary embolism

     

  5. Autonomic dysreflexia

 

PREFERRED RESPONSE 4

 

The patient has the classic symptoms of a pulmonary embolism. Symptoms of pulmonary embolism of polymethylmethacrylate (PMMA) following vertebral augmentation may occur with a delay. A symptomatic pulmonary embolism following vertebroplasty can occur either by migration of acrylic or the migration of fat and bone marrow cells. The MRI scan reveals a new superior endplate fracture involving L2. With this now being the third consecutive vertebral compression fracture in 2 months, one must be suspicious that these represent pathologic fractures, rather than osteoporosis. Risk factors for venous thromboembolic disease include increasing age, prolonged immobility, surgery, trauma, malignancy, pregnancy, estrogenic medications (eg, oral contraceptive pills, hormone therapy, tamoxifen [Nolvadex]), congestive heart failure, hyperhomocystinemia, diseases that alter blood viscosity (eg, polycythemia, sickle cell disease, multiple myeloma), and inherited thrombophilias. In addition to the risk associated with embolization of PMMA, the patient has been immobile for 7 days and was ultimately diagnosed with multiple myeloma.

 

 

(SAE11OS.53) A 48-year-old woman has an open subtrochanteric femur fracture. No other injuries are reported. After thorough evaluation, it is determined that she will need emergent surgical fixation. The patient and family indicate that they are practicing Jehovah's witnesses and desire adherence to the religious standards with respect to blood product usage. The patient signs a valid advanced directive confirming these wishes. Which of the following would be considered acceptable treatment? 

 

  1. Whole blood

     

  2. Platelets

     

  3. Plasma

     

  4. Starch product (ie, Hetastarch, Hespan)

     

  5. Donor-directed blood from a family member who is a practicing Jehovah's witness

 

PREFERRED RESPONSE 4

 

Jehovah's witnesses beliefs regarding blood products stems from direct interpretation of passages from the bible. The use of crystalloid, starch products such as Hetastarch and colloids are accepted. Typically Jehovah's witnesses will accept most medical treatment but refrain from the use of blood products including whole blood, packed red cells, platelets, white cells, or plasma. Any autologous transfusion, whether from the patient themself or donor directed, is forbidden. The use of cell-saver type processes is a matter of individual choice by the patient. The use of hemoglobin-based oxygen carriers are now accepted by many patients but it is important to respect the wishes of each individual patient. It is very important to discuss preoperatively with the patient and family their wishes and thoughts on what is acceptable to use. Many facilities have adopted bloodless-surgery protocols and committees that definitively outline the measures that can be used and take into consideration the many ethical issues involved in taking care of these patients.

(SAE10BS.89) A research study is initiated on 500 patients undergoing total hip arthroplasty. The patients are followed and outcome is assessed according to body mass index (BMI). The effects of BMI on outcome should be reported as which of the following? 

 

  1. Odds ratio

     

  2. Incidence rates

     

  3. Prevalence rates

     

  4. Relative risk

     

  5. Confidence intervals

 

PREFERRED RESPONSE 4

 

The study describes an example of a cohort study. Cohort studies follow a group of individuals over time and are optimal for studying the incidence, course, and risk factors of a disease. The effects in a cohort study are frequently reported in terms of relative risk (RR). Odds ratios are used to report effects in a case-control study. Incidence and prevalence rates are descriptors of a given characteristic either developed over time (incidence) or at one given time (prevalence). Confidence intervals are used to convey the significance of findings and are often used in lieu of or in conjunction with P values.

 

(SAE11OS.15) A minimally invasive plate osteosynthesis is seen in Figure 15. The resultant fracture healing can best be attributed to a fixation construct that was 

 

 

  1. stiff and stable.

  2. flexible and stable.

     

  3. facilitating direct osteonal healing.

     

  4. inhibitory to endochondral ossification.

     

  5. stimulatory to intramembranous ossification.

 

PREFERRED RESPONSE 2

 

Locked plating constructs with long-working lengths provide flexible but stable constructs that promote (not inhibit) endochondral ossification. Because of the longer working length they are not stiff, and these fractures do not heal with intramembranous ossification which occurs in bones like the calvarium. Direct osteonal healing is usually seen with constructs where absolute stability is achieved through interfragmentary compression, unlike in this case.

 

(SAE07HK.42) What property of titanium alloys accounts for their high corrosion resistance in vivo? 

 

  1. Self-passivation

     

  2. Ductility

  3. Hardness

     

  4. Modulus of elasticity

     

  5. Conductivity

 

PREFERRED RESPONSE 1

 

In both room temperature air and physiologic fluids, titanium alloys self-passivate or spontaneously form a layer of titanium oxide very rapidly. This layer makes titanium alloys resistant to surface breakdown.

 

(SBQ13PE.80) All of the following are true regarding the assessment of bone mineral density EXCEPT? 

 

  1. Bone mineral density loss is not apparent on standard radiographs until 30% to 40% of mineral is lost

     

  2. Dual energy x-ray absorbtiometry (DEXA) is only able to measure mineralization of the axial skeleton

     

  3. Single photon absorbtiometry allows for the analysis of bone mineralization in the appendicular skeleton alone

     

  4. Quantitative computed tomography (QCT) is associated with 10 times the radiation of a DEXA scan

     

  5. MRI allows for accurate discrimination of normal bone from osteopenic bones without exposing the patient to ionizing radiation

 

PREFERRED RESPONSE 2

 

Dual energy x-ray absorbtiometry (DEXA) scans allow for accurate assessment of bone mineralization in both the axial and appendicular skeleton.

 

Over the last decade, DEXA has become a safe, cost-effective and reliable method to quantify bone mineral density. The World Health Organization (WHO) has adopted DEXA derived BMD measurements to define normal bone, osteopenia, and osteoporosis in the adult and paediatric population. The typical DEXA analysis

therefore reports a Z-score, which is the number of standard deviations (SDs) that a patient's BMD is above or below the mean value for persons of the patient's age and sex. The T score is the number of SDs the patient's BMD is either above or below the mean value for young patients of the same gender.

 

Tortolani et al. provide an overview of the techniques used to assess bone mineral density, the pathophysiology of osteopenia, and the evaluation and treatment options for the general pediatric population as well as for patients with specific pediatric disorders. The authors encouraged orthopaedic surgeons to understand the problems of osteopenia in both otherwise healthy children and children with chronic disorders.

 

Binkovitz & Henwood provide a review of DEXA technique and interpretation with emphasis on the considerations unique to pediatrics. They found that changes in bone size over time, as occurs in normal pediatric development, confound DXA interpretation. This is one of the major limitations in the use of pediatric DXA. However, attempts to correct for or to circumvent this limitation have been proposed and show promise in improving DXA interpretation and our understanding of bone physiology.

 

T-score according to the World Health Organization (WHO):

 

A T-score of -1.0 or above is normal bone density. A T-score between -1.0 and -2.5 means you have low bone density or osteopenia. A T-score of -2.5 or below is a diagnosis of osteoporosis.

 

Incorrect Answers:

Answer 1: X-rays are a suboptimal diagnostic tool for osteopenia because bone mineral density loss is not apparent until 40% is already lost. Answer 3: Although single photon absorbtiometry (SPA) has shown reliable measurements of bone mineral density in the appendicular skeleton, measurements in the spine (axial skeleton) have been unreliable. Answer 4: QCT is associated with significantly higher radiation compared to DEXA and is therefore less frequently used. Answer 5: Conventional MRI scans can be used to measure bone mineral density, giving comparable results to quantitative computed tomography (QCT) without the exposure to ionizing radiation.

 

(SAE10BS.56) When the data are normally distributed, what statistical test is best used to compare means of three or more independent groups? 

  1. Kaplan-Meier analysis

     

  2. Analysis of variance

     

  3. Chi-square test

     

  4. Meta-analysis

     

  5. Log rank test

 

PREFERRED RESPONSE 2

 

Analysis of variance (ANOVA) is used to compare means of three or more independent groups with continuous variables that are normally distributed (eg, age, weight, height, etc). Kaplan-Meier analysis is used to analyze survivorship of subjects or products in an outcome study. Chi-square test is used to compare proportions for categorical variables. Meta-analysis is a systematic review method to analyze combined results of several independent studies, usually randomized clinical trials. Log rank is a statistical test to compare survivorship.

 

(SAE12TR.78) Long-term alendronate (Fosamax) use for osteoporosis has been associated with which of the following? 

 

  1. Scurvy

     

  2. Detached retina

     

  3. Uterine carcinoma

     

  4. Osteonecrosis of the femoral head

     

  5. Diaphyseal femoral insufficiency fractures

 

PREFERRED RESPONSE 5

 

Alendronate is a bisphosphonate that inhibits the ruffled border of the osteoclast. When used long term, this class of medication prevents the normal bone remodeling process. Long-term use has recently been shown to be associated with insufficiency

fractures of the femur. Osteonecrosis of the jaw has been described but not in other anatomic locations. Scurvy occurs because of a lack of vitamin C and use of bisphosphonates is not associated with uterine cancer or a detached retina.

 

(OBQ11.160) A decrease in alkaline phosphatase would most likely be manifest in which metabolic disorder? 

 

  1. Familial hypocalciuric hypercalcemia

     

  2. Hypophosphatasia

     

  3. X-linked hypophosphatemia

     

  4. Secondary hyperparathyroidism

     

  5. Tertiary hyperparathyroidism

 

PREFERRED RESPONSE 2

 

A decrease in tissue non-specific alkaline phosphatase (TNSALP) is found in hypophosphatasia.

 

TNSALP is found in osteoblasts and hydrolyzes inorganic phosphates, leading to an increase in serum phosphate levels, which helps to maintain physiologic levels. A decrease in this process impairs bone mineralization leading to rickets. In the perinatal period, hypophosphatasia and decreased mineralization leads to caput membraneceum, shortened limbs and respiratory failure. Childhood hypophosphatasia is marked by premature loss of deciduous teeth and rachitic deformities. Adult hypophosphatasia I is characterized by teeth and chest wall deformities (similar to adolescent hypophosphatasia) as well as recurrent metatarsal and femoral stress fractures.

 

Mornet reviewed hypophosphatasia and the alkaline phosphatase mutations. Screening for the 65 distinct mutation can aid in diagnosis and family counseling in severe forms.

 

Illustration A shows abnormal dentition found in hypophosphatasia. Illustration B shows widespread rachitic changes characteristic of hypophosphatasia.

Incorrect Answers:

Answers 1, 3, 4, 5: Patients with these conditions have elevated alkaline phosphatase.

 

(OBQ13.247) BMP is FDA approved for well-defined medical conditions in limited patient populations. In which of the following clinical scenarios is use of rhBMP-2 FDA approved? 

 

  1. In a 32-year-old male with an acute, open tibial shaft fracture treated with minimally-invasive locked plating 8 days from the initial injury

     

  2. In the lumbar spine in a 45-year-old female undergoing posterior decompression and instrumented posterolateral fusion from L4 to S1

     

  3. In the cervical spine in a 56-year-old female undergoing anterior cervical discectomy and fusion of C5-C6

     

  4. In the lumbar spine in a 52-year-old male undergoing L5-S1 anterior lumbar interbody fusion for degenerative disc disease and spinal stenosis

     

  5. In the cervical spine in a 60-year-old male undergoing posterior decompression and posterior instrumented fusion of C3-C7

 

PREFERRED RESPONSE 4

 

rhBMP-2 is FDA approved for use together with the lumbar tapered fusion device (LT Cage; Medtronic) in single-level ALIF from L2 to S1 levels in degenerative disc disease.

 

rhBMP-2 is also FDA approved for use in open tibial shaft fractures stabilized with an IM nail and treated within 14 days of initial injury. rhBMP-7 has received FDA humanitarian device exemption approval as an alternative to autograft in recalcitrant long bone nonunions where use of autograft is unfeasible and alternative treatments have failed. It is also approved as an alternative to autograft in compromised patients (with osteoporosis, smoking or diabetes) requiring revision posterolateral/intertransverse lumbar fusion for whom autologous bone and bone marrow harvest are not feasible or are not expected to promote fusion.

 

Epstein reviewed the indications for rhBMP-2 use and its off-label use. She found that

anterior cervical surgery with BMP2 led to dysphagia and airway complications. With posterior lumbar spine surgery, complications included vertebral osteolysis (bone resorption) and ectopic bone formation/heterotopic ossifications, which made revision surgery difficult.

 

Burkus et al. prospectively compared 46 patients undergoing single-level ALIF with BMP-2 vs with autograft in an industry sponsored study. They found that patients receiving BMP-2 had higher rates of fusion and improvement in pain and neurologic status at 12 and 24 months compared with autograft, and there were no adverse events. They recommend rhBMP-2 in ALIF procedures to eliminate pain and scarring from iliac crest bone harvest.

 

Illustration A shows the FDA approved combination of INFUSE (rhBMP-2) and LT-CAGE device.

 

Incorrect Answers:

Answer 1: rhBMP-2 is approved for open tibial shaft fractures stabilized with an IM nail and treated within 14 days of the initial injury. It is not approved for use with plates and screws. Answer 2: rhBMP-2 is not approved for posterior lumbar spine surgery. It is not approved for more than 1 spinal level. Answers 3 and 5: rhBMP-2 is not approved for cervical spine surgery.

 

(OBQ11.243) A clinical trial is underway for patients with wrist extensor tendinitis. One group of 100 patients are treated with short arm casting. Another group of 100 patients are treated with physical therapy. During analysis of the results, it becomes apparent that 30 patients in the physical therapy group did not complete the full course of physical therapy. Despite not completing a full course of physical therapy, these 30 patients were included in the physical therapy group for analysis. This analysis is an example of which of the following? 

 

  1. Per-protocol

     

  2. Crossover analysis

     

  3. Intent-to-treat

     

  4. Bayesian analysis

     

  5. Effect size

PREFERRED RESPONSE 3

 

The following is an example of intent-to-treat analysis.

 

The intent-to-treat approach aims to keep similar groups similar by not allowing for patient selection based on post-randomization outcomes (including failure to comply with the protocol). This type of analysis ensures the power of randomization so that important unknown variables that impact outcome are likely to be dispersed equally in each comparison group. Conversely, a per-protocol comparison in a clinical trial excludes patients who were not compliant with the protocol guidelines.

 

Berger et al., in a Level 5 review, discuss many of the principles beyond randomization that are critical for preserving the comparability of the different groups. They report that masking, allocation concealment, restrictions on the randomization, adjustment for prognostic variables, and the intent-to-treat approach to data analysis are important features of designing a good clinical trial.

 

Incorrect Answers:

Answer 1: Per-protocol- excludes patients who were not compliant with the protocol guidelines

Answer 2: Crossover studies involve switching patients between different arms of the study.

Answer 4: Bayesian analysis— An analysis that starts with a particular probability of an event (the prior probability) and incorporates new information to generate a revised probability (a posterior probability). Answer 5: Effect size— The difference in outcome between the intervention group and the control group divided by some measure of the variability, typically the standard deviation.

 

(SAE10HK.65) Which of the following statements best describes the process of articular cartilage degeneration in osteoarthritis? 

 

  1. In the second stage there is decreased catabolic activity with less matrix breakdown.

     

  2. In the second stage there is less chondrocyte proliferation and decreased matrix production.

     

  3. Matrix degradation includes increased proteoglycan production, more proteoglycan production, and longer glycosaminoglycan chains.

     

  4. Cartilage degeneration may be initiated by inflammation, overload, or decreased matrix production.

  5. Chondrocyte repair responses improve with aging.

 

PREFERRED RESPONSE 4

 

Inflammation, overload, or decreased matrix production may lead to cartilage degeneration. During the second stage of articular cartilage degeneration with osteoarthritis, there is increased chondrocyte activity with proliferation and increased production of extracellular matrix. At the same time, there is an increase in catabolic activity with removal of damaged matrix to facilitate matrix remodeling. Chondrocyte repair response decreases with aging. Matrix degradation includes decreased proteoglycan production, less aggregation, and shorter glycosaminoglycan chains.

 

(SAE13BS.60) Denosumab, a monoclonal antibody used to treat osteoporosis, works through inhibition of 

 

  1. receptor activator of nuclear factor kappa beta (RANK).

     

  2. RANK ligand (RANKL).

     

  3. osteoprotegrin (OPG).

     

  4. tumor necrosis factor (TNF).

 

PREFERRED RESPONSE 2

 

Denosumab is a monoclonal antibody that targets and inhibits RANKL binding to the RANK receptor, which is found on osteoclasts. As a result, it inhibits activation of osteoclast cells and slows the process of bone resorption and bone turnover via osteoclast inhibition. The end result is similar to bisphosphonates in terms of effector cell, but the mechanism of action is very different. RANKL binds to RANK, but OPG inhibits RANK binding to RANKL. TNF is an inflammatory cytokine, and monoclonal antibodies to TNF are used to treat systemic inflammatory disease such as rheumatoid arthritis.

 

(SAE08OS.181) What percent of adult patients with rheumatoid arthritis test positive for rheumatoid factor? 

 

  1. Less than 20%

     

  2. 20% to 40%

     

  3. 50%

     

  4. 80% to 90%

     

  5. More than 95%

 

PREFERRED RESPONSE 4

 

Rheumatoid factor is present in 80% to 90% of adult patients with rheumatoid arthritis. Although rheumatoid factor is very sensitive, it is not very specific. Antinuclear antibodies (ANA) can be detected in 30% to 40% of patients with rheumatoid arthritis.

 

(OBQ13.49) A 14-year-old patient has sustained a complete ACL tear of his right knee. Which of the following options has shown to be the most limiting factor for access to pediatric orthopaedic management in the United States? 

 

  1. Sex of the patient

     

  2. Type of health insurance

     

  3. Child greater than 10 years of age

     

  4. Acute knee injuries requiring operative treatment

     

  5. Timing of the referral

 

PREFERRED RESPONSE 2

The type of health insurance in the pediatric population has shown to be a significant factor for access to specialized healthcare in the United States.

 

Access to pediatric orthopaedic management has been well investigated. Numerous Level 4 studies have shown that orthopaedic offices in urban and rural areas prefer treating patients with private insurance over patients with Medicaid.

 

Iobst et al. telephoned 100 urban and rural orthopaedic outpatient offices to schedule an appointment for a 10-year-old patient with a forearm fracture. They showed that 8/100 offices would schedule an appointment within 1 week to the child with Medicaid insurance, as compared to 36/100 that gave an appointment to a child with private insurance.

 

Pierce et al. contacted 42 orthopaedic practices to schedule an appointment for a 14-year-old patient with an ACL injury. They showed that 38/42 offices scheduled an appointment for the child within 2 weeks with private insurance. This compared to 6/42 that scheduled an appointment for a similar child with Medicaid.

 

Incorrect Answers:

Answers 1,3,4,5: The limiting determinant to healthcare in the pediatric population has shown to be the type of health insurance. Sex, age of child, operative vs nonoperative injuries and timing of referral have not been shown to affect access to healthcare.

 

(OBQ14.186) Figure A demonstrates the molecular structure of a cell membrane protein important in propagating the action potential of a neuron. Which of the following medications acts by binding to the location marked with an X in the illustration? 

 

 

 

  1. Phentolamine

  2. Rocuronium

     

  3. Bupivacaine

     

  4. Midazolam

     

  5. Fentanyl

 

PREFERRED RESPONSE 3

 

Bupivacaine exerts its actions through blockade of the voltage gated sodium channel.

 

Local anesthetics of the amide family (lidocaine, bupivacaine) bind to the intracellular portion of voltage-gated sodium channels to block sodium influx. This prevents depolarization and the initiation or conduction pain. Lidocaine and bupivacaine have a duration of action of 2 and 4 hours, respectively (4 and 8 hours with epinephrine), and maximum doses are 4.5mg/kg and 2.5mg/kg, respectively (7mg/kg and 3mg/kg with epinephrine respectively). Both are metabolized in the liver, and excreted by the kidneys.

 

Phillips et al. review specific analgesics. Agents used to manage chronic pain include tricyclic antidepressants, anticonvulsants, GABA agonists, local anesthetic analogs, and NMDA antagonists. Opiates may trigger tolerance and lack of efficacy may develop. In those with refractory chronic pain, centrally administered analgesics may be considered, including opiates, dilute local anesthetic, NMDA receptor antagonists, clonidine, midazolam, baclofen, or calcium channel blockers. Single agents may be less effective than analgesic combinations.

 

Scholz discussed the function of local anesthetics and sodium channels. There are 3 states to sodium channels: (1) The closed state at potentials below -70mV. In this state, Na+ ions cannot pass from 1 side to another. (2) The open state, initiated by depolarization of the membrane to above -40mV. The channel opens to allow Na+ ions to diffuse through the pore, causing an inward current, depolarizing the membrane further. (3) The inactivated state follows activation during prolonged depolarization. In this state, inactivation is seen in macroscopic currents.

 

Figure A shows the voltage gated sodium channel. Local anesthetics such as lidocaine and bupivacaine act at the binding site marked "X".

 

Incorrect Answers:

Answer 1: Phentolamine competitively blocks alpha-adrenergic receptors and leads to vasodilatation. It antagonizes circulating epinephrine and/or norepinephrine. Answer 2: Rocuronium is a non-depolarizing neuromuscular blocking agent. It acts by competitively blocking the binding of acetylcholine to its receptors. Answer 4: Midazolam increases the efficiency of GABA to decrease the excitability of neurons. GABA receptors contain a chloride ion channel. By binding to the GABA receptor, midazolam increases the influx of chloride ions, hyperpolarizing the neuronal cell membrane, decreasing its excitability.

Answer 5: Fentanyl exerts its activity through binding to µ and kappa receptors. The supraspinal analgesic properties are mediated by the µ1 receptor, respiratory depression and physical dependence by the µ2 receptor, and sedation and spinal analgesia by the kappa receptor.

 

(SAE08OS.82) Limited weight bearing usually is recommended following open reduction and internal fixation of intra-articular lower extremity fractures. A bone graft, or bone graft substitute is often placed in the metaphyseal void beneath the reduced articular fragments. Which of the following bone grafts or bone graft substitutes will most likely permit earlier weight bearing without subsidence of the articular reduction? 

 

  1. Autogenous iliac crest bone graft

     

  2. Calcium phosphate

     

  3. Calcium sulfate

     

  4. Tricalcium phosphate

     

  5. Coralline hydroxyapatite blocks

 

PREFERRED RESPONSE 2

 

Most bone graft substitutes have a low compressive strength, similar to cancellous bone. Calcium phosphate cements, when hardened, have a much higher compressive strength compared to any of the other bone grafts or bone graft substitutes. In a study of 26 patients undergoing open reduction and internal fixation of displaced tibial plateau fractures, calcium phosphate was found to produce good outcomes. Because of the high mechanical strength of the cement, the authors allowed early weight bearing after a mean postoperative period of 4.5 weeks, with a range from 1 to 6 weeks. Despite early weight bearing, only two patients in this series had a partial loss of reduction. In biomechanical studies of displaced tibial plateau fractures, calcium phosphate compared favorably to cancellous bone graft. In one clinical series of patients undergoing open reduction and internal fixation for a calcaneus fracture, those patients whose reductions were supported with calcium phosphate were allowed to begin full weight bearing at 3 weeks and displayed no radiographic evidence of reduction loss. The effectiveness of calcium phosphate to resist deformation with cyclical loading in simulated calcaneal fractures has been confirmed in a biomechanical study.

 

(SAE07HK.50) Embolic material generated during total knee arthroplasty (TKA) shown in Figure 29 is composed of which of the following substances? 

 

 

 

  1. Fat only

  2. Fat and air

     

  3. Fat and marrow

     

  4. Fat and cement

     

  5. Fat and bone

 

PREFERRED RESPONSE 3

 

Emboli are created during TKA. Usually there is an increased incidence with the use of intramedullary rods that disrupt the marrow contents. These are not fat emboli per se. They are material composed of fat cells and marrow that act like pulmonary emboli to obstruct small arterioles in the lung. They are different from free fat emboli that are seen in fractures and that lead to chemical injury to the lung rather than obstructive injury.

(OBQ08.32) A 45 year-old woman who has not reached menopause yet falls from a standing height and sustains a distal radius fracture. A DEXA scan reveals a T-score of -2.2. Which of the following treatments is indicated in this patient? 

 

  1. 1,700 mg of calcium

     

  2. 1,200 mg of calcium

     

  3. 1,700 mg of vitamin D

     

  4. 1,200 mg of iron

     

  5. 1,700 mg of PTH

 

PREFERRED RESPONSE 2

 

The current recommendations for further prevention of any fragility fracture include 1200-1500mg of elemental calcium intake per day and 400-800 IU of vitamin D per day. Of note, these doses are indicated only for prevention and not sufficient for active treatment of osteoporosis (T score less than -2.5).

 

Medications that are approved by the FDA for active treatment of osteoporosis: alendronate, risedronate, raloxifene, estrogen, calcitonin. These medications and preventative measures help to reduce fragility fractures by as much as 50%.

 

Freedman et al. performed a retrospective study that looked at a cohort of patients with fragility fractures and then looked at the type and frequency of osteoporosis related interventions. They found that only 60% of patients actually were either prescribed a medication, given a referral, or ordered additional workup (DEXA scan).

 

Schulman et al. reviewed a series of 80 female patients regarding osteoporosis and bone health, and found that the outpatient sports medicine office setting was an excellent opportunity to educate patients on these topics. The patients' post-education test scores increased significantly after a brief counseling session, and increases in daily calcium intake and exercise levels were also seen.

 

(SAE13BS.97) A patient with Paget disease who is intolerant of bisphosphonates is given calcitonin. What is the mechanism of action of calcitonin? 

 

  1. Promotes reabsorption of phosphate in the renal tubules

  2. Interferes with osteoclast maturation

     

  3. Interferes with intestinal absorption of calcium

     

  4. Upregulates osteoblast formation

 

PREFERRED RESPONSE 2

 

Calcitonin is a hormone that reduces serum calcium concentration by directly interfering with osteoclast maturation via receptors. Calcitonin inhibits phosphate reabsorption and decreases calcium reabsorption in the kidneys. By attenuating cartilage breakdown and stimulating cartilage formation via inhibitory pathways of matrix metalloproteinases, calcitonin also has a chondro-protective effect on articular cartilage. Calcitonin has no major effects on intestinal absorption of calcium, but may aid in small-bowel secretion of sodium, potassium, chloride, and water. Calcitonin also has no receptor effect on osteoblasts.

 

(SAE13BS.35) A 14-year-old boy has failed physical therapy management for Scheuermann kyphosis, and an extension thoracolumbosacral orthosis brace is recommended. The boy and his parents are told that the brace will force his thoracic spine into normal sagittal alignment and put the anterior vertebral bodies of the thoracic segment into tension, which will induce bone growth and normalization of wedge-shaped vertebrae. What name is associated with this process? 

 

  1. Hooke's law

     

  2. Kirchhoff's law

     

  3. Wolff's law

     

  4. Heuter-Volkmann principle

 

PREFERRED RESPONSE 4

 

The Heuter-Volkmann principle shows that bone placed in longitudinal tension will tend to stimulate longitudinal growth, and that compressive longitudinal forces inhibit longitudinal growth, making this response the best choice. Hooke's law relates to stress being proportional to strain and is not directly related to bone growth. Kirchhoff's laws apply to electrical circuit design. Wolff's law states that bone

remodels in response to mechanical stress, with the correlate that increased stress causes increased growth, and decreased stress leads to bone loss.

 

(SAE13BS.98) A cartilage water content increase is the hallmark of which osteoarthritis stage? 

 

  1. Prearthritis

     

  2. Early

     

  3. Late

     

  4. Terminal

 

PREFERRED RESPONSE 2

 

The first stage of osteoarthritis is marked by an increase in water content secondary to disruption of the matrix framework. This is followed by an increase in chondrocyte anabolic and catabolic activity in response to tissue damage. Wnt-induced signal protein 1 increases chondrocyte protease expression. Failure to restore tissue balance ultimately leads to continued destruction and osteoarthritis. One hallmark of osteoarthritic cartilage is a reduced repair mechanism attributable to decreased chondrocyte response to growth factor stimulation (transforming growth factor-alpha and insulin-like growth factor-1). Mitochondrial dysfunction and increased production of reactive oxygen species may promote cell senescence, a progressive slowing of cellular activity. Microscopic evidence of cartilage degeneration begins with fibrillation of the superficial and transition zones, followed by disruption of the tidemark by subchondral blood vessels and eventual subchondral bone remodeling. This process ultimately leads to cartilage degradation with decreased water content in the late and terminal phases of osteoarthritis.

(SAE13BS.2) Sclerostin and dickkopf-1 (Dkk-1) are direct inhibitors of what pathway related to bone and/or cartilage regulation? 

 

  1. Bone morphogenetic protein (BMP)/SMAD pathway

     

  2. Receptor activator of nuclear factor kappa beta (RANK)/RANK ligand (RANKL) pathway

     

  3. Wnt/Beta-catenin (ß-catenin) pathway

     

  4. Parathyroid hormone (PTH) pathway

 

PREFERRED RESPONSE 3

 

Dkk-1 and sclerostin are proteins that inhibit the binding of the Wnt molecule to receptors LRP5/6. In the absence of sclerostin and Dkk-1, Wnt binds to its receptor, which in turn inhibits phosphorylation of the ß-catenin. The unphosphorylated ß-catenin then builds up in the cytoplasm of the cell, allowing it to be transported to the nucleus of the cell. Once in the nucleus, ß-catenin will lead to upregulation of a series of proteins involved in osteoblast formation differentiation. Knocking out or inhibiting sclerostin or Dkk-1 results in increased bone mass secondary to constitutive activation of the Wnt/ß-catenin pathway. The other responses are not directly affected by Dkk-1 or sclerostin. RANKL and RANK are expressed on osteoblasts and osteoclasts, respectively, and are involved in osteoblast-mediated osteoclast activation. BMPs work through SMADs to cause osteoblastic differentiation, and there is reported crosstalk between the Wnt and BMP pathways (but this is an indirect link). Finally, PTH at physiologic levels binds to osteoblasts, causing a series of events that lead to osteoblast-mediated osteoclast activation and subsequent increased bone resorption.

 

(SAE13BS.81) Figure 81 is the radiograph of a healthy 72-year-old man who has a 3-month history of medial knee pain. He denies any specific trauma. Until 3 months ago when the pain began, he had been an avid runner for many years. Initial treatment should be oral anti-inflammatory medication 

 

 

  1. alone.

  2. with food.

     

  3. with a proton pump inhibitor.

     

  4. with glucosamine hydrochloride.

 

PREFERRED RESPONSE 3

 

This patient has bone-on-bone end-stage arthritis that was asymptomatic until 3 months ago. Patients older than age 65 are at increased risk for adverse gastrointestinal effects when placed on nonsteroidal anti-inflammatory drugs, and they should be placed on a proton pump inhibitor prophylaxis at the same time. Use of glucosamine is controversial. Glucosamine hydrochloride has been shown in prospective studies to have no beneficial effect. However, glucosamine sulfate has been shown to have a positive effect on pain, but no effect on function.

 

(OBQ15.215) Which specific legislative Act in the United States was created to require reporting of annual monetary gifts or compensation of more than $10 by orthopaedic implant companies to physicians? 

 

  1. Patient Protection and Affordable Care Act

     

  2. Medicare Payment Reform Act

     

  3. Physician Financial Transparency Act

     

  4. Physician Payments Sunshine Act

  5. Health Insurance Portability and Accountability Act

 

PREFERRED RESPONSE 4

 

The Physician Payments Sunshine Act requires all payments by corporations to physicians beyond $10 per year to be reported to the Centers for Medicare and Medicaid Services.

 

Under this Act, all manufacturers of drugs and devices covered under Medicare, Medicaid, and SCHIP are obliged to federally report payments beyond $10 annually to physicians and academic centers. The Act was first introduced in 2007, enacted in 2010, and in 2014 the first data (from 2012) was reported publicly online in the Open Payment Program of the Centers for Medicare and Medicaid Services website.

 

Samuel et al analyze orthopedic surgeons available data from the Sunshine Act regarding industry payments and find over 110 million USD paid to approximately 15,000 orthopedic surgeons over the 5-month study period. No long term data exists to determine if these payments have any affect in healthcare.

 

Incorrect Answers:

Answers 1: The Patient Protection and Affordable Care Act (PPACA), known also by its shorter name of the Affordable Care Act (ACA) or it's nickname "Obamacare", was passed in March 2010. The Sunshine Act was one of many provisions passed within the PPACA (after the Sunshine Act failed to pass on its own in prior years), but the PPACA focused primarily on improving the quality and affordability of healthcare insurance and lowering the costs of healthcare. Answer 2: The Medicare Payment Reform Act of 1983 was a quickly drafted revision to the way Medicare payments were made, changing from fee-for-service to prospective payments allowing Medicare to determine payment amount rather than providers/hospitals.

Answer 3: This is a fictitious act. Answer 5: HIPPA is the 1996 legislation defining standards and protections for patient private health information and electronic exchange of records.

 

(SAE10PE.47) Nutritional rickets in the US occurs more frequently in infants older than 6 months of age who do not receive vitamin D supplementation and are Review Topic

  1. Caucasian and formula fed.

     

  2. Caucasian and breast fed.

     

  3. African American and formula fed.

     

  4. African American and breast fed.

     

  5. Asian and formula fed.

 

PREFERRED RESPONSE 4

 

Numerous reports suggest an increased frequency of nutritional rickets in the US in children with dark skin pigmentation who are breast fed past 6 months of age without vitamin D supplementation. Nutritional rickets is rare in light-skinned children or those who are formula fed.

 

(SAE13BS.90) An otherwise healthy 50-year-old man who is a smoker undergoes a posterior spine fusion with instrumentation for spondylolisthesis. What can the patient do to minimize his risk for pseudarthrosis? 

 

  1. Increase calcium and vitamin D intake

     

  2. Avoid all nonsteroidal anti-inflammatory drugs (NSAIDs)

     

  3. Maintain smoking cessation

     

  4. Engage in early physical therapy to strengthen the trunk musculature

 

PREFERRED RESPONSE 3

 

Smoking is the biggest risk factor for nonunion and should be strictly avoided. NSAIDs interfere with bone healing, but not as strongly as smoking. Early mobilization would potentially stress the construct, inducing movement that leads to nonunion. Without history of calcium and vitamin D deficiency, increasing intake would not decrease the risk of nonunion.

 

 

(SAE13BS.61) An article in a leading orthopaedic journal describes a randomized controlled study comparing 2 popular and comparable surgical methods for fixation of hip fractures. Patients were randomized by a predetermined computer-generated list. There was approximately 20% crossover between treatment groups. Evaluation of study outcomes (clinical and radiographs) was performed by one of the surgeons participating in the study. Enrollment goals for appropriate power were achieved and final 1-year follow-up was obtained for 65% of patients. According to Oxford Group criteria, what is the appropriate level of evidence for this study? Review Topic

 

  1. Level I

     

  2. Level II

     

  3. Level III

     

  4. Level IV

 

PREFERRED RESPONSE 2

 

Although this is a randomized controlled study, there is substantial crossover between the treatment groups, evaluation could not be blinded (radiographs) and was performed by one of the surgeons, and follow-up was moderate. According to Oxford Group criteria, this would be a Level II study.

 

 

(SAE13BS.92) A researcher decides she wants to look at the current total number of patients who have methicillin-resistant Staphylococcus aureus (MRSA) infections in a hospital on 1 particular day. What is the researcher measuring? 

 

  1. Correlation coefficient of MRSA

     

  2. Prevalence of MRSA

     

  3. Incidence of MRSA

     

  4. Relative risk of MRSA

PREFERRED RESPONSE 2

 

The prevalence of a disease is a measure of the number of cases of a disease at or during a specific time point or time period. In this case, the researcher wants to know the prevalence of disease on a given day. Incidence measures new cases of a disease or event per unit of time. Correlation coefficient is a measure of how 2 things correlate with one another, while relative risk is a statistical outcome that is often used in case-control or cohort studies to provide a measure of the risk of a particular disease occurring when a certain exposure has already occurred.

 

(OBQ07.109) When a Workers' Compensation patient recovers after an injury to a point that further restoration of function is no longer anticipated, he or she is said to have reached which of the following? 

 

  1. Functional capacity

     

  2. Maximum medical improvement

     

  3. Permanent disability

     

  4. Impairment rating

     

  5. Predesignation

 

PREFERRED RESPONSE 2

 

This is the definition of maximum medical improvement (MMI). The patient has essentially reached the plateau of his improvement.

 

Functional capacity evaluations (FCE) are based upon a theoretical model of comparing job demands to worker capabilities. The results of FCEs are often used to determine musculoskeletal capacity to return to work.

 

Strong et al. reported on the use of FCE in the Workers' Compensation system, and note how these FCE results are required by employers to determine the level of return to work of their employees. They also mention that the reports are frequently perceived with a negative tone. The employees reported a wider range of restrictions in their varied life roles than did the FCE reports, which deal more narrowly with work roles.

Pransky et al. reported that although FCE's are relied upon for determination of ability to perform physical work, several scientific, legal, and practical concerns persist. They note that test criteria often do not accurately reflect real-life job requirements or performance, and subjective evaluation remains common. They conclude that more research into predictive linking of FCE outcomes with occupational outcomes is necessary to determine their role in the Workers' Compensation system.

 

Incorrect Answers:

1: A functional capacity evaluation (FCE) is set of tests, practices and observations that are combined to determine the ability of the evaluated to function in a variety of circumstances (most often employment) in an objective manner. 3: Permanent disability is any lasting disability that results in a reduced earning capacity after maximum medical improvement is reached; this implies that MMI must be reached before this is determined. 4: Impairment rating is an objective data point obtained by a physician reviewing the patient's overall condition during a functional capacity evaluation. 5: This is the process a patient uses to tell their employer they want a personal physician to treat them for a work injury.

 

(SAE13BS.31) Which of the following antibiotics inhibits cell-wall production by interfering with the addition of cell-wall subunits 

 

  1. Aminoglycosides

     

  2. Fluoroquinolones

     

  3. Glycopeptides

     

  4. Rifamycins

     

  5. Macrolides

 

PREFERRED RESPONSE 3

(SAE10SM.91) Which of the following most accurately approximates the estimated risk of a musculoskeletal allograft containing the human immunodeficiency virus (HIV) despite adequate screening? 

 

1 1 in 600

 

2 1 in 6,000

 

3 1 in 60,000

 

4 1 in 1,600,000

 

5 1 in 6,000,000

 

PREFERRED RESPONSE 4

 

The calculated risk of a musculoskeletal allograft containing HIV despite adequate screening has been estimated to be approximately 1 in 1.6 million. This estimate is based on the risk of HIV in the population, projected population estimates, and current methods of donor screening.

 

(OBQ11.219) Which of the following best describes a Bonferroni correction? Review Topic

 

  1. An analysis that starts with a particular probability of an event (the prior probability) and incorporates new information to generate a revised probability (a posterior probability)

     

  2. Human behavior that is changed when participants are aware that their behavior is being observed.

     

  3. Used to assess the relationship between two normally distributed continuous variables

     

  4. A post-hoc statistical correction made to P values when several dependent or independent statistical tests are being performed simultaneously on a single data set

  5. The ability of a study to detect the difference between two interventions if one in fact exists

 

PREFERRED RESPONSE 4

A Bonferroni correction is a post-hoc statistical correction made to P values when several dependent or independent statistical tests are being performed simultaneously on a single data set.

 

To perform a Bonferroni correction, divide the critical P value (alpha level) by the number of comparisons being made. For example, if 10 hypotheses are being tested, the new critical P value would be (alpha level)/10. The statistical power of the study is then calculated based on this modified P value.

 

Guyatt et al. discusses hypothesis testing and the role of alpha levels and P values. They report that the Bonferroni correction is derived from testing a dependent or independent hypotheses on a set of data and finding that the probability of a type I error is offset by testing each hypothesis at a statistical significance level divided by the number of times what it would be if only one hypothesis were tested.

 

Incorrect

 

 

 

 

Answers:

Answer

1:

This

describes

Bayesian

analysis.

Answer

2:

This

describes

Hawthorne

effect.

Answer

3:

This

describes

Pearson

correlation.

Answer 5: This describes Study power.

 

(SAE08UE.25) A 49-year-old woman with serologically proven rheumatoid arthritis has Larsen grade II radiographic changes in the elbow. Examination reveals a preoperative arc of flexion of less than 90 degrees and there is no instability. Nonsurgical management has failed to provide relief. What is the best treatment option? 

 

  1. Semiconstrained total elbow arthroplasty

     

  2. Unlinked total elbow arthroplasty

     

  3. Fascial arthroplasty

     

  4. Open synovectomy

     

  5. Arthroscopic synovectomy

 

PREFERRED RESPONSE 5

Larsen grade I and II rheumatoid arthritis is best treated with synovectomy with arthroplasty reserved for later stages, especially in younger patients. Open synovectomy with or without a radial head excision has yielded good results for pain and function, with arthroscopic synovectomies yielding similar results. Arthroscopic synovectomy has been shown to be more effective in restoring function in patients with a flexion arc of less than 90 degrees.

 

(SAE07SM.26) What type of exercise is used early in the rehabilitation process to safely stimulate co-contraction of the scapular and rotator cuff muscles? Review Topic

 

  1. Resistive active motion

     

  2. Facilitated active motion

     

  3. Plyometrics

     

  4. Open kinetic chain

     

  5. Closed kinetic chain

 

PREFERRED RESPONSE 5

 

Closed kinetic chain exercises are used early in the rehabilitation process. The distal segment is fixed, and an axial load is applied which provides glenohumeral compression and reduces the demand on the rotator cuff. These exercises stimulate co-contractions of the scapular and rotator cuff muscles, load scapular stabilizers, and facilitate active motion. Facilitated active motion exercises use proximal segment motion to stimulate and facilitate motion in the target tissue. These exercises are often performed in diagonal movements. Resistive active motion exercises are used later in the rehabilitation process. These are typically open kinetic chain exercises that involve active glenohumeral motion with extrinsic loads such as weights or exercise tubing. During the later stages of upper extremity rehabilitation, plyometrics are added. These exercises help to prepare the athlete for return to sport. When performed at slower speeds, these exercises emphasize stabilization and control. As the speeds increase, muscles begin to work in the stretch-shortening sequence associated with sports participation.

 

 

(OBQ11.159) Which of the following genetic defects does not follow Mendelian patterns of inheritance? 

 

  1. Defect in the fibroblast growth factor (FGF) receptor 3

     

  2. Mutation of a gene coding for a sulfate transport protein

     

  3. Defect in the Gs-alpha protein

     

  4. Defect in the fibrillin gene

     

  5. Deficient activity of the enzyme ß-glucosidase (glucocerebrosidase)

 

PREFERRED RESPONSE 3

 

Defect in the Gs-alpha protein, leading to McCune-Albright syndrome, do not follow Mendelian patterns of inheritance.

 

Gs mutations are seen in fibrous dysplasia including monostotic (80%), polyostotic (20%) or, rarely as part of a syndrome (McCune Albright). Fibrous dysplasia is not inheritable and exhibits mosaicism, whereby the random mutation in the GNAS gene occurs early in development and as a result there is a portion of normal version of the GNAS gene, while other cells have the mutated version. McCune-Albright syndrome (polyostotic fibrous dysplasia, café-au-lait spots, precocious puberty) is caused by a sporadic mutation of the Gs-alpha subunit of the receptor/adenylyl cyclase–coupling G proteins and does not follow Mendelian inheritance patterns.

 

Mendelian patterns of inheritance include one of four modes: autosomal dominant, autosomal recessive, X-linked dominant, and X-linked recessive. The approximate rate of mendelian disorders in humans is 1%. Mendelian genetics allows for specific patterns of inheritance controlled by a single gene pair (“monogenic”).

 

Lietman et al report that addition of a peptide nucleic acid (PNA) primer to the PCR testing can better identify the mutant GNAS alleles in DNA from peripheral blood cells from patients with McCune-Albright syndrome and fibrous dysplasia.

 

Leet et al performed a survey study of 20 patients with polyostotic fibrous dysplasia and found that the loss of the normal femoral neck-shaft angle and the disease burden in the lower extremities appear to have the greatest effect on functional activity.

 

Illustration A shows a case of x-linked recessive pattern of inheritance. Incorrect Answers:

Answer 1: Achondroplasia is autosomal dominant Answer 2: Diastrophic dysplasia is autosomal recessive Answer 4: Marfan's syndrome is autosomal dominant Answer 5: Gaucher's disease is autosomal recessive

 

(SAE11PA.56) A 72-year-old woman is evaluated for sacrococcygeal pain sustained after a twisting injury. Radiographic and MRI evaluation confirms the presence of a nondisplaced fracture at the sacrococcygeal junction. Over a 3-week period, the pain has gotten significantly better. No additional lesions or injuries are noted. Laboratory studies show a serum calcium level of 8.8 mg/dL (normal 8.6-10.3 mg/dL) and a 25-OH Vitamin D level of 14 ng/mL (normal 30-80 ng/mL). What is the most appropriate treatment for this patient? 

 

  1. Expectant observation

     

  2. Calcium supplementation

     

  3. High dose vitamin D supplementation

     

  4. Bisphosphonate therapy

     

  5. Surgical fixation of the sacrococcygeal fracture

 

PREFERRED RESPONSE 3

 

Chronic Vitamin D deficiency leads to problems with bone health and has been shown to increase the risk of falls in the elderly. Appropriate supplementation of Vitamin D has been shown to decrease this risk. Conversion in the skin decreases with age and may be nearly nonexistent in darkly pigmented individuals. Vitamin D3 is the preferred form for supplementation, but D2 is the form most available by prescription in the US. Hypervitaminosis D is rare and very high doses can be tolerated without significant concern for toxicity. Because the patient has sustained one insufficiency fracture, she is at risk for insufficiency fractures in other skeletal locations, rendering expectant observation insufficient. Her serum calcium is normal, and with a low Vitamin D level, calcium utilization in her system would be inadequate. Bisphosphonate therapy in addition to calcium and vitamin D supplementation may provide a good long-term solution, but should not be instituted until the bone mineral imbalance has been adequately corrected. Surgical fixation of this fracture is not indicated, particularly in lieu of improving symptoms.

 

 

(OBQ13.150) Figures A and B show radiographs of a 24-year-old female with a soccer injury. A physical examination reveals an isolated, closed injury with no clinical features of neurovascular injury or compartment syndrome. She has been consented to be treated with intramedullary nail fixation. A pre-operative note by the anaesthesiology team makes reference to the patients fair skin and natural red-hair color. How will this information affect the post-operative management of this patient? 

 

 

 

  1. Longer duration of anticoagulation due to increased risk of DVT

  2. Avoiding anticoagulation medications due to increased risk of bleeding

     

  3. Require higher dosages of post-operative analgesia

     

  4. Longer period of non-weight bearing on surgical limb

     

  5. Avoiding opioids due to higher risk of unrecognized allergies

 

PREFERRED RESPONSE 3

 

Female patients with natural red-hair may require higher dosages of post-operative analgesia compared to other hair types.

 

Melanocortin-1-receptor (MC1R) is one of the key proteins involved in hair color and skin tone. Mutations of the MC1R alleles can render this protein non-functional, which results in a phenotype of red-hair and fair skin. Mutations of the MC1R have shown to modulate the pain response and opioid efficacy in these patients. Women are

more commonly affected and often require more anaesthetic and higher dosages of opioid to achieve comparable MAC level and pain-relief, respectively, as women with other hair types.

 

Liem et al. showed that a greater concentration of induction and maintenance agents (sevoflurane and desflurane, respectively) were required to sustain comparable MAC levels in red-haired patients as dark haired patients.

 

Fillingim et al. reviewed the affect of gender, sex and pain. They concluded there is a biopsychosocial element of pain that is perceived differently by men and women. In terms of postoperative and procedural pain, the outcome might be more severe in women than men.

 

Delaney et al. looked at the involvement of the melanocortin-1 receptor in acute pain in mice. They found that while the MC1R is better known as a gene involved in mammalian hair colour, it was shown to be involved in the pain pathway of inflammatory but not neuropathic origin. Mutations of MC1R showed increased tolerance to noxious pain stimulus in mice.

 

Figures A and B are AP and lateral radiographs of a left tibia. There is a low energy, distal third shaft fracture with no cortical apposition on the AP view.

 

Incorrect Answers:

Answer 1,2: Mutations in the Melanocortin-1-receptor (MC1R) has not shown to affect the coagulation pathway, with no increased risk of bleeding or clotting. Answer 4: Mutations in the Melanocortin-1-receptor (MC1R) does not affect fracture fixation or weight-bearing status post-operatively Answer 5: There is not a higher risk of opioid allergy in these patients.

 

(OBQ13.182) A Spanish speaking child sustained the injury seen in Figure A after a fall at school. He was casted in the emergency department without the assistance of an interpreter and advised to return to see an orthopaedic surgeon in 1 week. However, the family returns to the emergency department with the child 3 months later, still in the cast. What is the most likely reason the child did not attend the recommended orthopaedic follow-up visit.? 

 

 

  1. The child is a victim of neglect

  2. The child had no symptoms of pain

     

  3. He was allowed to return to school wearing the cast

     

  4. Concerns of cost

     

  5. Follow-up instructions were not effectively communicated

 

PREFERRED RESPONSE 5

 

The most likely reason the child did not attend the recommended orthopaedic followup visit was a language barrier preventing effective communication of the intended follow-up instructions.

 

Communication skills and cultural competence is a key element in good orthopaedic care. Poor communication can often lead to devastating outcomes. In this example, poor communication resulted in this patient being lost to follow-up. Language barriers must be accommodated and alternative methods of communication must be utilized.

 

Levinson et al. examined how patients present their medical issues in clinical encounters and how physicians respond to these clues in routine primary care and surgical settings. They showed that good communication relies mostly on the physicians ability to identify patient clues within the clinical encounter. Poor communication between the physician tended to delay clinical visits, poor follow-up and unsatisfactory outcomes.

 

Figure A is an AP radiograph of the elbow in a skeletally immature patient. Figure B is a lateral radiograph of the elbow with a posterior fat pad sign, suggestive of an occult fracture.

 

Incorrect Answers:

Answer 1,2,3,4: The most likely reason for loss to follow-up in this scenario is miscommunication regarding follow-up.

 

(SAE07HK.46) A 62-year-old woman with a bone mass density (BMD) T-score of -

2.0 sustained a subcapital fracture of her hip. She is an avid tennis player, and history reveals no previous fractures. What is the most appropriate follow-up care? Review Topic

 

  1. Antiresorptive bisphosphonate medication

     

  2. A repeat dual-energy x-ray absorptiometry scan (DEXA) and treatment if the T-score is less than -2.5

     

  3. A repeat DEXA scan and treatment if the T-score is greater than -1.5

     

  4. No treatment since the BMD is not in osteoporotic range

     

  5. (PTH) taraparatide hormone followed by surgery

 

PREFERRED RESPONSE 1

 

A DEXA scan is most appropriately used to establish a baseline score. Even if the bone mineral density is not within the osteoporotic range (T-score less than -2.5), a prior fragility fracture is a strong risk factor for a second fracture as a result of factors other than bone density, such as worsening vision or balance, confusion, or other predispositions to falls. The guidelines of the National Osteoporosis Foundation indicate that, following a fragility hip fracture, active anti-osteoporotic medication should be initiated, whether or not a DEXA scan is performed. A recent study showed that antiresorptive therapy following a hip fracture reduces not only the risk of a second fracture but also overall mortality.

 

(SAE10BS.7) Rickets can be best characterized by which of the following radiographic changes in children? 

 

  1. Narrowed physes

  2. Widening of the ribs anteriorly

     

  3. Rounded epiphyses

     

  4. Punched out metaphyseal defects with subperiosteal resorption

     

  5. Sclerotic bands in the upper and lower margins of each vertebra

 

PREFERRED RESPONSE 2

 

Rickets is a condition of inadequate mineralization of osteoid and cartilage at the growing ends of bone in children. It is associated with a nutritional deficiency in children that affects open physes, as opposed to osteomalacia which is a deficiency in people with closed growth plates. There are classic radiographic findings and disorders that mimic rickets. Classic findings are widened physes related to widening of the physis from accumulation of nonmineralized osteoid and cartilage, irregular metaphyses, and loss of definition of the zone of provisional calcification. The changes are seen predominantly in the bones with greatest growth (tibia, distal femur, distal radius) and rarely seen in slower growing bones such as the tubular bones in the hand. Widening of the anterior ribs, the rachitic rosary, may be seen. Hyperparathyroidism with accumulation of fibrous tissue in the metaphyses can produce lucencies that may appear similar to the nonmineralized osteoid of rickets. Hyperparathyroidism has erosion around the edge of the growth plate. There are punched out lesions and resorption subperiosteally. Rounded epiphyses and marked separation of the epiphysis and metaphysis are the classic differentiation of Jansen syndrome, a dominantly inherited metaphyseal chondrodysplasia. Sclerotic metaphyseal ends are similar to rickets, but with maturation, the spaces fill with irregular calcification classic to Jansen syndrome. Sclerotic of the vertebra, or "rugger jersey" spine, is typical of renal osteodystrophy.

 

(SAE13BS.38) A 35-year-old woman began to train for a half marathon. After 8 weeks of increasing her mileage, what changes can you expect in her Achilles tendon? 

 

  1. Net decrease of type I collagen

     

  2. Net increase of type I collagen

     

  3. Increased diameter of collagen fibrils

     

  4. Increased cross-sectional area of the tendon

PREFERRED RESPONSE 2

 

Training increases turnover of type I collagen, promoting both synthesis and degradation of collagen and a net increase synthesis of type I collagen in tendon-related tissue. Strenuous endurance training has resulted in decreased collagen cross-links, suggesting increased collagen turnover, but decreased collagen maturation. In human studies, physical training results in increased turnover of collagen. Synthesis and degradation are elevated initially when beginning an exercise program, but degradation products decrease overall. It is not known if activity levels in humans affect the diameter of collagen fibrils or the cross-sectional area of tendons.

 

(OBQ14.72) Integrins function in which of the following ways? 

 

  1. Maintain bone and cellular matrix

     

  2. Form bone by producing non-mineralized matrix

     

  3. Form cartilage under intermediate strain and low oxygen tension

     

  4. Attachment of osteoclasts to bone surfaces

     

  5. Binds to RANKL on the osteoblast, preventing RANK activation

 

PREFERRED RESPONSE 4

 

Integrins are groups of molecules essential for osteoclast attachment to the bone surface.

 

Integrins are a large family of heteromeric cell surface receptors composed of non-covalently bound alpha and beta subunits which interact with extracellular matrix molecules, serum constituents and various adhesion molecules. Specifically, aVß3 is a type of integrin found on osteoclasts that attaches to bone by coupling with vitronectin.

 

Bikle reviews the cause of bone loss during periods of unloading. They state the during periods of rest, integrin expression is decreased, leading to disuse osteoporosis. They argue that drugs that selectively regulate the integrin signaling pathway may

reverse bone loss during periods of disuse.

 

Illustration A shows an example of an osteoclast attaching to bone via the integrin aVß3.

 

Incorrect

 

 

 

 

 

 

 

Answers:

Answer

 

1:

This

 

is

the

function

of osteocytes.

Answer

 

2:

This

 

is

the

function

of osteoblasts.

Answer

3:

 

This

is

the

function

of

osteoprogenitor cells.

Answer 5: This is the function of osteoprotegerin.

 

(SAE10BS.95) What is the most serious complication that may occur with a ceramic-on-ceramic bearing in a total hip prosthesis? 

 

  1. Wear

     

  2. Squeaking

     

  3. Dislocation

     

  4. Fracture

     

  5. Subsidence

 

PREFERRED RESPONSE 4

 

Fortunately, due to improvements in strength and quality control, fracture of a modern ceramic hip bearing is very rare. However, when a ceramic bearing does fracture, numerous fragments may become embedded in the capsular tissues. During revision, the surgeon must make every reasonable effort to locate and remove these fragments. Among other complications, fragments that remain behind can become entrapped between the bearing surfaces of the new implant and initiate severe wear. In normal use, ceramic bearings have excellent wear resistance. Squeaking may be extremely annoying to the patient, but is not inherently dangerous and sometimes resolves spontaneously. Dislocation is a serious complication regardless of the type of bearing, but it sometimes may be treated without requiring a reoperation and, if revision is required, it typically is not as problematic as with a fractured ceramic bearing. Infection is to be avoided, but it is not a complication directly related to the use of a ceramic bearing.

 

 

(SAE13BS.15) Implants composed of polylactic acid are excreted by what system after they are absorbed? 

 

  1. Hepatic

     

  2. Renal

     

  3. Respiratory

     

  4. Gastrointestinal

 

PREFERRED RESPONSE 3

 

Polylactic acid suture and suture anchors are popular bioabsorbable orthopaedic implants. This material undergoes hydrolysis of the ester background in vivo. Resulting lactic acid enters the tricarboxylic acid (Krebs) cycle and is excreted as carbon dioxide by the lungs. Polyglycolic acid and poly(p-dioxanone) may also be excreted by the kidneys.

 

(OBQ15.207) In regards to a genetic disorder, which of the following is an example of "anticipation?" 

 

  1. Gene characteristics more severe and earlier in onset in subsequent generations

     

  2. A disorder inherited from a genetic mutation specific to maternal DNA

     

  3. Gene characteristics expressed to varying degrees in different individuals

     

  4. Variation in the relative frequency of a genotype due to chance

     

  5. The presence of an extra copy of a chromosome

 

PREFERRED RESPONSE 1

Genetic anticipation is a phenomenon in which a genetic disorder becomes progressively more severe and earlier in onset with each generation. Examples of disorders exhibiting anticipation include Huntington's disease and myotonic dystrophy.

 

Genetic anticipation is an important concept in understanding the development and genetic implications of many heritable disorders. It is a common phenomenon in trinucleotide repeat expansion disorders. These disorders are due to unstable microsatellite trinucleotide repeats that expand beyond the normal threshold. In subsequent generations these expansions become longer and thus express disease characteristics at a younger age of onset, and often with greater severity.

 

Martorell et al. investigated the development of CTG trinucleotide repeats in patients with myotonic dystrophy type 1 (DM1) and their relatives. They discovered unaffected individuals carry a pre-mutation sequence which can lead to trinucleotide repeat expansion in subsequent generations and thus produce offspring with the disorder.

 

Kamsteeg et al. compare the characteristics of DM1 and DM2. Both are due to trinucleotide repeat expansions. However, while DM1 can present with earlier onset and increasing severity in each generation, DM2 does not exhibit this genetic anticipation.

 

Incorrect Answers

Answer 2: "Genomic imprinting" is when a disorder is linked to a parent-specific origin. An example of maternal genomic imprinting is Angelman Syndrome. An example of paternal genomic imprinting is Prader Willi. Answer 3: "Variable penetrance" is when gene characteristics are expressed in varying degrees.

Answer 4: "Genetic drift" is the chance variation in the relative frequency of a genotype within a population. Answer 5: "Trisomy" is the presence of an extra copy of a chromosome. Down Syndrome is trisomy 21, which is due to an extra copy of chromosome 21.

 

(SAE10BS.36) A new composite material is being evaluated for potential use as a prosthetic ligament. In measuring the tensile strength of this material, why is it important to consider and accurately control the loading rate? 

 

  1. Ligaments are viscoelastic.

     

  2. Ligaments are elastic.

  3. Ligaments experience creep.

     

  4. Ligaments undergo stress relaxation.

     

  5. Strain in ligaments is recoverable.

 

PREFERRED RESPONSE 1

 

Ligaments are viscoelastic, meaning that their deformation under load depends not only on the magnitude of the load but also on the rate of loading. Under higher load rates (faster loading), natural ligaments are stiffer and stronger. Therefore, it is important in such an experiment to apply a loading rate that is physiologically relevant, ie, that represents typical or worst case activities. Creep refers to deformation of a viscoelastic material over time under constant load. Stress relaxation refers to a decrease in stress over time under a constant displacement.

 

(SAE08OS.96) A patient with a bone mineral density (BMD) T-score of -2.6 would be considered to have 

 

  1. marginally decreased bone density.

     

  2. osteopenia.

     

  3. bone within normal limits for an elderly individual.

     

  4. osteoporosis.

     

  5. metastatic bone disease.

 

PREFERRED RESPONSE 4

 

According to the World Health Organization, a patient with a BMD T-score of -2.5 or lower is considered to have osteoporosis. A score between -1.0 and 2.5 is considered osteopenia. The BMD T-score provides no information about metastatic bone disease.

 

(SAE13PE.80) Plain radiographs do not provide an accurate assessment of bone mineral density (BMD) until what percentage of mineral has been lost? 

 

1 5%

 

2 20%

 

3 40%

 

4 90%

 

PREFERRED RESPONSE 3

 

Radiographic evidence of BMD loss is not apparent until 40% reduction. Osteopenia should not be ruled out based on an apparently normal mineralized bone.

 

(SAE10BS.96) An osteoprogenitor cell is expected to commit to a bone lineage in the presence of what transcription factor? 

 

  1. PPARy2

     

  2. MyoD

     

  3. Sox 9

     

  4. C/EBPa

     

  5. Runx2

 

PREFERRED RESPONSE 5

 

Mesenchymal stem cells are pluripotent cells that can differentiate into many lineages including osteoblasts, adipocytes, myoblasts, chondroblasts, and fibroblasts. Runx2 and Osx appear to be required for differentiation to osteoblastic lines. PPARy2 and

C/EBPa are transcription factors leading to adipocytes, MyoD promotes myoblasts, and Sox 9 corresponds to chondroblasts.

 

(SAE08UE.124) In a locking plate screw construct, axial forces are borne by which of the following? 

 

  1. Plate

     

  2. Screw closest to the fracture

     

  3. Screw most distal to the fracture

     

  4. Bone-plate interface

     

  5. Opposite cortex

 

PREFERRED RESPONSE 1

 

In a traditional plate system, fracture security depends on the friction between the plate and the underlying bone. Bicortical fixation will decrease the toggle and improve stability. Locking plates absorb axial forces transmitted from the screws. Such plates do not require plate compression against the bone, thus preserving periosteal blood supply.

 

(SAE13BS.83) A 45-year-old previously healthy woman has experienced weakness and fatigability for 2 months. She states she feels best in the morning, but tires easily with exertion. If she sits and rests her strength improves, but she easily tires with each activity. When her fatigue is most severe, she has double vision. Physical examination is positive for ptosis with upward gaze after 20 seconds. When she holds her arms out straight she shows good initial strength, but rapidly decreasing strength with time. What is the pathologic cause of her muscle weakness? Review Topic

 

  1. Ig antibodies at the neuromuscular (NM) junction

  2. Decreased release of acetylcholine at the NM junction

     

  3. Decrease in myelin sheath of axonal nerves with loss of NM junction

     

  4. Absence of dystrophin with excess calcium at sarcolemma

 

PREFERRED RESPONSE 1

 

The patient has myasthenia gravis, which has its onset in middle age and causes progressive weakness because of the loss of acetylcholine receptors secondary to autoimmune antibodies at the NM junction. Rest periods allow uptake of acetylcholine and initial strength, but easy fatigability. Treatment is aimed at immunomodulation; acetyl cholinesterase inhibitors often coupled with thymectomy can control symptoms. Decreased release of acetylcholine at the NM junction is the effect of a nondepolarizing drug or toxin botulinum. Patients with muscular dystrophy lack dystrophin that acts at the sarcolemma to regulate calcium channels, and onset of this condition occurs at a younger age. The decrease in myelin indicates Charcot-Marie-Tooth disease and is often seen with long axon degeneration, such as in the feet and lower legs.

 

(SAE07SM.44) Which of the following factors is most critical to the success of a meniscal allograft transplantation? 

 

  1. Accurate graft size

     

  2. Donor cell viability

     

  3. Reestablishment of the central meniscal blood supply

     

  4. Suppression of the immune response

     

  5. Cryopreservation of the donor graft

 

PREFERRED RESPONSE 1

 

Success of a meniscal allograft transplantation is strongly dependent on accurate graft sizing, typically within 5% of the native meniscus. Previous studies have established that donor cell viability is not mandatory for the survival of these grafts since they are replaced by the recipient’s cells (at least peripherally) within several weeks. Thus,

cryopreservation of the graft to ensure cell viability is not necessary. There is a limited immune response to musculoskeletal allografts; therefore, immunosuppression, as is required for visceral organ transplantation, is not indicated.

 

(SAE12TR.33) Which of the following clinical scenarios represents the strongest indication for locked plating technique in a 70-year-old woman? 

 

  1. Segmentally comminuted ulnar fracture

     

  2. Simple diaphyseal fracture of the humerus

     

  3. Transverse midshaft displaced clavicle fracture

     

  4. Periprosthetic femur fracture distal to a well-fixed total hip arthroplasty

     

  5. Schatzker 2 fracture of the tibia with severe joint depression and comminution

 

PREFERRED RESPONSE 4

 

Locking screw fixation is a relatively new option in the armamentarium of orthopaedic surgeons treating fractures. The understanding of the biomechanics, implications to healing, and optimal indications and surgical techniques is still in evolution. A periprosthetic proximal femur fracture with a stable prosthesis is best treated with open reduction and internal fixation with locking proximal fixation with or without cerclage cables. Diaphyseal fractures treated with compression plating or bridge plating can be treated well with conventional implants unless osteoporosis is severe. An AO/OTA B-type partial articular fracture is also better suited to standard buttress plating with periarticular rafting lag screws. Locking fixation is not always required for a transverse displaced midshaft clavicle fracture.

 

(SAE13HK.33) What is the plasma half-life of warfarin? 

 

  1. 1 to 2 hours

  2. 4 to 6 hours

     

  3. 12 to 18 hours

     

  4. 36 to 42 hours

 

PREFERRED RESPONSE 4

 

Warfarin, which is dosed daily, can take 72 to 96 hours to reach therapeutic levels. It has a plasma half-life of 36 to 42 hours. Low-molecular heparins have a plasma half-life of 4 to 5 hours, and fondaparinux has a half-life of 17 to 21 hours. Warfarin will not affect the International Normalized Ratio (INR) until 2 to 3 days after it is given. Patients on chronic warfarin therapy should have treatment stopped 3 to 5 days before elective surgery to allow the INR to normalize.

 

(SAE10PE.39) The use of bisphosphonates in children with osteogenesis imperfecta is becoming more widely accepted as treatment to improve quality of life and to decrease the risks of fracture. What is the mechanism by which bisphosphonates work? 

 

  1. Inhibits osteoclasts

     

  2. Stimulates osteoblasts

     

  3. Increases gastrointestinal absorption of calcium

     

  4. Decreases renal excretion of calcium

     

  5. Acts as a transcription factor to increase production of type I collagen

 

PREFERRED RESPONSE 1

 

The mechanism by which bisphosphonates act is by inhibiting osteoclasts. One mechanism of bisphosphonates is to cause osteoclast apoptosis. Another mechanism of bisphosphonates is to disrupt the cytoskeleton of osteoclasts, resulting in loss of the ruffled border. The uncoupling of bone resorption and bone formation with decreased bone resorption results in increased bone mineralization. This translates into fewer fractures in patients with osteogenesis imperfecta and improved quality of life.

 

(SAE07HK.61) Familial (Leiden) thrombophilia is of importance in joint arthroplasty because of an abnormality in the clotting cascade. Which of the following statements best describes the condition? 

 

  1. It is a disease caused by an abnormality of platelets that leads to increased blood clotting.

     

  2. It is a disease caused by an abnormality of vascular endothelium that leads to increased blood clotting.

     

  3. It is a disease caused by an abnormality of hepatic metabolism that leads to decreased production of factor V and decreased blood clotting.

     

  4. It is a disease caused by an abnormality of factor V that leads to decreased inactivation of factor Va by activated protein C (aPC) and increased blood clotting.

     

  5. It is a familial, genetic disease that requires placement of a Greenfield filter in all individuals who have the abnormality, prior to surgery.

 

PREFERRED RESPONSE 4

 

Factor V Leiden is a disease caused by an abnormality of factor V in which a single amino acid substitution of glutamine for arginine in the protein C cleavage region leads to decreased inactivation of factor V and thus a greater tendency to form clots. More than half of all individuals with Factor V Leiden will develop deep venous thrombosis in the presence of a single additional risk factor such as long bone fracture or total joint arthroplasty.

 

(OBQ13.263) An 82-year-old osteoporotic woman undergoes total hip arthroplasty for osteoarthritis. During implant trialing, a crack is heard. Intraoperative fluoroscopy reveals a long, spiral fracture of the distal femur. The fracture is reduced and fixed with an anatomic locking plate. The rest of the total hip arthroplasty proceeds uneventfully. Following surgery the surgeon has a meeting with the family and

apologizes and provides full disclosure, accepts responsibility, provides a detailed explanation as to what happened, and gives reassurance that steps will be taken to prevent recurrences. This communication approach will most likely 

 

  1. Lead to lower patient satisfaction rates

     

  2. Lead to higher patient satisfaction rates

     

  3. Lead to higher likelihood the patient will take legal action against surgeon

     

  4. Lead to higher likelihood the patient will change physicians

     

  5. Prevent any legal action

 

PREFERRED RESPONSE 2

 

The surgeon has provided all the elements of good communication with the patient and family. His actions will lead to improved patient satisfaction, compared to nondisclosure and attempting to push or shirk responsibility.

 

In the event of a medical error or adverse event, effective communication with the patient and family is necessary. Effective communication should comprise: an apology, full disclosure (an explanation of what happened), accepting responsibility, and corrective steps that will be taken to prevent recurrence.While accepting responsibility is integral to the explanation process, it is different from accepting blame.

 

Mazor et al. found patients would more likely change physicians and seek legal advice in situations with a life-threatening outcome where physicians chose nondisclosure. They recommend full disclosure, acceptance of responsibility, an apology, detailed explanations, and assurances that steps will be taken to prevent recurrences will result in positive outcomes in terms of patient satisfaction, trust, and emotional response.

 

MacDonald et al. addressed medical errors in an editorial. Besides full disclosure, they feel that an apology is necessary. This includes an acknowledgement of the event and one’s role in the event, and a genuine expression of regret. Apology laws exist to to reduce concerns about legal implications of disclosure and apology.

 

Incorrect Answers:

Answer 1: This approach will not lead to diminished satisfaction. Answers 3 and 5: This approach will not lead to increased risk of legal action, but it may not prevent litigation under some circumstances. Answer 4: Full disclosure may decrease the likelihood of patients changing physicians.

 

(OBQ14.176) Regarding bone erosion in rheumatoid arthritis, which of the following statements is true? 

 

  1. Interference with Wnt signalling may reduce bone erosion

     

  2. TNF and IL-6 blockade leads to slowing of bone erosion

     

  3. The extent of bone erosion is independent of the extent of synovitis

     

  4. M-CSF and RANKL stimulate bone resorption by synovial fibroblasts

     

  5. The presence of serum anti-citrullinated protein antibodies is predictive of the extent of synovitis but not bone erosion

 

PREFERRED RESPONSE 2

 

TNF, IL1 and IL-6 receptor blockade helps to slow/arrest bone erosion in RA and is also effective in reducing synovitis.

 

Cytokines TNF, IL-1 and IL-6 are key players in RA. TNF stimulates migration of osteoclast precursors from the bone marrow into the periphery, and stimulates expression of surface receptors to facilitate differentiation. In the joint, M-CSF and RANKL stimulate differentiation towards osteoclasts. Final differentiation into bone-resorbing osteoclasts is achieved following contact with the bone surface.

 

Schett et al. reviewed bone erosions in RA. They state that the main triggers of bone erosion are synovitis, RANKL, and anti-citrullinated protein antibodies. In RA, there is an abundance of osteoclasts in bone erosions, but a paucity of mature osteoblasts, suggesting the presence of molecules that block osteoblast differentiation.

 

Ideguchi et al. investigated whether repair of erosions occurs in patients with rheumatoid arthritis (RA) treated with conventional disease-modifying anti-rheumatic drugs (DMARDs). They detected repair of erosions in 10.7% of RA patients treated with DMARDs. They recommend the use of DMARDs to reduce disease activity and thus reduce erosions.

 

Illustration A shows the action of antirheumatic drugs on osteoclast differentiation and bone erosion.

 

Answer 1: Interference with Wnt signalling impairs bone formation, thus INCREASING bone erosion. The Wnt signaling pathway is a bone anabolic pathway. TNF induces expression of Dickkopf-related protein 1( Dkk-1) in synovial fibroblasts. Dkk-1 interferes with Wnt signalling and blocks new bone formation. Other Wnt antagonists Frizzled-related protein-1 and sclerostin also inhibit bone formation and inhibit repair of bone erosion.

Answer 3: The greater the synovitis, the more extensive the bone erosive process. Some anti-rheumatic drugs (steroids, methotrexate, leflunomide) are bone sparing simply because they reduce synovitis. Answer 4: M-CSF and RANKL stimulate osteoclast differentiation from monocyte-lineage cells. Blockage of RANKL by denosumab slows bone erosion (but does not retard inflammation).

Answer 5: Anti-citrullinated protein antibodies (ACPA) are produced by plasma cells and can appear long before the onset of synovitis, and independently predicts bone erosion. ACPA bind to citrullinated vimentin in osteoclasts, stimulating TNF release, and enhancing differentiation into mature osteoclasts.

 

(OBQ13.117) A 35-year-old patient is involved in a motor vehicle accident and sustains multiple fractures including a closed comminuted proximal meta-diaphyseal tibia fracture. The surgeon is considering bridge plating the fracture using a minimally invasive approach. Which of the following is true regarding bridge plating? Review Topic

 

  1. A locked plate construct (locked screws) or hybrid construct (locked and non-locked screws) is necessary.

     

  2. Periosteal stripping is performed through two incisions proximal and distal to the fracture.

     

  3. Bridge plating is performed following direct reduction of the fracture.

     

  4. AO Type A diaphyseal fractures are best treated with this technique.

     

  5. Bridge plating with a long working length creates a flexible, axially stable construct.

 

PREFERRED RESPONSE 5

 

In bridged plating, only the most proximal and distal screw holes are filled. This creates a flexible, axially stable construct.

 

Bridge plating is applicable to all long-bone fractures with complex fragmentation and where intramedullary nailing or conventional plate fixation is not suitable. The construct preserves the blood supply to the fracture fragments as the fracture site is undisturbed during the operative procedure. It provides RELATIVE stability, allowing for some motion at the fracture site, leading to callus formation and secondary bone healing. The construct is FLEXIBLE because of increased distance between the 2 screws closest to the fracture (long working length), allowing for stress distribution and permitting more motion at the fracture site. The construct is also

AXIALLY STABLE because the plate acts as an extramedullary splint and resists axial compression.

 

Livani et al. advocate using an anterior or antero-lateral approach for minimally invasive plating of the humerus. They recommend that distal access is obtained first, allowing identification of the lateral antebrachial cutaneous nerve. For distal fractures, they recommend extending the plate down to the lateral column.

 

Apivatthakakul et al. defined minimally-invasive plate osteosynthesis (MIPO) danger zones from the lateral epicondyle. They found the musculocutaneous nerve averaged 18-43% of the humeral length, the danger zone for the radial nerve averaged 36-59% of the humeral length, and the most dangerous screws that penetrated or touched the radial nerve lay 47-53% of the humeral length.

 

Illustration A shows a distal tibia fracture. Illustration B shows radiographs 5 months after bridge plating of this fracture. There is callus formation, characteristic of indirect bone healing.

 

Answer 1: Locked plates are not necessary for bridge plating. Conventional plate/screws may be used. Answer 2: Bridge plating through a minimally invasive approach avoids periosteal stripping and the plate lies in a submuscular location. It is especially important where comminution is present and preservation of tenuous periosteal blood supply is critical. Answer 3: Bridge plating is usually applied following some form of indirect reduction. Indirect reduction involves manipulating fragments into the correct position without opening the fracture site, thus minimizing damage to the blood supply. The main principle of indirect reduction is distraction. Answer 4: AO Type A simple diaphyseal fractures are best treated with intramedullary nailing (relative stability) or anatomic reduction and compression plate fixation (absolute stability).

 

(OBQ15.22) Which of the follow medications activates antithrombin III? Review Topic

 

  1. Warfarin

     

  2. Aspirin

     

  3. Rivaroxaban

  4. Dabigatran

     

  5. Heparin

 

PREFERRED RESPONSE 5

 

Activation of antithrombin (AT) III is the mechanism of action of heparin.

 

Heparin works by binding to and enhancing the ability of antithrombin III to inhibit factors IIa, III, Xa. It is metabolised by the liver. The risks associated with its use include bleeding and heparin induced thrombocytopenia (HIT). The reversal agent is protamine sulfate.

 

Brown et al. performed a pooled analysis of 14 randomized controlled trials (RCTs) on VTE rates. They showed that VTE rates with aspirin were not significantly different than the rates for vitamin K antagonists (VKA), low molecular weight heparins (LMWH), and pentasaccharides. They concluded that aspirin may be used for VTE prophylaxis after major orthopaedic surgery.

 

Murphy et al. reviewed VTE prophylaxis in pediatric patients. Using public health data over 10 years, 285 611 clinical encounters were reported as lower extremity orthopaedic trauma. The incidence of VTE events was 0.058 %. Adolescents and polytrauma patients with injuries of the femur/femoral neck, tibia/ankle, and pelvis are more commonly affected.

 

Roehrig et al. first examined the coagulation enzyme Factor Xa (FXa) as a target for antithrombotic therapy. This study contributed to the development of rivaroxaban, which is commonly used as an oral antithrombotic agent.

 

Illustration A shows the interaction between anticoagulant drugs and the coagulation cascade.

 

Incorrect Answers:

Answer 1: Warfarin inhibits vitamin K 2,3-epoxide reductase, thereby limiting the production of vitamin K-dependent clotting factors (II, VII, IX, X) as well as Protein C and Protein S. Answer 2: Aspirin inhibits the production of prostaglandins and thromboxanes through irreversible inhibition of cyclooxygenase (COX, 1 and 2) and thus inhibits platelet aggregation.

Answer 3: Rivaroxaban is a direct inhibitor of factor Xa. Answer 4: Dabigatran is a direct thrombin inhibitor.

(SAE10BS.32) What collagen type is considered the major cartilage collagen, representing 90% to 95% of the total? 

 

  1. I

     

  2. II

     

  3. III

     

  4. IX

     

  5. X

 

PREFERRED RESPONSE 2

 

The major cartilage collagen is known as type II. The articular cartilage matrix also contains types V, VI, IX, X, and XI. Collagen I is the most abundant form found in tendon and bone, and collagen III is found in smaller quantities in connective tissues.

 

(SAE11OS.19) An orthopaedic surgeon makes an incision on a right knee and realizes that the patient was supposed to have a left total knee arthroplasty. The surgeon should do which of the following? 

 

  1. Leave the wound open and talk to the family immediately.

     

  2. Close the wound, abort the surgery, and talk to the patient and family when the patient is awake.

     

  3. Close the wound, complete the left knee arthroplasty, and talk to the family after the surgery is complete.

  4. Complete the surgery and talk directly to the patient the following day on rounds.

     

  5. Discuss the problem in the office the next week in a calm reassuring manner.

 

PREFERRED RESPONSE 3

The AAOS recommendation is to complete the correct surgery, repair the incorrect surgery to as close to normal as possible, and then discuss it openly with the family after the surgery is complete. Prompt informing is necessary. Aborting the surgery then results in the patient requiring a second anesthesia and surgical time needlessly.

 

(OBQ13.90) A 55-year-old woman has T-score -2.0 at the femoral neck. According to the World Health Organization Fracture Risk Assessment Tool (FRAX), she has a ten-year probability of sustaining a hip fracture of 1.5% and a ten-year probability of sustaining a major osteoporotic fracture of 8.9%. Which of the following statements is true regarding her antiresorptive therapy management? 

 

  1. Antiresorptive therapy should be started based on her T-score

     

  2. Antiresorptive therapy should be started based on her risk of hip fracture alone

     

  3. Antiresorptive therapy should be started based on her risk of major osteoporotic fracture alone

     

  4. Antiresorptive therapy should not be started

     

  5. Antiresorptive therapy should be started based on her risks of both hip fracture and major osteoporotic fracture

 

PREFERRED RESPONSE 4

 

This patient has osteopenia. Assessment by FRAX shows that ten-year risk of hip fracture is less than 3% and her ten-year risk of major osteoporosis-related fracture is less than 20%. Therefore, antiresorptive therapy is not indicated at this time.

 

According to the 2008 National Osteoporosis Foundation guidelines, pharmacologic treatment for osteoporosis should be considered if patients are postmenopausal women or men greater than 50 years old AND meet one of the following criteria: (1) they have a prior hip or vertebral fracture, (2) they have a T score -2.5 or less at the femoral neck or spine, (3) they have a T score between -1.0 and -2.5 at the femoral neck or spine AND a 10-year risk of hip fracture greater than 3% or 10-year risk of major osteoporosis-related fracture greater than 20%.

 

FRAX (World Health Organization Fracture Risk Assessment Tool) calculates 10-year risk of fracture based on the following variables: age, sex, race, height, weight,

BMI, history of fragility fracture, parental history of hip fracture, use of oral glucocorticoids, secondary osteoporosis and alcohol use to calculate 10-year risk of fracture.

 

Unnanuntana et al. reviewed the assessment of fracture risk. Besides FRAX score and T-score, they discussed biochemical markers of bone formation and resorption, which are useful for monitoring the efficacy of antiresorptive / anabolic therapy, and may help identify patients at high risk for fracture.

 

Ekman et al. reviewed the role of the orthopaedic surgeon in minimizing mortality and morbidity associated with fragility fractures. The surgeon should consider prescribing appropriate medications, physical therapy, assessing fall risk and preventing falls and changing lifestyle factors (exercise, smoking and alcohol).

 

Illustration A shows the FRAX online tool (http://www.shef.ac.uk/FRAX/tool.aspx?country=9). Illustration B shows the clinical risk factors considered in FRAX calculation.

 

Incorrect Answers:

Answer 1: T-score of -2.0 is not an indication for initiating treatment. Answers 2, 3, 5: Her FRAX score does not show a risk of hip or osteoporosis-related fracture high enough to be an indication for initiating treatment.

 

(SAE07SM.68) A player on a professional football team sustains a knee injury and is diagnosed with an anterior cruciate ligament rupture. When employed as the team physician, your ethical obligation is to inform 

 

  1. the player but not the team.

     

  2. the team but not the player.

     

  3. neither the team nor the player.

     

  4. both the team and the player.

     

  5. the team, the player, and the media

 

PREFERRED RESPONSE 4

When you are employed as a team physician, you are obligated to inform the players and the team organization of all athletically relevant medical issues. This differs significantly from the normal rule of patient confidentiality. If the player came to see you and you were not the team physician, you may not inform the team unless the player so desires. As the team physician, you are not obligated to inform the media.

 

(SAE08OS.62) New computer scanning technology for socket design has achieved which of the following improvements in amputee care? 

 

  1. Improved socket fit

     

  2. Reduced skin breakdown from excessive pressure on the residual limb

     

  3. Reduced the time of socket fabrication

     

  4. Reduced the cost of prosthetic devices

     

  5. Decreased pain at the limb socket interface

 

PREFERRED RESPONSE 3

 

One of the new technologies available to the prosthetist is the use of digital scanners to obtain a model of the residual limb. Typically, a digital image of the residual limb is obtained by either running a wand over the surface anatomy, allowing the computer program to record the shape, or by using a ring-type scanner that does not make contact with the residual limb but records the shape by bouncing a laser beam off the limb. In either instance, the end result is a digital image that can then be viewed and modified on the computer to improve the weight-bearing aspects of a socket shape or take pressure off sensitive bony areas. The final shape is then carved out of a foam block for use in the socket fabrication process. Computer-aided socket design is still dependent on input from the prosthetist, and therefore does not represent an improvement in overall socket fit. Digital scanning works well on specific types of devices, such as transfemoral sockets, where there is more soft tissue relative to bone. The scanner does not have the ability to detect the density of tissue beneath the scanned surface. Heterotopic ossification and other anomalies will be missed if a thorough examination of the underlying anatomy is neglected. Therefore, the advantages of using scanning technology are for producing and fabricating a socket shape quickly, storing the shape digitally for future use, and increasing efficiency with a plasterless facility.

 

 

(SAE13BS.37) A 4-year-old boy has bilateral genu varum and is in the fifth percentile for height for his age. A younger sister has less severe genu varum. Radiographs reveal physeal cupping and widening on both the distal femur and proximal tibia. Laboratory studies show sodium 145 mEq/L (reference range, 136-142 mEq/L), potassium 4.0 mEq/L (reference range, 3.5-5.0 mEq/L), calcium 9.0 mg/dL (reference range, 8.2-10.2 mg/dL), phosphorous 2 mg/dL (reference range, 4-6.5 mg/dL), vitamin D 50 ng/mL (reference range, 30-100 ng/mL), and urine phosphorus 2 g/24-hour collection (reference range, 0.4-1.3 g). What effect would treatment with only Calcitriol (1,25 dihydroxy vitamin D3) have? 

 

  1. Restore normal limb alignment

     

  2. Restore normal limb alignment and height

     

  3. Have no effect on limb alignment and height

     

  4. Stabilize degree of genu varum, but not improve limb alignment

 

PREFERRED RESPONSE 3

 

This patient has familial hypophosphatemic rickets, a vitamin D-resistant form of rickets that is an X-linked inherited disorder. Patients are short (< 10th percentile). Varus occurs both in the distal femur and proximal tibia. Patients have increased urinary excretion of phosphorus, leading to hypophosphatemia. Calcium levels are within defined limits and vitamin D levels can be normal as well. Treatment should include phosphate and 1,25 dihydroxy vitamin D3 (calcitriol). Phosphate administration increases the plasma concentration, which lowers plasma ionized calcium concentration and further reduces plasma calcitriol concentration (removal of hypophosphatemic stimulus). Secondary hyperparathyroidism results because of both hypocalcemia and removal of the normal inhibitory effect of calcitriol on parathyroid hormone (PTH) synthesis. Elevated PTH levels will increase urinary phosphate excretion, defeating the aim of oral therapy. Addition of calcitriol is necessary to increase the intestinal absorption of calcium and phosphate to prevent secondary hyperparathyroidism. Massive doses of vitamin D alone can restore normal radiographic appearances to the epiphyses, but normal growth is not restored unless phosphate replacement is adequate.

(OBQ14.198) What is the main biologic effect of aggrecan in cartilage? Review Topic

 

  1. Extracellular matrix protein involved in the organization of collagen

     

  2. Proteoglycan involved in the hydrophilic behaviour of cartilage

     

  3. Cartilage matrix protein that plays a role in cartilage tissue organization

     

  4. Collagen component responsible for stability

     

  5. Non-collagenous extracellular matrix protein that regulates chondrocyte proliferation

 

PREFERRED RESPONSE 2

 

Aggrecan binds hyaluronic acid to attract water, which accounts for its hydrophilic property.

 

Aggrecan is the predominant proteoglycan in cartilage. It contains a large number of negatively charged sequences that attract water called sulfated glycosaminoglycan (GAG) chains. Its the N-terminal globular domain of aggrecan that binds hyaluronan to form huge aggregates. Together with its chondroitin sulfate chains, they help to create a hydrophilic viscous gel that decreases the coefficient of friction as well as to help absorb compressive loads.

 

Ulrich-Vinthe et al. reviewed the biology of articular cartilage. They report that matrix metalloproteinases and aggrecanases play a major role in aggrecan degradation and their production is upregulated by mediators associated with joint inflammation and overloading.

 

Illustration A shows a depiction of the function of aggrecan in articular cartilage. In the relaxed state, the aggregates draw water into cartilage. With compressive loads, the water is displaced to cushion the load. Upon removal of the load, the water content is restored.

 

Incorrect Answers:

Answer 1: This describes the biological effect of Decorin. Answer 3: This describes the biological effect of Matrilin 1. Answer 4: This describes the biological effect of Type IX collagen Answer 5: This describes the biological effect of Cartilage oligomeric matrix protein.

(SAE11PA.94) What is the primary problem in rickets osteomalacia? 

 

  1. Defect in the zone of proliferation within the physis

     

  2. Defect in type I collagen

     

  3. Defect in the ext-1 gene

     

  4. Low level of calcium

     

  5. Production of dysplastic fibrous bone

 

PREFERRED RESPONSE 4

 

Rickets is a disorder of bones in children that results from decreased calcium available in the blood resulting in poor mineralization of bone that can lead to fractures and deformity. The most common cause of rickets is from vitamin D deficiency but it can also be caused by poor nutrition or gastrointestinal disease that results in poor calcium absorption such as celiac disease or severe diarrhea from other causes. Rickets is not primarily a physeal disorder. Osteogenesis imperfecta is caused by a defect in type I collagen. A defect in the ext-1 gene is often seen in patients with multiple hereditary exostoses. Fibrous dysplasia also can result in bone deformity and fractures due to production of dysplastic fibrous bone but is not caused by calcium or vitamin D deficiency.

 

(SAE13BS.43) A 45-year-old man develops a nonunion after undergoing external fixation of an open femoral shaft fracture. The procedure is revised with an open reduction and internal fixation, all intraoperative cultures are negative, and specimens from the fracture site are sent for biopsy. Histological analysis most likely will reveal 

 

  1. positive gram stain and polymorphonuclear cells that are too numerous to count.

     

  2. bland-appearing cartilage callus.

     

  3. Haversian remodeling.

     

  4. lamellar bone.

PREFERRED RESPONSE 2

 

This patient has a nonunion and intraoperative cultures that are negative. The external fixation technique would be expected to heal the fracture through an endochondral mechanism. Bland-appearing cartilage callus fits with the nonunion and with the expectation for cartilage present in the tissues taken from the failed endochondral healing site. Positive gram stains and polymorphonuclear cells that are too numerous to count suggest an active infection. Haversian remodeling and lamellar bone would be expected to be seen in fully healed and remodeling bone.

 

(OBQ15.84) Which of the following treatments for osteoporosis is a direct inhibitor of RANK ligand (RANK-L)? 

 

  1. Romosozumab

     

  2. Zoledronic acid

     

  3. Denosumab

     

  4. Teriparatide

     

  5. Blosozumab

 

PREFERRED RESPONSE 3

 

Prolia, or denosumab, is a newly approved drug used to treat osteoporosis and has a mechanism of action similar to osteoprotegerin (inhibits binding of RANKL to RANK).

 

RANKL (Receptor activator of nuclear factor kappa-B ligand) is a key molecule for osteoclast differentiation and activation. Inhibition of RANKL activity with anti-RANKL antibody reduces osteoclastogenesis, resulting in inhibition of bone resorption.

 

Capozzi et al. author a review article on denosumab. They state the medication confers improved bone mineral density and prevents new fragility fractures similar to alendronate. However, denosumab presents less risk of atypical femoral fractures and

osteonecrosis of the jaw.

 

Yasuda et al. present a review that details the creation of three elegant animal models to mimic metabolic bone disease and how the animal models can create a template to help cure human metabolic bone disease. These enable modeling of osteoporosis, hypercalcemia, and osteopetrosis by treating normal mice with soluble RANKL (sRANKL), adenovirus expressing sRANKL, and anti-mouse RANKL neutralizing antibody, respectively. They report that these animal models can be established in about 14 days using normal mice.

 

Illustration A demonstrates the mechanism of action of bisphosphonates and denosumab.

 

Incorrect Answers:

1: Romosozumab is the first humanized anti-sclerostin monoclonal antibody that has been demonstrated to increase bone formation. 2: Zoledronic acid (Reclast) is a nitrogen containing bisphosphonates that inhibits osteoclast resorption by inhibiting the enzyme farnesyl diphosphate synthase. 4: Teriparatide (Forteo) comprises the first 34 amino acids of the 84 amino acid parathyroid hormone (PTH) and can reproduce the primary effects of PTH by activating adenyl cyclase. 5: Blosozumab is an investigational monoclonal anti-sclerostin antibody showing osteoanabolic properties with the potential to improve clinical outcomes in patients with osteoporosis.

 

(OBQ14.148) An 76-year-old woman falls from standing and sustains the injury shown in Figure A. Her most recent T score was -1.9, 3 months prior to presentation. If labwork were performed, which values would be consistent with her bone density score? 

 

 

  1. High parathyroid hormone, normal calcium, normal alkaline phosphatase, low phosphorus, low vitamin D

  2. High parathyroid hormone, low calcium, high alkaline phosphatase, low phosphorus, low vitamin D

     

  3. Normal parathyroid hormone, high calcium, low alkaline phosphatase, low phosphorus, low vitamin D

     

  4. Low parathyroid hormone, normal calcium, normal alkaline phosphatase, low phosphorus, low vitamin D

     

  5. Low parathyroid hormone, low calcium, high alkaline phosphatase, low phosphorus, low vitamin D

 

PREFERRED RESPONSE 2

 

In osteopenia, elevated parathyroid hormone (PTH), high alkaline phosphatase as well as low calcium (Ca), phosphorus and vitamin D would be expected.

 

In the setting of osteopenia/osteoporosis, there is a positive feedback to increase PTH in response to low serum calcium levels. In response, there is a corresponding increase in alkaline phosphatase and decrease in phosphorous and circulating vitamin D levels.

 

Fraser writes a concise, yet thorough synopsis on primary and secondary hyperparathyroidism. In the article, the summary regarding osteopenia/osteoporosis (typically a state of hypovitamin D) begins by stating an initial state of decreased ionised calcium, which increases PTH, results in 3 primary effects: an attempt to increase gut absorption of Ca, mobilize Ca from the bone via osteoclasts and activate vitamin D at the kidney (1,25-vitamin D).

 

Figure A exhibits a left femoral neck fracture, which is a fragility fracture associated with poor bone density. Illustration A is a figure from Fraser's article exhibiting the feedback loop from the hypothalamus, pituitary, adrenal/glandular axis.

Incorrect answers:

Answers 1,3: These states represents a potential primary parathyroidism which may caused by an endocrine disorder or tumor. Answers 4,5: These states represent a mismatch in the hypothalamic, pituitary, adrenal/gland axis, which may be caused by either a primary parathryoid tumor, or a secondary endocrine mismatch disorder.

 

(SAE10BS.71) A study is proposed in which 50 patients with osteonecrosis of the knee are compared with 23 patients without osteonecrosis in terms of their alcohol consumption levels. This is an example of what type of study? 

 

  1. Case control

     

  2. Cohort

     

  3. Cross-sectional

     

  4. Randomized

     

  5. Longitudinal

 

PREFERRED RESPONSE 1

 

In a case control study, all the subjects are selected based on whether they have (cases) or do not have (controls) the disease or outcome of interest. Case control studies are retrospective as they always look back to see how a certain risk factor may be different between the two groups. The main aspect of a cross-sectional study is that it is designed to look at a representative sample of the entire population of interest at a single point in time. Longitudinal studies follow groups of subjects over a period of time. A cohort study follows a particular group in relation to an event, studying them at intervals in time and uses objective outcome criteria. In a randomized controlled study, subjects are divided randomly into control and experimental groups to balance both the known and unknown differences between the groups.

 

(SAE13BS.53) What method of spinal fixation requires the largest force to disrupt the bone-implant interface? 

 

  1. Sublaminar cables

     

  2. Laminar hooks

     

  3. Pedicle hooks

     

  4. Pedicle screws

 

PREFERRED RESPONSE 4

 

Pedicle screws have been established to produce a superior bone-implant interface in the nonosteoporotic spine according to numerous studies, but, interestingly, clinical outcomes using the varied implants have not been different.

 

(SAE13BS.80) What dominant intracellular proteins become directly phosphorylated as a result of bone morphogenetic protein (BMP) binding to its receptors? Review Topic

 

  1. Myc

     

  2. SMADs

     

  3. Beta-catenin (ß-catenin)

     

  4. Adenylate cyclase

 

PREFERRED RESPONSE 2

 

BMP proteins, which are part of the larger transforming growth factor-beta super family, bind to serine/threonine receptors on the cell surface. This binding causes phosphorylation of SMAD proteins, which in turn forms a complex that enters the nucleus and initiates the transcription of several genes involved in osteoblastic

differentiation. Adenylate cyclase is a transmembrane protein that acts intracellularly to activate the G protein pathways. Myc is a proto-oncogene that encodes for a transcription factor involved in numerous cell-activation pathways, but is not directly phosphorylated by BMP receptor. ß-catenin is an intracellular molecule that plays a key role in the Wnt signaling pathway. This pathway is also involved in osteoblastic differentiation, but differs in target proteins within the cell.

 

(SAE11OS.27) A total knee arthroplasty is recommended to a mentally competent 68-year-old woman who has disabling knee pain caused by degenerative arthritis. Her son has researched the procedure on the internet and prefers the Acme Female Knee for his mother. You have designed the Axis Woman's Knee, for which you receive royalties, and use it exclusively. Which of the following ethical principles takes precedence in guiding her treatment? 

 

  1. Informed consent

     

  2. Patient autonomy

     

  3. Fiduciary responsibility

     

  4. Physician paternalism

     

  5. Justice

 

PREFERRED RESPONSE 1

 

Informed consent incorporates a number of ethical principles relevant to this case. The fundamentals of medical ethics include nonmaleficence, beneficence, autonomy, and justice. The patient is competent and capable of exercising her autonomy in choosing the Acme Female Knee. She also depends on her physician's paternalism and knowledge in looking out for her best interests, which in his opinion, may be use of the Axis Woman's Knee. The physician has a fiduciary responsibility to inform the patient that he has a financial interest in the implant system he recommends. A thorough informed consent will respect the patient's autonomy, explain the rationale for the physician's recommendation, and notify the patient that there may be a perceived conflict of interest. The ethical principle of justice has no relevance in this case.

 

(SAE13BS.50) Decreased sun exposure leads to decreased bone health via what mechanism? 

 

  1. Impaired ability of the liver to hydroxylate cholecalciferol

     

  2. Impaired ability of the kidneys to hydroxylate cholecalciferol

     

  3. Impaired parathyroid hormone (PTH) production by the parathyroid glands

     

  4. Impaired absorption of calcium by the gastrointestinal (GI) tract

 

PREFERRED RESPONSE 4

 

Ultraviolet light from the sun is needed for skin cells to transform 7-dehydrocholesterol into cholecalciferol (vitamin D3). Once in the cholecalciferol form, it will then go to the liver and get hydroxylated into 25-hydroxyvitamin D and then to the kidneys to become 1,25-dihydroxyvitamin D. Lack of sunlight does not influence the ability of the liver or kidneys to perform hydroxylation, but it does affect the ability of the skin to create cholecalciferol, which in turn means no 1,25-vitamin D. Lack of 1,25 vitamin D has a direct impact on the GI tract's ability to absorb calcium because the GI tract is an end organ affected by active vitamin D. The absence of 1,25 vitamin D would lead to increased levels of PTH, not decreased production of PTH.

 

(SAE13BS.41) What mechanism is most likely responsible for the initiation of mechanical failure seen at the midstem modular junction of modular revision hip stems? 

 

  1. Fretting fatigue

     

  2. Etching

     

  3. Crevice corrosion

     

  4. Abrasive wear

PREFERRED RESPONSE 1

 

The junction fracture is initiated in a fretting-fatigue mechanism and completed by a bending moment. Fretting fatigue occurs when contacting components experience cyclic loads while small oscillatory motion occurs between them. This increases tensile and shear stress, leading to small flaws that result in crack propagation. In the referenced study no evidence of corrosion was seen because there was no etching, pitting, corrosion products, or chloride formation. Etching is a finding seen in corrosion. Abrasive wear occurs when a rough surface glides against a softer surface. It is generally seen in polyethylene wear from bone or cement in third-body wear. Risk factors for stem breakage include high body mass index and lack of proximal bone support at the modular stem-junction area.

 

(OBQ15.61) An increase in advanced glycation end-products (AGEs) is characteristic of which of the following clinical conditions and results in which pathologic process? 

 

  1. Increased AGES is characteristic of aging articular cartilage and results in decreased articular cartilage stiffness

     

  2. Increased AGES is characteristic of osteoarthritis and results in increased articular cartilage stiffness

     

  3. Increased AGES is characteristic of unresurfaced patellar cartilage after total knee arthroplasty and results in articular cartilage thinning

     

  4. Increased AGES is characteristic of rheumatoid arthritis and results in synovial thickening

     

  5. Increased AGES is characteristic hemarthrosis and results in articular cartilage staining

 

PREFERRED RESPONSE 2

 

Advanced glycation end-products (AGEs) are found in aging and osteoarthritis (OA) and result in increased articular cartilage stiffness and increased brittleness.

 

AGEs are produced from spontaneous nonenzymatic glycation of proteins when sugars (glucose, fructose, ribose) react with lysine or arginine residues. Because of the low turnover, cartilage is susceptible to AGEs accumulation. The accumulation of AGEs has been thought to play a role in the development of OA of the knee and ankle.

Li et al. reviewed age-related changes in cartilage. They state that with aging, there is excessive collagen cross-linking increases cartilage stiffness, while shortening/degradation of aggrecan leads to loss of sugar side chains and water-binding ability, while increased levels of AGEs are associated with a decline in anabolic activity. There is also increased chondrocyte death and/or apoptosis.

 

Anderson et al. reviewed the relationship between osteoarthritis and aging. They state that knee cartilage thins with aging (especially on the femoral and patellar sides, suggesting a gradual loss of cartilage matrix. AGEs formation leads to modification of type II collagen by cross-linking of collagen molecules, increasing stiffness and brittleness and increasing susceptibility to fatigue failure.

 

Incorrect Answers:

Answer 1: While AGEs are found in aging cartilage and predispose to OA formation, increased cartilage stiffness (rather than softening) results. Answer 3: Unresurfaced patellar cartilage will show similar changes to the other compartments prior to resurfacing. Thinning is a result of loss of cartilage matrix, rather than accumulation of AGEs. Answer 4: AGEs are characteristic of OA (rather than RA). It is found in articular cartilage (rather than synovium). Answer 5: AGES are not characterstic of hemarthrosis and do not result in cartilage staining.

 

(SAE10BS.86) During the first stage of osteoarthritis, which of the following processes predominates? 

 

  1. Loss of articular cartilage proliferative response

     

  2. Release of mediators by chondrocytes that stimulate anabolic and catabolic response

     

  3. Decreased proteoglycan and aggrecan concentration and increased water content

     

  4. Decline in chondrocyte anabolic response

     

  5. Increased synthesis of matrix macromolecules

 

PREFERRED RESPONSE 3

 

Articular cartilage degeneration and ensuing osteoarthritis can be divided into three stages. In the first stage, the water content increases and proteoglycan aggregation and

aggrecan concentration are both decreased. Increased water content tends to decrease the stiffness of the matrix, rendering cartilage tissue more susceptible to further mechanical damage. In the second stage, chondrocytes detect tissue damage and respond by releasing mediators to increase proliferation. Clusters or clones of proliferating chondrocytes are hallmark of the response to articular degeneration. In the third stage, both the proliferative response and anabolic activity are decreased. In this stage, the loss of articular cartilage is more evident and leads to clinical signs of degenerative joint disease.

 

(SAE08PA.101) An orthopaedic surgeon frequently uses hip and knee prostheses from a specific manufacturer. The surgeon becomes acquainted with the manufacturer’s representative who provides the support for these prostheses in the hospital. They develop a personal relationship outside of work through a common interest in sailing. Together they become interested in buying a sailboat. The manufacture’s representative suggests a partnership in a boat costing $200,000. The manufacture’s representative would purchase a 90% interest and the surgeon a 10% interest in the boat. There would be no restrictions on use of the boat by the surgeon. What should the orthopaedic surgeon do? 

 

  1. Accept the offer because it would allow the physician to have a boat and enjoy a hobby of sailing with a minimal financial outlay.

     

  2. Accept this offer because it would have nothing to do with the orthopaedic practice and because the physician feels strongly that it would have no influence on practice decisions.

     

  3. Reject the offer because it represents an unacceptable conflict of interest.

     

  4. Accept the offer but disclose the relationship to patients when using the representative’s products.

     

  5. Accept the offer but only with a equal interest arrangement in the purchase and use of the boat

 

PREFERRED RESPONSE 3

 

Rejecting this proposal is the only appropriate course of action. Accepting it would, in essence, be receiving a huge gift from industry in the form of a sailboat. Physicians frequently assert that they are not influenced by gifts and relationships with industry representatives, but evidence is to the contrary. Such an arrangement constitutes a tremendous incentive to use the manufacturer’s products. The fact that the boat

partnership seems completely outside of the orthopaedic business relationship does not excuse it. Conflicts of interest should always be resolved and in the best interest of patient care, and in this case the best course clearly is to avoid the conflict of interest totally. An equal interest in the boat does not eliminate the conflict of interest.

 

(SAE07SM.4) The force generated by a muscle is most highly dependent on its 

 

  1. cross-sectional area.

     

  2. fiber type.

     

  3. length.

     

  4. morphology.

     

  5. level of conditioning.

 

PREFERRED RESPONSE 1

 

The cross-sectional area of a muscle determines to a great extent the force generated by the muscle. The force of a muscle contraction is controlled by the amount of myofibrils that contract; the greater the amount of contracting myofibrils, the greater the force of contraction. Fiber types have less to do with the force of contraction and more to do with the duration and speed of contraction. Muscle length affects contraction force through the Blix curve. The morphology of a muscle can affect the cross-sectional area by varying the angle of the fibers in relation to the force vector. Conditioning mostly affects duration and fatigability.

 

(SAE10BS.16) Which of the following changes occur in articular cartilage with age? 

  1. Cell numbers increase with fairly uniform distribution.

     

  2. Chondroitin sulfate concentration increases.

     

  3. Keratin sulfate concentration decreases.

     

  4. Proteoglycan synthesis decreases.

     

  5. Water content increases.

 

PREFERRED RESPONSE 4

 

Proteoglycan synthesis decreases with age, as does water content. Cell numbers of cartilage decrease compared to immature cartilage, which has a fairly uniform distribution of abundant cells. A high concentration of chondroitin 4-sulfate has been noted in immature animals, with a fairly rapid diminution in the value noted with aging accompanied by an increase in the concentration of chondroitin 6-sulfate. With advancing age, the total chondroitin sulfate concentrations fall and that of keratin sulfate increases until approximately age 30, where it levels off.

 

(SAE13BS.67) Osteoclastic bone resorption is stimulated primarily by what molecular interaction? 

 

  1. Parathyroid hormone (PTH)-osteoclasts

     

  2. Interleukin-6 (IL-6) and IL-8

     

  3. Receptor activator of nuclear factor kappa beta (RANK)-RANK ligand (RANKL)

     

  4. Osteoprotegrin (OPG)-RANKL

 

PREFERRED RESPONSE 3

 

OPG is a receptor that competitively binds with RANKL, blocking the interaction with RANK and inhibiting osteoclastogenesis. PTH, secreted by the chief cells of the parathyroid gland, is active in calcium homeostasis independent of inflammatory arthropathies. PTH increases serum calcium indirectly by binding to osteoblasts, increasing expression of RANKL and decreasing expression of OPG. The interaction of RANKL to RANK in turn stimulates osteoclast precursors to fuse, forming

osteoclasts to enhance bone resorption. The pannus of rheumatoid arthritis and monosodium urate crystals of gouty tophi have been shown to trigger release of inflammatory cytokines such as IL-6, IL-8 and tumor necrosis factor alpha. The key to osteoclastic bone resorption of inflammatory arthropathy is regulated by the interaction of RANKL, expressed in osteoblasts and activated T cells, and RANK, expressed in osteoclast progenitors and mature osteoclasts. In inflammatory arthropathy, RANKL expression is increased and OPG is reduced, resulting in increased cortical and subchondral bone.

 

(SAE10BS.68) Which of the following growth factors binds and activates the lipoprotein receptor-related protein 5/6 (LRP5/6) during bone development? Review Topic

 

  1. Transforming growth factor-beta

     

  2. Platelet-derived growth factor

     

  3. Parathyroid hormone

     

  4. Bone morphogenetic protein

     

  5. Wnt protein

 

PREFERRED RESPONSE 5

 

Wnt and Hedgehog (Hh) signaling pathways are key regulators of bone formation. Mutations in the Wnt and Hh pathways result in skeletal malformations as well as osteoarthritis. Wnts are a large family of secreted proteins (19 different members in the human genome) that transduce their signal through several pathways. The most studied of these pathways is the Wnt/beta-catenin pathway, in which Wnt protein binds to the LRP5/6 receptor at the cell surface and activates an intracellular cascade. This cascade leads to translocation of beta-catenin into the nucleus to activate transcription of genes that control osteoblast differentiation. Agents that stimulate this pathway are under investigation for treatment of osteoporosis.

(SAE10BS.22) Methicillin-resistant staphylococcus aureus (MRSA) develops its resistance to penicillinase-stable antibiotics via which of the following actions? 

 

  1. Alterations in cell wall permeability

     

  2. Genetic mutation

     

  3. Creation of a biofilm barrier

     

  4. Production of active efflux pump

     

  5. Altering peptidoglycan subunits

 

PREFERRED RESPONSE 2

 

After the introduction of penicillins, bacteria developed the ability to hydrolyze these antibiotics using B-lactamase. In response, penicillinase-stable antibiotics were developed, the first of which was methicillin, since replaced with oxacillin and nafcillin. Drug resistance to this class of antibiotics is achieved via a genetic mutation of mecA encoding an altered penicillin binding protein. The gene product of this mutation, PBPa has a low affinity for these antibiotics and cannot be inhibited by them. Altering cell wall permeability is found in resistance to tetracyclines, quinolones, and trimethoprim, as well as B-lactam antibiotics. Biofilm barriers are produced by bacteria such as salmonella. Active efflux pumps provide resistance to erythromycin and tetracycline, and altering the peptidoglycan subunit is found in resistance to vancomycin.

 

(OBQ15.55) Which of the following medications inhibits release of neurotransmitters by binding to presynaptic calcium channels? 

 

  1. Denosumab

     

  2. Sertraline

     

  3. Tramadol

     

  4. Gabapentin

  5. Linezolid

 

PREFERRED RESPONSE 4

 

Gabapentin acts by inhibiting presynaptic calcium channels, thus preventing the release of neurotransmitters.

 

Gabapentin (also known as Neurontin) is a medication that is commonly used to treat neuropathic pain. It acts by binding the alpha2delta subunit of voltage-dependent calcium channels on the presynaptic membrane. This serves to increase GABA synthesis, as well as inhibit the release of excitatory neurotransmitters. These neurotransmitters are believed to be part of the pathway leading to neuropathic pain.

 

Bennett et al. provide a review of the pharmacology of gabapentin for the use of neuropathic pain. They note effective antihyperalgesic and antiallodynic properties of gabapentin but not significant anti-nociceptive action. Among patients with neuropathic pain they found an average pain score reduction of 2.05 points on an 11 point Likert scale, which compared favorably to placebo.

 

Mehta et al. explored outcomes of gabapentin and pregabalin (Lyrica) for use in patients with spinal cord injury. Both agents were found to decrease pain and secondary conditions such as sleep disturbance. They did not directly compare these agents to other analgesic medications.

 

Guy et al. present a meta-analysis of the use of anticonvulsants (such as gabapentin) to treat pain in patients with spinal cord injury. Large effect size was seen in 4 of 6 studies looking at the effectiveness of gabapentin.

 

Illustration A show the mechanism of currently available antiepileptic drugs (AEDs) that target several molecules at the excitatory synapse. Gabapentin and pregabalin bind to the a2d subunit of voltage-gated Ca2+ channels, which is thought to be associated with a decrease in neurotransmitter release.

 

Incorrect Answers

Answer 1: Denosumab works by inhibiting the receptor activator of nuclear factor kappa beta ligand (RANK-L) Answer 2: Sertraline is a selective serotonin reuptake inhibitor (SSRI). It increases concentrations of serotonin in the synaptic cleft by inhibiting it's reuptake. Answer 3: Tramadol acts as a mu-opioid receptor agonist. Answer 5: Linezolid is a bacteriostatic antibiotic medication that acts by disrupting translation of mRNA

 

(SAE10BS.82) The nucleus pulposus of the intervertebral disk consists of chondrocyte-like cells that have a limited vascular supply and generate energy through which of the following? 

 

  1. Anaerobic glycolysis

     

  2. Kreb's cycle

     

  3. Ketosis

     

  4. Gluconeogenesis

     

  5. Lipogenesis

 

PREFERRED RESPONSE 1

 

The intervertebral disk is an avascular structure in the adult. Nucleus pulposus cells have a critical need for glucose because they obtain their energy primarily by glycolysis, even in the absence of oxygen. Disk cells do not require oxygen to remain alive but they die at low glucose levels or acidic pH. Nutrients are supplied from the blood vessels at the margins of the disk and have to traverse the cartilaginous end plate to reach the disk cells. The loss of the nutrient supply through the vertebral body will starve the cells in the disk center and may be a major factor in disk degeneration.

 

(SAE07HK.93) Which of the following substances makes up the majority by weight of the extracellular matrix for articular cartilage? 

 

  1. Keratin sulfate

     

  2. Collagen type II

     

  3. Water

     

  4. Protein

  5. Chondroitin sulfate

 

PREFERRED RESPONSE 3

 

The extracellular matrix consists of water, proteoglycans, and collagen. Water makes up the majority (approximately 65% to 80%) of wet weight; 95% of the collage is type II with much smaller amounts of other collagens, including types IV, VI, IX, X, and XI. The exact functions of these other collagens are unknown, but they are believed to be important in matrix attachment and stabilization of the diameter of collagen fibrils.

 

(SAE07PE.23) Figure 11 shows the radiograph of a 2-year-old child with marked genu varum and tibial bowing. Based on these findings, what is the best initial course of action? 

 

 

 

  1. Obtain serum phosphorous, calcium, and alkaline phosphatase levels.

  2. Obtain a scanogram to assess for limb-length discrepancy.

     

  3. Perform bilateral valgus osteotomies to correct the deformities.

     

  4. Measure the child for a varus prevention orthosis.

     

  5. Educate the family about physiologic genu varum and conduct a follow-up examination in

  6. months.

PREFERRED RESPONSE 1

 

The radiograph shows multiple wide physes, consistent with a diagnosis of rickets. A low serum phosphorous level and an elevated alkaline phosphatase level are the hallmarks in diagnosing familial hypophosphatemic Vitamin D-resistant rickets. Serum calcium is usually normal or low normal. This disease is inherited as an X-linked dominant trait and usually presents at age 18 to 24 months. The disease results from a poorly defined problem with renal phosphate transport in which normal dietary intake of vitamin D is insufficient to achieve normal bone mineralization. Renal tubular dysfunction is associated with urinary phosphate wasting. Treatment involves oral phosphate supplementation, which can cause hypocalcemia and secondary hyperparathyroidism. To prevent associated problems, high doses of Vitamin D are administered. While obtaining a scanogram may be clinically indicated in an associated limb-length discrepancy, and subsequent corrective surgery may be indicated, either of these choices would not be the first course of action. An orthosis may slow the progression of genu varum in this disorder but is less important than establishing the correct diagnosis to begin pharmacologic treatment. This amount of varum and tibial bowing far exceeds the normal limits of physiologic genu varum. Skeletal dysplasias usually are not associated with abnormal laboratory values.

 

(SAE08PA.18) It has been shown that bisphosphonate-based supportive therapy (pamidronate or zoledronate) reduces skeletal events (onset or progression of osteolytic lesions) both in patients with multiple myeloma and in cancer patients with bone metastasis. The use of biphosphonate therapy has been associated with Review Topic

 

  1. increased medical complications of treatment.

     

  2. osteonecrosis of the jaw.

     

  3. improved long-term survival rates.

     

  4. anorexia.

     

  5. decreased quality of life measures.

 

PREFERRED RESPONSE 2

The use of bisphosphonates has been recently associated with the development of osteonecrosis of the jaw. Length of exposure seems to be the most important risk factor for this complication. The type of bisphosphonate may play a role and previous dental procedures may be a precipitating factor. Bisphosphonates are a class of therapeutic agents originally designed to treat loss of bone density (ie, alendronate). The primary mechanism of action of these drugs is inhibition of osteoclastic activity, and it has been shown that these drugs are useful in diseases with propensities toward osseous metastases. In particular, they are effective in diseases in which there is clear upregulation of osteoclastic or osteolytic activity, such as breast cancer and multiple myeloma, and have developed into a mainstay of treatment for individuals with these diseases. Although shown to reduce skeletal events, there has been no improvement in patient survival.

 

(OBQ15.126) Passage of a sodium ion through a voltage-gated channel leads to which of the following? 

 

  1. Apoptosis of gram negative bacteria

     

  2. Binding of RANKL to osteoblasts

     

  3. Inhibition of micturition

     

  4. Generation of a nerve action potential

     

  5. Deposition of salts in adipose tissue

 

PREFERRED RESPONSE 4

 

Passage of sodium through a voltage-gated channel will lead to generation of a nerve action potential.

 

Voltage-gated channel are shut when the membrane potential is near the resting potential of the cell, but they rapidly begin to open if the membrane potential increases to a precisely defined threshold value. When the channels open (in response to depolarization in transmembrane voltage), they allow an inward flow of sodium ions, which changes the electrochemical gradient, which in turn produces a further rise in the membrane potential. This then causes more channels to open, producing a greater electric current across the cell membrane, and so on.

 

Lee et al. present a review article on nerve conduction and needle electromyography studies. They note that the three types of nerve conduction study are motor, sensory,

and mixed, of which motor is the least sensitive. In addition, they report that peripheral nerve entrapment initially results in focal demyelination; thus, nerve conduction velocity slows across the site. However, with radiculopathy and nerve root compression, the nerve conduction study may be normal.

 

Catterall presents a review article covering an overview of structural models of voltage-dependent activation, sodium selectivity and conductance, drug block and both fast and slow inactivation. He notes that voltage-gated sodium channels initiate action potentials in nerve, muscle and other excitable cells.

 

Illustration A is a diagram that shows the electrical recordings of an action potential, along with labels of each section of the process.

 

(SAE07PE.1) A pediatric orthopaedic surgeon refers a child to a neurologist. The neurologist’s office requests the office records of the pediatric orthopaedic surgeon. To maintain Health Insurance Portability and Accountability Act (HIPAA) compliance, what must the surgeon obtain from the parent(s) prior to sending records? 

 

  1. No additional consent needed

     

  2. Verbal approval

     

  3. Written approval

     

  4. Written approval with notarization

     

  5. Telephone consent witnessed by a nurse

 

PREFERRED RESPONSE 1

 

The privacy rules do not require an individual’s written authorization for certain permitted or required uses and disclosures of the medical records. Patient or parental authorization is not required for disclosures for certain purposes related to treatment, payment, or health care operations. Specifically, HIPAA does not require a covered entity to obtain patient authorization for many of the health care industry’s most fundamental activities such as providing care.

 

(SAE07PE.50) A 3-year-old child has bilateral genu varum and short stature. Radiographs show physeal widening and generalized osteopenia. The femora and tibiae show anterolateral bowing. Laboratory studies show low normal serum calcium values, significantly decreased serum phosphate levels, elevated alkaline phosphatase levels, and normal parathyroid hormone (PTH),and vitamin-D levels. These findings are most consistent with: 

 

  1. nutritional rickets.

     

  2. renal osteodystrophy.

     

  3. primary hyperparathyroidism.

     

  4. hypophosphatasia.

     

  5. vitamin D-resistant rickets.

 

PREFERRED RESPONSE 5

 

The clinical condition is most consistent with vitamin D-resistant rickets.

 

Children with vitamin D-resistant rickets are short in stature and have genu varum, physeal widening, and generalized osteopenia. The abnormality in inherited vitamin D-resistant rickets is the renal tubule’s inability to resorb phosphate leading to hypophosphatemia. Laboratory findings in the condition are normal or near normal serum calcium values, significantly decreased serum phosphate levels, elevated alkaline phosphatase levels, and normal PTH and vitamin-D levels. The most common form is inherited as an X-linked dominant trait.

Illustration A is chart that shows the lab values of different types of Rickets. Incorrect Answers:

Answer 1: Nutritional rickets has a normal or low serum phosphate levels, normal or low serum calcium values, and decreased levels of 25(OH) vitamin D and 1,25-dihydroxyvitamin D.

Answer 2: Children with renal osteodystrophy tend to have genu valgum, and laboratory findings include elevated serum phosphate, alkaline phosphatase, and PTH levels, and low serum calcium values. Findings of renal disease include elevated BUN and creatinine.

Answer 3: Primary hyperparathyroidism usually is caused by a parathyroid adenoma, and the child generally has abdominal problems and hypercalcemic crisis. Laboratory findings include elevated serum calcium values, alkaline phosphatase levels, and PTH levels, and decreased serum phosphate levels. Answer 4: Hypophosphatasia is a rare condition characterized by a deficiency of

alkaline phosphatase in the serum and tissues, leading to generalized abnormal mineralization of bone.

 

(SAE08UE.16) The condition shown in Figures 9a and 9b is most likely the result of 

 

 

 

  1. infection.

  2. uric acid deposition.

     

  3. trauma.

     

  4. a virus.

     

  5. severe cold exposure.

 

PREFERRED RESPONSE 2

 

The clinical photograph and radiograph show gout, which is the result of urate deposition in the joint and soft tissues. Radiographs frequently reveal periarticular erosions. The crystals are intracellular and negatively birefringent under the polarized microscope. Treatment for acute flares include colchicines, indomethacin, and corticosteroids (including injections). Medications such as allopurinol help prevent recurrent flares. Tophi such as that seen in this patient are often confused with and associated with infection.

 

 

(SAE07HK.56) Figure 33 shows the venogram of a patient who has a long history of alcohol abuse. Warfarin should be used cautiously because of the interaction with which of the following factors? 

 

 

 

  1. IV

  2. V

     

  3. VI

     

  4. VII

     

  5. VIII

 

PREFERRED RESPONSE 4

 

Warfarin acts by inhibiting clotting factors II, VII, IX, X. The actual mechanism of action is by inhibition of hepatic enzymes, vitamin K epoxide, and perhaps vitamin K reductase. This inhibition results in lack of carboxylation of vitamin K-dependent proteins (II, VII, IX, X). The anticoagulant effect of warfarin can be reversed with vitamin K or fresh-frozen plasma. The use of alcohol may lead to liver dysfunction and an even more limited margin of available factors.

 

(OBQ14.106) Which of the following classes of antibiotics works by binding to the 30S-ribosomal subunit? 

 

  1. Aminoglycosides

     

  2. Penicillins

     

  3. Rifampin

     

  4. Macrolides

     

  5. Quinolones

 

PREFERRED RESPONSE 1

 

Aminoglycosides work by inhibiting peptide elongation by binding to the 30S-ribosomal subunit.

 

Aminoglycosides are among the oldest classes of antibiotics. They are act by binding to the 30S ribosomal subunit and are considered bactericidal. Due to their effectiveness on Gram-negative bacteria they are often used in conjunction with cephalosporins for treatment of open fractures. Care must be taken when using aminoglycosides due to their potential nephrotoxicity and ototoxicity.

 

Mader et al. present an instructional course lecture reviewing common antibiotics and their mechanisms of action. For aminoglycosides, they comment that their primary use is for aerobic Gram-negative organisms, particularly enterobacter species and P. aeruginosa. Aminoglycosides have realtively poor activity against Gram-positive organisms and should not be used for staph or strep species.

Illustration A is a diagram showing the mechanism of action of different antibiotics. Incorrect Answers:

Answer 2: Penicillins work by inhibiting bacterial cell wall synthesis. Answer 3: Rifampin inhibits RNA polymerase Answer 4: Macrolides bind the 50S-ribosomal subunit Answer 5: Quinolones inhibit DNA gyrase

(SAE13BS.47) The initiating cellular event in development of posttraumatic osteoarthritis is attributed to which of the following? 

 

  1. Chondrocyte aging as the result of matrix degradation

     

  2. Chondrocyte death from apoptosis

     

  3. Cysteine protease-inhibited chondrocyte destruction

     

  4. Interleukin-2-mediated chondrocyte hypertrophy

 

PREFERRED RESPONSE 2

 

A relatively large percentage of patients sustaining intra-articular fractures develop posttraumatic arthritis despite surgical restoration of joint incongruity and alignment. Fracture-related chondrocyte death (apoptosis) concentrated along matrix cracks in the superficial layer of cartilage has been linked to the pathogenesis of posttraumatic osteoarthritis. Apoptosis is accentuated by a series of aspartate-specific cysteine proteases. Inhibition of this cascade is a target of emerging pharmacological treatment options.

 

(SAE10BS.88) Genetic analysis has revealed a strong linkage between osteoarthritis in women with which of the following molecules? 

 

  1. TGF-beta

     

  2. Collagen type IX

     

  3. Bone sialoprotein

     

  4. Hyaluronic acid

     

  5. Basic fibroblast growth factor

 

PREFERRED RESPONSE 2

 

It is believed that both genetic and environmental factors play a role in the development of osteoarthritis (OA). Recent studies revealed that several genetic predispositions may be contributing to the disease process. A recent genetic analysis

of over 400 families showed that type IX collagen is linked to development of OA in women. Mice with a type IX collagen gene deletion are also susceptible to early arthritis. Other genes, such as cartilage enzyme aggrecanase ADAMTS-5, have also been associated with OA.

 

(SAE10BS.18) Corticosteroids inhibit the formation of effectors of the inflammatory pathway via inhibition of 

 

  1. phospholipase A2.

     

  2. lipoxygenase.

     

  3. cyclooxygenase-1.

     

  4. thromboxane A2.

     

  5. prostacyclin.

 

PREFERRED RESPONSE 1

 

Corticosteroids prevent the formation of the pre-inflammatory mediator arachidonic acid from cell membrane phospholipids by inhibiting phospholipase A2. This inhibition prevents the earliest step in the inflammatory cascade. Arachidonic acid proceeds through the lipoxygenase pathway to produce leukotrienes, responsible for autocrine and paracrine signaling, or through the cyclooxygenase pathway (either COX-1 or COX-2) to produce prostaglandins. Thromboxane A2, present in platelets, activates phospholipase C which causes cellular effects resulting in platelet aggregation. Prostacyclin and thromboxane A2 work as antagonists. Prostacyclin is released from endothelial cells, inhibiting platelet aggregation and causing vasodilation.

 

(SAE13BS.8) A surgeon decides to report outcomes for a new surgical procedure that he has performed on 10 patients who have a rare type of arthritis. He provides data on

the functional and subjective patient outcomes. This type of study design is best described as a 

 

  1. case series.

     

  2. case-control study.

     

  3. cohort study.

     

  4. randomized clinical trial.

 

PREFERRED RESPONSE 1

 

The type of study design in which a series of cases is presented with outcomes (without a control population or comparison group) is known as a case series. This type of study design, although frequently seen in orthopaedic literature, provides the lowest level of evidence. There is no control group and the population is usually poorly defined. This type of study can be helpful as a starting point for further analysis. A randomized trial provides the highest level of evidence in medical research, featuring a comparison group and randomized (and usually blinded) placement of subjects into study groups. In case-control studies, cases are compared to a control group. The control group has not been randomized, but may be a naturally occurring group of subjects who have not had the same exposure or intervention as the case group. A cohort study can be retrospective or prospective and usually looks at a large group of people over time to assess exposures and incidence of disease.

 

(SAE10BS.48) Receptor activator of nuclear factor kappa b (RANKL) and macrophage colony stimulating factor (MCSF) signaling pathways are necessary for the formation of multinucleated osteoclasts that resorb bone. Which of the following cells are known to produce RANKL? 

 

  1. Macrophages

     

  2. Osteoclasts

     

  3. Osteoblasts

     

  4. Monocytes

     

  5. Histiocytes

PREFERRED RESPONSE 3

 

Osteoclast differentiation and function depend on the establishment of specific patterns of gene expression achieved through networks of transcription factors activated by osteoclastogenic cytokines such as RANKL and MCSF. RANKL and MCSF are produced by osteoblasts and T cells. Key transcriptional factors responsible for osteoclatogenesis require activation of transcriptional factors such as PU.1, NF-kappaB, AP-1, NFATc1, Mitf, Myc, and Src in osteoclast precursors that are of monocyte/macrophage origin.

 

(SAE07PE.21) What is the most important consideration in the preoperative evaluation of a child with polyarticular or systemic juvenile rheumatoid arthritis (JRA)? 

 

  1. Cervical spine assessment

     

  2. Temporomandibular joint (TMJ)/jaw assessment

     

  3. Dental assessment

     

  4. Stress dosing with corticosteroids

     

  5. Opthalmology examination

 

PREFERRED RESPONSE 1

 

The cervical spine may be involved in a child with polyarticular or systemic JRA; fusion or instability can occur. Radiographic assessment of the cervical spine should include lateral flexion-extension views. The potential exists for spinal cord injury during intubation or positioning in the presence of an unstable cervical spine. Limitations of the TMJ and micrognathia may affect ease of intubation and administration of anesthesia via a mask. If the TMJ and jaw are involved, some patients may have dental findings such as dental caries and even abscesses which can affect surgery. Some children, particularly those with systemic arthritis, may be taking corticosteroids long-term and may need stress dosing with complex surgeries. Although it is important to routinely check for uveitis and iritis in children with JRA,

this usually is not needed preoperatively. Uveitis and iritis are less likely in a child with systemic JRA.

 

(SAE10BS.3) A surgeon is planning the application of a single plane external fixation construct as provisional fixation for an unstable knee dislocation. There are two pin sizes available, one with twice the diameter of the other. The bending stiffness of the larger pin is greater than that of the smaller by a factor of which of the following? 

 

1 2

 

2 4

 

3 8

 

4 16

 

5 32

 

PREFERRED RESPONSE 4

 

The increase in bending stiffness is proportional to the radius to the 4th power, so a doubling results in an increase by a factor of 16.

 

(SAE11OS.156) A prospective outcome study is performed at a single institution to analyze the potential differences in treating intertrochanteric hip fractures with a plate/screw device versus an intramedullary device. No specific randomization is performed because an equal number of surgeons have preferences for the use of one of these devices and they are allowed to continue their preferred method. Hip-specific and general health-related outcome measures are used, an excellent follow-up rate of 85% of the patients at 2 years is accomplished, and there appears to be results that favor the intramedullary device but the confidence intervals are wide. This study would be considered to carry what level of evidence? Review Topic

  1. I

     

  2. II

     

  3. III

     

  4. IV

     

  5. V

 

PREFERRED RESPONSE 2

 

This is a prospective comparative study but is not randomized or blinded and is therefore a Level II therapeutic study. To qualify as Level I, it would need to be a high-quality randomized trial with narrow confidence intervals regardless of a significant difference or no difference in outcomes. Level III would be case-control studies or retrospective comparisons. Level IV is case series and Level V is expert opinion.

 

(SAE13BS.95) Bacterial resistance to antibiotics in biofilm is an example of Review Topic

 

  1. avoidance.

     

  2. decreased susceptibility.

     

  3. inactivation.

     

  4. mutation.

     

    PREFERRED RESPONSE 1

     

    Three basic mechanisms of antibiotic resistance have been identified: avoidance, decreased susceptibility, and inactivation. Biofilm formation is a classic example of avoidance, whereby the biofilm creates a physical barrier to the antibiotic. Bacteria can decrease their susceptibility to antibiotics by mutating the antibiotic target or generating a mechanism to inactivate the antibiotic. Biofilm formation develops when a sufficient mass of bacteria forms on a surface. The cell-to-cell signaling becomes sufficient to activate transcription of genes needed for biofilm formation in a process

    known as quorum sensing. Once the bacteria produce a mature biofilm, they enter a greatly reduced or stationary phase of growth. Lastly, high-shear environments seem to stimulate biofilm production.

     

    (SAE10BS.52) What percentage of bone weight is collagen? 

     

    1 5% to 10%

     

    2 20% to 25%

     

    3 50% to 60%

     

    4 70% to 75%

     

  5. 90% to 95%

 

PREFERRED RESPONSE 2

 

Bone is a composite of both inorganic and organic material. The inorganic component of bone comprises 60% to 70% of the tissue, water accounts for 5% to 8%, and the organic matrix makes up the remainder. Collagen accounts for 90% of the organic component and thus 20% to 25% of bone weight. Collagen accounts for the flexibility of bone. The inorganic component of bone is made primarily of calcium and phosphorous, in the analogue of hydroxyapatite, and other ions including sodium, magnesium, and carbonate.

 

(SAE11UE.32) A 65-year-old woman with rheumatoid arthritis is unable to actively extend her index, middle, ring, and little fingers secondary to tendon rupture. In performing a flexor digitorum sublimis (FDS) of the middle/ring finger to extensor digitorum communis (EDC) transfer to restore active metacarpophalangeal (MCP) joint extension, the FDS should be passed 

  1. ulnarly, around the ulna in a dorsal direction.

     

  2. radially, around the radius in a dorsal direction.

     

  3. through the interosseous membrane.

     

  4. through the intermetacarpal spaces between the index, middle, ring, and little fingers.

     

  5. through the lumbrical canals of the index, middle, ring, and little fingers.

 

PREFERRED RESPONSE 2

 

Although the early use of FDS as a transfer to restore finger extension in patients with radial nerve palsy was performed by passing the tendon through the interosseous membrane, Nalebuff and Patel later modified this procedure for the rheumatoid arthritis patient by passing the FDS radially, around the radius in a dorsal direction. They felt that this provided a number of advantages, including: 1. technical ease, 2. avoidance of synovial disease on the dorsum of the wrist, and 3. correction of ulnar deviation of the fingers through the line of pull from the radial side of the forearm.

 

(OBQ15.250) Induction coupling stimulates bone growth through all of the following direct effects EXCEPT: 

 

  1. Increased proliferation of osteoblasts

     

  2. Decreased osteoclast differentiation

     

  3. Increase release of TGF-beta1

     

  4. Increased expression of BMP2

     

  5. Increased expression of BMP7

 

PREFERRED RESPONSE 2

 

Induction coupling stimulates bone growth by increasing expression of BMP7, BMP2, TGF-beta1, and by increasing osteoblasts proliferation. Induction coupling has not been shown to have the effect of decreasing osteoclast differentiation.

In basic science studies, electrical stimulation (i.e. induction coupling), has been shown to promote bone healing via release of growth factors that induce osteoblast differentiation/proliferation. Electrical current can be placed around bone in various ways, creating a current to stimulate growth factor release and subsequent osteoblast proliferation.

 

Aaron et al. summarized, in a systematic review, the effects of various types of electrical stimulation on bone and bone healing. Regardless of type (i.e. inductive coupling, capacitive coupling, direct current), they report electricity and/or electromagnetic fields promote gene expression of growth factors that promote an osteogenic environment.

 

Illustration A depicts a cathode placed directly to allograft with a subcutaneously placed electrical stimulator. Illustration B depicts cathodes placed anteriorly and posteriorly around the hip, connected to an outside power source to create the necessary current. Illustration C depicts an inductive coil placed laterally on the skin in order to create an electrical current.

 

Incorrect answers:

Answers 1,3-5: All are true effects of electrical stimulation on bone.

 

(OBQ07.134) What medication has been shown to decrease osteolysis after total joint replacement surgery? 

 

  1. Bisphosphonates

     

  2. NSAIDs

     

  3. TNF-alpha inhibitors

     

  4. Calcium and vitamin D supplementation

     

  5. BMP-7

 

PREFERRED RESPONSE 1

 

Bisphosphonates have been shown to decrease osteolysis after total joint replacement surgery.

 

Aseptic loosening and osteolysis are the primary causes of implant failure in total

joint arthroplasty. Early findings indicate that bisphosphonates upregulate bone morphogenetic protein-2 production and stimulate new bone formation, leading to decreased osteolysis in total joint replacement surgery. While further investigation is required, bisphosphonates may play a future role in improving the long-term duration of joint arthroplasties.

 

Shanabhag et al. reviewed the use of bisphosphonates and reported that they had the potential to enhance bone ingrowth into implant porosities, prevent bone resorption under adverse conditions, and dramatically extend the long-term durability of joint arthroplasties. They recommended further investigation into the subclasses to determine which ones are most beneficial.

 

Arabmotlagh el al. performed a prospective study on use of alendronate after total hip arthroplasty. They reported that the alendronate-treated patients had significantly less periprosthetic bone loss on DXA scans after 6 years.

Illustration A shows evidence of osteolysis (arrows) around a total hip arthroplasty. Incorrect Answers:

2-5: These medication classes do not decrease osteolysis after total joint arthroplasty.

 

(OBQ14.166) A funnel plot is used in meta-analyses to perform which of the following functions: 

 

  1. Illustrate the relative strength of treatment effects in multiple studies

     

  2. Detect publication bias

     

  3. Graph of the sensitivity versus 1-specificity of a diagnostic test

     

  4. Determine the sample size required to detect an effect of a given size with a given degree of confidence

     

  5. Predict the unknown value of a variable from the known value of two or more variables

 

PREFERRED RESPONSE 2

 

A funnel plot is the most commonly used statistical test for detection of publication bias in meta-analyses.

 

Publication bias occurs because studies with a non-significant result, so-called

negative studies, have a higher likelihood of being rejected than positive studies, and are oftentimes not even submitted for publication. Funnel plots, which plot the effect size of a study against a measure of the study’s size are used to detect this bias. This method is based on the fact that larger studies have smaller variability, whereas small studies, which are more numerous, have larger variability. Thus the plot of a sample of studies without publication bias will produce a symmetrical, inverted-funnel shaped scatter, whereas a biased sample will result in a skewed plot.

 

Vavken et al. reviewed orthopaedic meta-analyses in order to determine whether publication bias was assessed and to evaluate its effect on the outcomes of these meta-analyses. They found that only 35% of all orthopaedic meta-analyses published between 1992 and 2008 in English and German assessed publication bias. Adjustment for publication bias did not produce significantly different results, but the magnitude of the pooled estimates in the affected meta-analyses changed by 29% on average.

 

Illustration A depicts a symmetrical funnel plot with no evidence for publication bias. Illustration B shows a skewed funnel plot suggesting publication bias, as it is missing studies in the lower left corner, i.e. ‘‘negative studies’’. Illustration C depicts a forest plot comparing the incidence of squeaking between ceramic-on-ceramic (COC) and ceramic-on-polyethylene (COP). Illustration D is an example of a ROC curve examining the probability of DVT.

 

Incorrect Answers:

Answer 1: Forest plots illustrate the relative strength of treatment effects in multiple studies, NOT funnel plots. Answer 3: A graph of the sensitivity versus 1-specificity of a diagnostic test is referred to as a receiver operating characteristic (ROC) curve and is typically used to determine the accuracy of diagnostic tests. Answer 4: A power analysis is used to determine the sample size required to detect an effect of a given size with a given degree of confidence prior to the start of a clinical study.

Answer 5: A multiple regression analysis is used to predict the unknown value of a variable from the known value of two or more variables in a study.

 

(SAE08OS.151) A deficiency of which of the following is associated with an increased risk of venous thromboembolism? 

 

  1. Fibrinogen

     

  2. Protein S

     

  3. Factor VII

  4. Factor VIII

     

  5. Prothrombin

 

PREFERRED RESPONSE 2

 

Protein C and Protein S are endogenous proteins whose action is to inhibit the coagulation cascade. Deficiencies in these proteins are a risk factor for the development of thrombus. Prothrombin is the precursor to thrombin, which is the final common pathway for both the intrinsic and extrinsic coagulation cascade. Thrombin acts to convert fibrinogen to fibrin and thus clot formation.

 

(SAE13BS.45) In articular cartilage, Interleukin 1 (IL-1) increases 

 

  1. matrix metalloproteinase.

     

  2. proteoglycan synthesis.

     

  3. collagen production.

     

  4. nitric oxide synthetase.

 

PREFERRED RESPONSE 1

 

IL-1 stimulates matrix metalloproteinase that directly degrade cartilage. It also stimulates enzymes such as cyclooxygenase 2 and nitric oxide synthetase, which further cause tissue catabolism and damage. Glucosamine has anabolic effects on proteoglycan synthesis and can also prevent tissue catabolism by preventing an IL-1 beta-induced decrease in proteoglycan synthesis. Transforming growth factor beta has chondroprotective functions and has been shown to increase both collagen and proteoglycan synthesis while inhibiting matrix degradation and cell proliferation. Insulin-like growth factor 1 (IGF1) is produced by articular chondrocytes and increased collagen and proteoglycan synthesis. It has a role in the development of osteoarthritis. Decreased expression of IGF1 and increased binding proteins decrease the availability of the growth factors, accelerating tissue catabolism in arthritic cartilage.

 

 

(SAE10BS.72) Intermittent daily administration of recombinant parathyroid hormone (rhPTH) is an FDA-approved treatment for osteoporosis. Intermittent rhPTH treatment targets which of the following cells in osteoporotic patients? 

 

  1. Osteoclasts

     

  2. Macrophages

     

  3. Chondrocytes

     

  4. Osteoblasts

     

  5. Endothelial cells

 

PREFERRED RESPONSE 4

 

PTH is an anabolic agent that enhances osteoblastic bone formation on both cortical and cancellous surfaces. It is synthesized in the parathyroid glands as a 115 amino acid precursor and is cleaved into the active 84 amino acid form. The biological activity in the clinically used recombinant PTH is in the 1-34 amino acid sequence at the N-terminus of the molecule. There are PTH receptors expressed by osteoblasts that mediate the anabolic bone response to intermittent PTH administration. Chronic elevation of PTH leads to stimulation of osteoclasts, producing bone loss.

 

(SAE08UE.68) What is the primary indication for performing a total wrist arthroplasty in a patient with painful rheumatoid arthritis? 

 

  1. Ipsilateral total elbow arthroplasty

     

  2. Contralateral wrist arthrodesis

     

  3. Type III degenerative changes of the wrist

     

  4. Age older than 55 years

  5. Less than 30 degrees of wrist flexion/extension

 

PREFERRED RESPONSE 2

 

The most conservative indications for a total wrist arthroplasty are to spare motion on one side and to improve activities of daily living. Component loosening, dislocation, and wound problems are frequent. Suitable patients can be of various ages, wrist motion, and radiographic stages of arthritis. Ipsilateral total elbow arthroplasty, type III degenerative changes of the wrist, age older than 55, and limited range of motion are neither primary indications nor contraindications to a total wrist arthroplasty.

 

(OBQ14.155) What term in statistics defines rejecting the null hypothesis when it is in fact true? 

 

  1. Type-I error

     

  2. Type-II error

     

  3. Confounding error

     

  4. Variance

     

  5. Negative likelihood ratio

 

PREFERRED RESPONSE 1

 

Rejecting the null hypothesis when it is true is an example of a type-I error.

 

Type-I errors, to put it simply, detect an effect that is not present. In contrast, a type-II error fails to detect an effect that is present. In simple studies, the rate of a type-I error is denoted by a (alpha). For a 95% confidence level, the value of alpha is 0.05. This means that there is a 5% probability that we will reject a true null hypothesis.

Illustration A shows the difference between type-I and type-II errors. Incorrect Answers;

Answer 2: A type-II error accepts the null hypothesis when it should be rejected. Answer 3: A confounder is a variable that has associations with both the dependent and independent variables, potentially distorting their relationship. Confounders are

not technically considered "errors," but instead are variables that properly constructed studies attempt to avoid. Answer 4: Variance is an estimate of the variability of each individual data point from the mean.

Answer 5: Negative likelihood ratio describes how the likelihood of a disease is changed by a negative test result.